Download as pdf or txt
Download as pdf or txt
You are on page 1of 566

Table of Contents

Title page
Key to Symbols
Preface by Veselin Topalov

Chapter 1 – Topalov: A World Apart


Chapter 2 – The Start of Our Cooperation
Chapter 3 – Learning the job
Part 1 Part 2
Chapter 4 – London: first sucess!
Chapter 5 – Zug: more success!
Part 1 Part 2
Chapter 6 – Tough times in Thessaloniki
Chapter 7 – Rollercoaster in Beijing!
Part 1 Part 2
Chapter 8 – Preparing for the Candidates
Chapter 9 – The Candidates Tournament
Part 1 Part 2 Part 3 Part 4
Chapter 10 – A Few Novelties More

Exercises – Win Like Topalov!

2
My Magic Years with Topalov

by
Romain Edouard

Thinkers Publishing 2019

www.thinkerspublishing.com

First edition 2019 by Thinkers Publishing


Copyright © 2019 Romain Edouard
All rights reserved. No part of this publication may be reproduced, stored in a retrieval system or
transmitted in any form or by any means, electronic, mechanical, photocopying, recording or otherwise,
without the prior written permission from the publisher.
All sales or enquiries should be directed to Thinkers Publishing, 9850 Landegem, Belgium.
Email: info@thinkerspublishing.com
Website: www.thinkerspublishing.com

Managing Editor: Romain Edouard


Assistant Editor: Daniël Vanheirzeele
Software: Hub van de Laar
Proofreading: Ian Marks, Roger Emerson
Graphic Artist: Philippe Tonnard
Cover Design & Cartoons: Mieke Mertens
Back cover photo: Sabrina Chevannes
Production: BESTinGraphics
ISBN: 9789492510471
D/2018/13730/29

3
Key to Symbols

! a good move
? a weak move
!! an excellent move
?? a blunder
!? an interesting move
?! a dubious move
™ only move
N novelty
‰ Lead in developpment
ʘ Zugzwang
= equality
∞ unclear position
© with compensation for the sacrificed material
² White stands slightly better
³ Black stands slightly better
± White has a serious advantage
µ Black has a serious advantage
+– White has a decisive advantage
–+ Black has a decisive advantage
‚ with an attack
ƒ with an initiative
„ with counterplay
… with the idea of
¹ better is
≤ worse is
+ check
# mate

4
Preface

by Veselin Topalov

Photo Jean-Michel Péchiné (Europe Échecs)

In the second half of 2010 my manager Silvio Danailov told


me he had received an email from a young French GM,
offering his cooperation. We asked him to send an example
of how he works, an analysis of some opening variation.
Romain sent a file with an improvement in the Najdorf and I
liked it. In my experience a young player willing to work
hard is more valuable than a renowned but unmotivated
GM, regardless of playing strength. Later I met Romain
personally during the Chess Olympiad in Khanty-Mansiysk
and we agreed on a training session in Salamanca.
Looking back, I would say our first sessions were the most
fruitful. That’s logical, as in 2010 it was easier for humans
to find good ideas more quickly than computer engines than
it was in 2014, the same way that in 2018 it is easier than it
will be in the years to come.
Of all our discoveries, I am most proud of the move 14.Ba4!
played against Nakamura during the Melody Amber rapid
and blitz tournaments in Monaco, a very deep idea which
was very well developed by Romain. Lots of beautiful lines
have still to be revealed. Also the strong novelty 12.Ne5!
from the last round of the same event against Aronian gave me a big advantage, although the result was
a loss.
Often, the quality of the work of trainers or seconds has nothing to do with the final result. I don’t think
they’re responsible for what happens during time trouble in a blindfold game or in the fifth hour of a
long and tense struggle.
What was also very important was that we had the good luck to anticipate the new move 9.c5!? played
by Morozevich in Zug in 2013, knowledge of which gave me an important point. So an otherwise
original and fresh idea turned out a fiasco for my opponent. Another excellent discovery in Lanzarote
was 9.Be5!?, played against Kramnik. This surprise allowed me to get a huge advantage only a few
moves later, something almost impossible in such a well-studied opening as the Queen’s Gambit.
There were many other improvements and some of them have still not been played.
Our last serious training was in the winter of 2014 when I prepared for the Candidates Tournament in
Khanty-Mansiysk. I believe I prepared very well, but the problem was that results came much later than
needed. However, as I’ve already said, I don’t think Romain can be blamed for my loss against Svidler

5
with the black pieces, for example. We found a great idea, revived a line with a bad reputation and I got
an edge out of the opening, only to spoil it completely and lose. As a result the Candidates was a
disaster me, but I would say my good play in the tournaments which followed in 2014 and 2015 was
largely due to the work we did in Lanzarote in the winter of 2014. For example the new move 7...g5!?,
which gave me victory over Carlsen in Saint Louis a year and a half later in 2015, was prepared back
then.
Romain crossed the 2700 benchmark in June 2014; I would like to think it was because he worked with
me, not because he stopped working with me J.
This book is about how ideas are born during preparation, about what happens behind the scenes. Until
Romain told me he was writing it I had no idea of his plans. I believe chess fans will like the honesty of
the stories as nothing is hidden. I hope you enjoy it.

Veselin Topalov
Salamanca, October 2018

6
Chapter 1
Topalov: A World Apart

For the purpose of writing this book I decided to look at all the games Veselin has played from 1995
until the present, as there were many I didn’t know! I must say that, although seeing great moves from
a 2800 player sounds normal, it was impossible not to be astonished by some of his games.
Topalov is one of the kings of practical decisions in chess. He regards chess as more a sport than a
science. If he thinks an idea will work over the board, the notion of risk is irrelevant to him.
He wants to be on the attack and believes an objectively inferior position isn’t necessarily bad if his
opponent needs to find several difficult defensive moves. “If that’s the only move for my opponent,
let’s enter the line and see if he sees it!” is his philosophy.
He never liked peace over the board or routine play. The moments where he has refused to repeat
moves or has sacrificed something strictly out of intuition are countless.
In short, Topalov’s aim has always been to hit hard and bring his own touch to the game, and I think he
has succeeded!
I would like to open this book with what might be the most beautiful game Topalov has ever played: a
masterpiece against Garry Kasparov, at the Euwe Memorial, Amsterdam 1996.
Topalov, Veselin (2700)
Kasparov, Garry (2775)
Amsterdam 1996

1.e4 c5 2.Nf3 d6 3.d4 cxd4 4.Nxd4 Nf6 5.Nc3 a6 6.Bc4 e6 7.Bb3 Nbd7 8.f4 Nc5 9.0-0!?

7
Position after: 9.0-0!?

The sharpest line: White invites Black to take the e4-pawn, otherwise he will play e4-e5. The other
option is 9.Qf3.

9...Ncxe4 10.Nxe4 Nxe4 11.f5 e5 12.Qh5 Qe7

12...d5

Position after: 12...d5

This is the main move here nowadays. Two days after this game Nigel Short used this improvement to
beat Topalov himself: 13.Re1 Bc5 [13...Qc7!? 14.Bxd5 Nf6 15.Bxf7+ Qxf7 16.Rxe5+ Be7 17.Qe2
Kf8 18.Bg5 has also been played twice, with enough compensation for the piece, but probably not
more.] 14.Rxe4 Bxd4+ [14...0-0!? 15.Rg4 Bxd4+ 16.Kh1 e4 is also playable.] 15.Be3 0-0 16.Rxd4
exd4 17.Bxd4 f6. White has compensation for the exchange, but isn’t better: 18.Bc5?! Re8 19.Rd1 Kh8
20.Rxd5 Bd7 0-1 (46) Topalov, V (2700) – Short, N (2665) Amsterdam 1996.

13.Qf3

8
Position after: 13.Qf3

13...Nc5

13...exd4? 14.Re1+–

14.Nc6!? Qc7 15.Bd5!

Nowadays engines find this move quickly, but they weren’t so helpful back in 1996!
15.Nb4? Nxb3 16.Qxb3 b5! 17.Nd5 Qc6 followed by ...Bb7 would be excellent for Black.

15...a5

The human move, trapping the knight on c6.


After 15...Bd7 Black’s life would be no fun: 16.Nb4

9
Position after: 16.Nb4

16...Be7 [16...Qb6? 17.Bxf7+!! Kxf7 18.Nd5 Qc6 19.Qh5+ Kg8 20.f6+–] 17.Qh5! [17.Bxf7+? Kxf7
18.Nd5 Qd8 19.Qh5+ Kg8–+; 17.Bc4 Bc6 18.Nd5 Bxd5 19.Bxd5©] 17...Rf8™ [17...0-0 18.Rf3 Qc8
19.Bh6! Bf6 20.Raf1 …Rg3 with a winning attack.] 18.Be3 and White has huge compensation for the
pawn.

16.Bg5!

Position after: 16.Bg5!

10
The key move. White wants to play Nd8! Of course Black can’t get rid of the g5-bishop by playing ...f6
because of Qh5+.

16...Ra6?

16...Bd7 was the only move, though if White finds the most challenging reply, Black needs a heroic
defence.
A) 17.Bxf7+? just fails: 17...Kxf7 18.Qh5+ Kg8 19.f6 Be8 0-1 Degraeve, J (2551) – Ninov, N (2500)
Béthune 2006.
B) 17.f6?! doesn’t make Black’s life particularly difficult: 17...g6! [17...Bxc6? 18.fxg7 Bxg7
19.Bxf7+ wins easily.] 18.Ne7 Ne6!

Position after: 18...Ne6!

and Black wants to go ...Bxe7 and ...f5. Of course White has some compensation, but the game is very
unclear: either side could lose within a few moves! For example 19.Bh4 [19.h4 Bxe7 20.fxe7 f5
21.Bf6 Qc5+ 22.Rf2 Nd4 23.Qd1 Qxd5 24.Bxh8 Kxe7³] 19...Bxe7 [19...g5!? 20.Bxe6 Bxe6
21.Bxg5 Bxe7 22.fxe7 Rg8∞] 20.fxe7 f5∞.
C) 17.Ne7!N Wow! Scary stuff, but Black is fine with accurate play.
C1) 17...f6? 18.Qh5+ Kxe7 [18...Kd8 19.Ng6 Be8 20.Be3±] 19.Bxf6+! [19.Qf7+?? Kd8 20.Bxf6+
Kc8–+] 19...gxf6 [19...Kxf6 20.Qh4+ g5 21.fxg6+ Kg7 22.Rf7++–] 20.Qf7+ Kd8 21.Qxf6+ Kc8
22.Qxh8 Qd8 23.f6 is crushing for White.
C2) 17...Bxe7 18.Bxe7 f6! [18...Kxe7 19.f6+ Kd8 20.fxg7 Re8 21.Bxf7+–; 18...Qb6 19.f6‚]
19.Qg4

11
Position after: 19.Qg4

19...Be6!! and Black holds: 20.Bxe6 [20.Bxd6 Qxd6 21.Bxe6 Nxe6 22.fxe6 0-0=] 20...Qxe7
21.Qh5+ Kd8 22.Bd5 Rc8!? White has compensation, but Black is probably not worse as he will
play ...Kc7 and ...Kb8. When White plays a3 trying to push b4, Black answers ...a4, and it is not
easy to ‘kill’ Black.

17.Nd8!!+–

Position after: 17.Nd8!!+–

12
Black is already lost!

17...f6™ 18.Nf7 Rg8 19.Be3 g6 20.Ng5!

Position after: 20.Ng5!

What a magical knight!

20...Rg7

20...fxg5 21.f6! Rh8 22.f7+ Kd8 23.Bxg5++–

21.fxg6 Rxg6 22.Bf7+

Black can do nothing but sacrifice his queen. It took White some time to convert, which is normal, but
the rest of the game isn’t too relevant.
I give it here just in case you would like to see it to the end!

22...Qxf7 23.Nxf7 Kxf7 24.Bxc5 dxc5 25.Rad1 Be7 26.Rd5 Bg4 27.Qe4 Kg7 28.Rfd1 Bxd1 29.Rxd1
Re6 30.Qf5 Kf7 31.Re1 b6 32.h4 Rg7 33.Kf1 Bd6 34.Kf2 Bc7 35.Kf3 Ke7 36.Re4 Kf7 37.Rg4 Re7
38.Ke4 Rxg4+ 39.Qxg4 Bd8 40.a4 Kf8 41.c3 Rg7 42.Qc8 Ke8 43.Qe6+ Kf8 44.g4 Rf7 45.h5 Rg7
46.h6 Rg6 47.Qd5 Be7 48.Kf5 Rxh6 49.Qb7 e4 50.Qb8+ Kf7 51.Qxb6 e3 52.Qe6+ Ke8 53.Qxe3
Rg6 54.Qe4 Rg5+ 55.Kf4 Kd7 56.Qb7+ Ke6 57.Qc8+ Kf7 58.Qc7 h5 59.gxh5 Rxh5 60.Qxa5 Bd6+
61.Ke4 f5+ 62.Kd5 Be7 63.Qc7 Rh6 64.a5 Rd6+ 65.Ke5 Rf6 66.Qc8 1-0

This game shows that one can recognize Topalov’s style of play in his opening preparation.

You can imagine what kind of instructions he gives to his seconds: “Be creative and aggressive!”.

In 2005, he became World Champion in San Luis, Argentina. A few months prior to winning the title,
he won a stunning game against Vishy Anand, which is symbolic of everything Veselin loves about

13
chess: sacrifices, intuition – and blood. It gives me pleasure to continue the book with this game.
Topalov, Veselin (2778)
Anand, Viswanathan (2785)
Sofia 2005

1.d4 Nf6 2.c4 e6 3.Nf3 b6 4.g3 Ba6 5.b3 Bb4+ 6.Bd2 Be7 7.Nc3 c6 8.e4 d5 9.Qc2 dxe4 10.Nxe4
Bb7

Position after: 10...Bb7

11.Neg5!?

An absolute novelty at the time – and quite a bit of bluff since it has only been repeated twice! Typical
Topalov!

11...c5

11...h6? is exactly what White was hoping for: 12.Nxf7! Kxf7 13.Ne5+ Kg8 14.Bh3 Bc8 15.0-0-0©
(or even just 15.0-0) and White has both dynamic and positional compensation: look at Black’s
development!

12.d5 exd5 13.cxd5

14
Position after: 13.cxd5

Here Black has already to make an important decision: and Anand made the right one!

13...h6!

A) 13...0-0? would already be the decisive mistake: 14.0-0-0 h6 [14...Bxd5? 15.Bc3 g6 16.Bc4+–]
15.h4!

Position after: 15.h4!

15
A very important idea that became possible as a result of Black making the mistake of castling.
15...Nbd7 [15...hxg5 16.hxg5 Nxd5 17.Qh7#] 16.Bc3 and with the d6-threat and Bb5 coming next,
targeting the d7-knight, the black position is on the brink of collapse.
B) 13...Nxd5? 14.Bb5+ Kf8 [14...Bc6 15.Nxf7 Kxf7 16.Ne5+] 15.Nxf7! [15.0-0©] 15...Kxf7
16.Qf5+ Bf6 17.Ng5+ Kg8 [17...Kf8 18.Ne6++–] 18.Qe6+ Kf8 19.Qf7#
C) 13...Bxd5? 14.0-0-0!

Position after: 14.0-0-0!

is quite bad for Black: 14...Qc8 At least escaping from the d-file. [Just giving up the bishop is not
enough either: 14...Bxf3 15.Nxf3 0-0 16.Bc3 Qe8 17.Ng5 g6 18.Bg2 Nc6 19.Rhe1 and White wins
material.] 15.Bc3 Bxf3 16.Nxf3 Nc6 17.Re1 and Black’s position is a nightmare, since 17...0-0?! is
met by 18.Ng5 g6 19.Rxe7 Nxe7 20.Bxf6+–.
D) 13...Qxd5?? loses right away to 14.Bc4+–.

14.Nxf7!?

14.Bb5+ Nbd7 15.Ne6 fxe6 16.Qg6+ Kf8 17.dxe6 Qe8!=

14...Kxf7 15.0-0-0

16
Position after: 15.0-0-0

15...Bd6

The most logical move, avoiding Ne5+ and also d5-d6 which is coming soon, if not immediately.
A) 15...Bxd5? 16.Bc3+–
B) If 15...Nbd7 White could play 16.Bf4!? with the idea d5-d6. Black would have to continue 16...g5
17.Bxg5! when it definitely looks bad for him.

16.Nh4! Bc8

A strange-looking move to say the least, but the threat of Bh3 was just too troublesome.
16...Nbd7 17.Bh3 Nf8 [17...Bxd5? is still impossible: 18.Rhe1! Re8 19.Qg6+ Kg8 – if 19...Kf8 then
20.Nf5 wins – 20.Bxh6 Bf8 21.Bxg7 Bxg7 22.Nf5+–] 18.Bc3 Kg8 19.Rhe1© would be extremely
difficult to handle for Black.

17.Re1!

17
Position after: 17.Re1!

Again an inspired move, with the idea of going Re6! The engine gives White sufficient compensation
for a draw with various other options (17.Qg6+, 17.Bc4 or even 17.Kb1), but this is definitely the
trickiest continuation for Black.

17...Na6!

Again the best move by Anand. Black develops and aims for counterplay with ...Nb4.
A) Black could also try to develop his kingside first. After 17...Rf8 the game might objectively be a
draw, but look at the kind of defence Black would have to find: 18.Qg6+ [18.Bxh6 gxh6?? 19.Qg6#]
18...Kg8 19.Bxh6 Rf7 [19...Qc7 20.Re6! is worse as White threatens Rxd6.] 20.Re6! (…Nf5)

18
Position after: 20.Re6!

20...Nc6! The only move according to the engine. 21.Nf5 Ne5™ [21...Ne7? 22.Rxe7! Bxe7
23.Nxg7+–] 22.Rxe5!? [22.Qg5 Nf3 23.Qg6 Ne5 is a fun perpetual.] 22...Bxe5 23.Nxg7 Kh8™
24.Qxf7 Qg8 25.Qe7 Qh7™ 26.Bg5 Qe4™ 27.Ne6 Qe1+ and Black has perpetual check.
B) 17...Re8 18.Qg6+ Kg8 19.Bc4 is just bad for Black: the tension on the e-file is not to his
advantage!

18.Re6!!

19
Position after: 18.Re6!!

This rook is untouchable, as the d5-pawn has been for several moves. White will play Bc3 next and
Black’s position will just collapse.

18...Nb4!

Best again!
A) 18...Kg8, pretending that he might want to take the e6-rook, doesn’t help much, as after 19.Bc3
Black would still be very brave to take it: 19...Bxe6? 20.dxe6 Nc7 21.Bc4+–.
B) 18...Nc7 19.Bc4 is also extremely dangerous for Black: White threatens Rxd6 and Bf4, while Bc3
can also be the next move.
C) Of course not 18...Bxe6? 19.dxe6+ and Bxa6 next.

19.Bxb4!?

19.Qg6+!? Kg8 20.Bc4 was definitely the other option.

Position after: 20.Bc4

Black might hold by seeking counterplay on the queenside: 20...b5!? [20...Bxe6? is still wrong: 21.dxe6
Qe8 22.e7+ Nbd5 23.Bxd5+ Nxd5 24.Qxd6+–.] 21.Bxb5 [In case of 21.Rhe1 Black would now be in
time for 21...Bxe6! 22.dxe6 Qe8! and he is fine.] 21...Nbxd5 Now it’s time to take this pawn! Once
again a crazy line follows: 22.Nf5 Bf8™ 23.Rd6 Bxf5™ 24.Rxd8 Bxg6 25.Rxa8 Kh7 and the engine
evaluates the position as equal.

19...cxb4

20
Position after: 19...cxb4

20.Bc4

20.Qc6 would regain some material, but that’s not what White wants: 20...Rb8!? [20...Bxe6 leads to
another crazy line: 21.dxe6+ Kg8 22.e7 Qc7™ 23.Bc4+ Kh7 24.Qf3 Rhg8™ 25.Ng6! and Black is in
trouble, for example 25...b5? 26.Qf5 bxc4 27.Nf8+ Kh8 28.Qh7+! Nxh7

Position after: 28...Nxh7

29.Ng6#.] 21.Rxd6 Bd7 and Black will counterattack on the c-file.

21
20...b5?

Jumping the gun! Black should have kept this key move for later.
20...Kg8! was correct. It seems that Black holds if he can play like an engine: 21.Nf5 [21.Qg6 Bxe6!?
22.dxe6 Rc8 Black is ready to take on c4 with (counter) check whenever White plays e6-e7. 23.Nf5
Qf8 24.Re1 Ne8 25.Kb1 Rxc4 26.bxc4 Be7∞] 21...b5! 22.Rxd6 Qf8 23.Bd3 Bxf5 24.Rxf6 Qxf6
25.Bxf5 Qc3∞.

21.Bxb5 Be7?!

Not a good move, but the position was bad already.


A) 21...Kg8 22.Bc4 and White will play Nf5. Black’s position is a disaster. As I said earlier, by
playing ...b5 too early, Black wasted an important bullet. Now the c4-bishop is entrenched.
B) 21...Nxd5? loses at once: 22.Qg6+ Kg8 23.Re8++–.

22.Ng6 Nxd5

Position after: 22...Nxd5

23.Rxe7+?!

Not spoiling all the advantage, but instead White could have forced Black to resign! My feeling is that
Topalov thought he was winning at once and missed Black’s 25th move.
23.Re5! (…Rxd5) 23...Bb7 24.Qf5+ and for example 24...Bf6 leads to a nice mate: 25.Qe6+ Kxg6
26.Bd3#.

22
23...Nxe7 24.Bc4+

24.Rd1!? was a bit easier: 24...Nxg6 25.Rxd8 Rxd8 26.Bc4+ Kf6 27.Qb2+ Ne5 28.f4+–.

24...Kf6!?

24...Be6

Position after: 24...Be6

25.Nxe7! The right way to win the queen. [25.Bxe6+?! Kxe6 26.Re1+ Kf6 27.Nxe7 Rc8! and Black
has chances.] 25...Bxc4 [25...Qxe7 26.Re1+–] 26.Qf5+! Kxe7 27.bxc4 Qd4 28.Re1+ Kd6 29.Rd1+–

25.Nxh8 Qd4!

25...Qxh8? 26.Rd1 is hopeless for Black.

26.Rd1™

23
Position after: 26.Rd1™

It turns out White wins anyway!


26.Kb1?? Bf5–+

26...Qa1+ 27.Kd2 Qd4+ 28.Ke1 Qe5+ 29.Qe2

29.Kf1?? Bh3+ 30.Kg1 Rxh8–+

29...Qxe2+ 30.Kxe2 Nf5

30...Bg4+ 31.f3 Bxf3+ 32.Kxf3 Rxh8 33.Rd6++–

31.Nf7+–

24
Position after: 31.Nf7+–

White has an extra pawn and the initiative, which Topalov easily converted.

31...a5 32.g4 Nh4 33.h3 Ra7 34.Rd6+ Ke7 35.Rb6 Rc7 36.Ne5 Ng2 37.Ng6+ Kd8 38.Kf1 Bb7
39.Rxb7 Rxb7 40.Kxg2 Rd7 41.Nf8 Rd2 42.Ne6+ Ke7 43.Nxg7 Rxa2 44.Nf5+ Kf6 45.Nxh6 Rc2
46.Bf7 Rc3 47.f4 a4 48.bxa4 b3 49.g5+ Kg7

Position after: 49...Kg7

25
And now a pretty finish...

50.f5! b2 51.f6+ Kh7 52.Nf5!

Black resigned as he gets mated with g6+, g7+, etc.

1-0

The new computer era is diminishing the possibility of this kind of idea, but the ‘Topi touch’ has never
completely disappeared.
One would think that, in the years after this game with Anand, computers would have improved so
much that such creativity had become impossible.
In Wijk aan Zee 2008, Topalov again proved this assessment wrong...
Topalov, Veselin (2780)
Kramnik, Vladimir (2799)
Wijk aan Zee 2008

1.d4 d5 2.c4 c6 3.Nf3 Nf6 4.Nc3 e6 5.Bg5 h6 6.Bh4 dxc4 7.e4 g5 8.Bg3 b5 9.Be2 Bb7 10.0-0 Nbd7
11.Ne5 Bg7

Position after: 11...Bg7

In this well-known position in the Moscow Variation, Topalov decided to go for a new idea: a piece
sacrifice! The engine manages to solve Black’s problems here, but the question was whether Kramnik
would be able to find these moves over the board, as it was almost certain that he wouldn’t be
expecting the next move...

26
12.Nxf7!?

A new move at the time. I remember Topalov smiling at the press conference, after showing the game,
saying: “Now I have to see if Ivan (Cheparinov, his second at the time) has more of these ideas!”.

12...Kxf7 13.e5 Nd5 14.Ne4 Ke7!

The black king has to escape to the other side.


14...Kg8 15.Nd6 Qb6 16.Bh5! Rf8 17.Qg4 Nc7 18.Qe4 with the idea Qg6 is pretty dubious for Black.

15.Nd6 Qb6 16.Bg4

Recently Grischuk tried 16.a4!? here and almost beat Ding Liren. But of course, objectively, Black is
also fine in that case.

16...Raf8 17.Qc2

Position after: 17.Qc2

White wants to play Qg6.


All the moves until now have been more or less normal, but now Kramnik had to find an only move not
to end up much worse. Probably Topalov knew about it, but guessed correctly that Kramnik wouldn’t
find it.

17...Qxd4?

A logical move, but a blunder!

27
The right way to play was shown later, at least for the next four moves: 17...Rhg8!! 18.Rad1 [18.Qg6
Nc7„] 18...c5 19.Qg6 Nc7™ 20.Nxb7

Position after: 20.Nxb7

20...cxd4™ [20...Qxb7? loses to 21.d5! exd5 22.Qd6+ Kd8 23.Qxd7#] 21.Qe4 d3? This one’s wrong.
[21...Nd5! 22.Nd6 d3∞ or even 22...Bxe5!?] 22.Rxd3? [After 22.b3! Black would be suffering.]
22...cxd3 23.Qb4+

Position after: 23.Qb4+

28
23...Nc5™ 24.Nxc5 a5™ 25.Qa3 Rd8™„ Black is slightly better! 0-1 (28) Bromberger, S (2510) –
Sandipan, C (2585) Zürich 2009.

18.Qg6 Qxg4 19.Qxg7+ Kd8 20.Nxb7+ Kc8

20...Kc7? 21.Nc5+–

21.a4!

Position after: 21.a4!

21...b4 22.Rac1 c3

22...Rfg8 23.Qf7 Rf8 24.Nd6+ Kc7 25.Qg6 and the white queen escapes.

23.bxc3 b3

Doing everything to keep the c-file closed.


A) 23...Nxc3 24.Qe7+–
B) In case of 23...bxc3 the most convincing would have been 24.Rfd1! Qe2 25.Nd6+ Kc7 26.Nb5+!
cxb5 27.Rxd5 exd5 28.e6++–.

24.c4 Rfg8 25.Nd6+ Kc7 26.Qf7 Rf8

29
Position after: 26...Rf8

27.cxd5?

Topalov style! Giving this move a question mark breaks my heart, but sadly it doesn’t work,
objectively speaking.
A) 27.Qg6? would be a huge blunder: 27...Nf4 28.Bxf4 Rhg8! and the tide is turning.
B) 27.h3! was best, but it was difficult to calculate it until the end: 27...Rxf7™ (If the queen moves
instead, the e6-pawn will be hanging.) 28.hxg4 Nf4 29.Nxf7 Ne2+ 30.Kh2 Nxc1 31.Rxc1 Rb8
32.Rb1 b2 33.Nxh6 Nc5

30
Position after: 33...Nc5

34.f4! gxf4 35.Bxf4 Nxa4 36.g5 Nc3 37.Rxb2 Rxb2 38.g6+–

27...Rxf7 28.Rxc6+ Kb8 29.Nxf7

Position after: 29.Nxf7

29...Re8?

31
Black missed an amazing save here: 29...Qe2!! 30.dxe6 [30.Rcc1 Rc8 31.Rb1 Nc5 32.dxe6 Re8∞;
30.Rc3 b2 31.Rb3+ Kc7 32.Rc3+ Kb8=; 30.Nxh8 Qxf1+ 31.Kxf1 b2 tr.] 30...Qxf1+! 31.Kxf1 b2
32.exd7 b1=Q+ 33.Ke2 Qe4+ 34.Kd2

Position after: 34.Kd2

and White is lucky that Black doesn’t have anything better than a draw here, e.g. 34...Qd5+ 35.Ke3
Qxc6 36.Nxh8 Qd5 37.Nf7 Qc5+=.

30.Nd6 Rh8 31.Rc4! Qe2 32.dxe6 Nb6 33.Rb4 Ka8 34.e7 Nd5 35.Rxb3 Nxe7 36.Rfb1 Nd5 37.h3

32
Position after: 37.h3

Now it’s all over. White dominates the board and is even slightly ahead on material.

37...h5

37...Nf4 doesn’t help, for instance 38.Rb7 Qa2 39.Kh2! Ne6 40.R1b2 Qxa4 41.Nb5!+–.

38.Nf7 Rc8 39.e6 a6 40.Nxg5 h4 41.Bd6!

Position after: 41.Bd6!

Keeping the bishop on this diagonal and pushing the e-pawn are the two priorities!

41...Rg8 42.R3b2 Qd3 43.e7 Nf6 44.Be5 Nd7 45.Ne6!

33
Position after: 45.Ne6!

An elegant finish!

45...Nxe5 46.Nc7+ Ka7 47.Rb7#

A pleasant victory for Topalov, two years after ‘Elistagate’!

1-0

Give Topalov the initiative – and he will hit you hard!


Let’s close this chapter with a fourth attacking masterpiece by him. This game was played just three
days after the Topalov – Anand game we saw earlier in this chapter.
Topalov, Veselin (2778)
Ponomariov, Ruslan (2695)
Sofia 2005

1.d4 Nf6 2.c4 e6 3.Nf3 b6 4.g3 Ba6 5.b3 Bb4+ 6.Bd2 Be7 7.Nc3 0-0

34
Position after: 7...0-0

Deviating from the earlier game, where Anand played 7...c6.

8.Rc1!?

An interesting waiting move, forcing Black to make a decision: to play 8...d5 with a classical position,
or play 8...c6 and allow 9.e4 with a slightly different version of the line mentioned below. Ponomariov
chose the second option.
8.e4 d5 9.cxd5 Bxf1 10.Kxf1 exd5 11.e5 Ne4 is the very main line.

8...c6

8...d5 9.cxd5 exd5 10.Bg2 would be a totally different game. Karjakin and Leko have played that
position several times as Black.

9.e4 d5 10.e5! Ne4 11.Bd3

This is something White could do only because Black played ...c6, otherwise here Black would play
...Bb7 with an excellent position. Now Black can’t keep his knight on e4.

11...Nxc3

Of course 11...Nxd2 is the other option, but it leads to an easier game for White: 12.Qxd2 Nd7 13.0-0
Bb4 14.cxd5! (an excellent decision without a dark-squared bishop) 14...Bxc3 15.Rxc3 Bxd3 16.Qxd3
cxd5 17.Rfc1± 1-0 (41) Van Wely, L (2655) – Carlsen, M (2646) Netherlands 2006.

35
12.Rxc3

Position after: 12.Rxc3

12...c5

Again the most logical. White is trying an atypical plan of keeping his king in the centre, therefore
Black should open the position so this idea doesn’t work smoothly.
12...Bb4?! 13.Rc2 Bxd2+ 14.Rxd2² would be a great exchange for White; with pawns on e6 and d5 vs
pawns on d4 and e5, Black shouldn’t exchange dark-squared bishops, just like in the French defence.

13.dxc5?!

Take on c5 first, or play h4 straight away? That was probably the question Veselin asked himself.
Objectively, I believe the second choice was better, but, by picking the first one, Vesko actually
reached an improved version of it!
13.h4!? indeed leads to unclear positions.

36
Position after: 13.h4!?

Black has two possibilities (...h6 or ...g6), but you will see that in both cases he needs to defend
aggressively:
A) 13...g6!? 14.dxc5 [14.h5 dxc4! proves too slow] 14...Bb7! [14...d4 15.Rc2 bxc5?! would allow
16.Be4, and after 16...Nd7 White doesn’t go for the exchange, but continues 17.Bh6 Re8 18.h5 with a
nice edge: the e4-bishop is a monster and its exchange will take time.] 15.Bh6 [15.h5 d4 16.Rc1 Nd7
and White has problems with his e5-pawn.] 15...Re8 16.h5

Position after: 16.h5

37
and now the engine suggests a crazy draw: 16...d4 17.hxg6 fxg6 18.Qc2 Bxf3 19.Bxg6 Bxh1
20.Bxh7+ Kh8 21.Bg7+ Kxg7 22.Qg6+ Kh8 23.Qh5 Kg7 24.Qg6+=.
B) 13...f5?! 14.exf6 Rxf6 15.Bg5 cxd4 16.Bxf6 Bxf6 17.Rc1 and White is better.
C) 13...cxd4? loses to the prosaic 14.Bxh7+! Kxh7 15.Ng5+ and the white queen joins the attack for a
decisive result.
D) 13...h6 14.Bb1!? [14.dxc5 d4! Before White plays Bb1. 15.Rc1 Bb7!‚] 14...cxd4™

Position after: 14...cxd4™

15.Nxd4 [15.Qc2 doesn’t work: 15...g6 16.h5 dxc3 17.Bxh6 fails: 17...f5! 18.exf6 Rxf6 19.hxg6 dxc4
20.g7 Qd3–+.] 15...dxc4 16.Qg4! [16.Bxh6 f5!∞; 16.Qc2 g6∞] And now Black has to find 16...Qd5™
17.Be4 [17.0-0 Qxe5 18.Re1 h5 19.Qe4 Qxe4 20.Bxe4 Nd7 21.Bxa8 Rxa8©] 17...Qxe5 18.Nf3 Qc7
19.Bxa8 [19.Bxh6 Bf6„] 19...Nd7

38
Position after: 19...Nd7

Amazing! Black is a rook down – and is actually slightly better! 20.Bxh6 [20.Be4? Nf6–+] 20...Bf6©

13...bxc5?!

Black needed to do something more active than just taking the pawn back.
Again he should play 13...d4 before White plays Bb1, in order to force the white rook back.

Position after: 13...d4

39
14.Rc1 Bb7!? Nowadays such pawn sacrifices are well-known because the engine shows them in lots of
positions, but many years ago it was a different story. With the black bishop on b7, the white attack is
too slow. [14...bxc5!? 15.Be4 Nd7 is also interesting.] 15.h4?! [15.Bb1 f5µ; 15.cxb6 axb6 and White is
worse; the a2-pawn is hanging, and Black will play ...Nd7 to target the e5-pawn.] 15...Qc7!

Position after: 15...Qc7!

16.Bxh7+? Kxh7 17.Ng5+ Bxg5 18.hxg5+ Bxh1–+ The nuance!

14.h4!

40
Position after: 14.h4!
Meditating (photo Anastazia Karlovich)

Now Black will never be in time to play ...d4, and White has
a straightforward attack.

14...h6?

A) 14...d4?? 15.Bxh7+! Kxh7 16.Ng5+ Kg8 17.Qh5 and


mate to follow.
B) 14...g6! was necessary: 15.cxd5 [15.h5 d4 16.Rc1
Bb7∞] 15...exd5 [15...Qxd5 16.Bxa6 Nxa6 17.Bg5 Bxg5
18.Qxd5 exd5 19.hxg5²] 16.0-0 d4 17.Bxa6 Nxa6 18.Rc4
Qd5 19.Qe2 and White will play Ne1-d3 with the more
pleasant position, but no drama for Black.

15.Bb1! f5

15...d4 16.Qc2 g6 17.h5! dxc3 18.Bxh6 and Black will get


mated. If you compare the line 13.h4 h6 14.Bb1 cxd4
15.Qc2, Black doesn’t have ...Qd3 to defend himself.

16.exf6 Bxf6 17.Qc2!

Position after: 17.Qc2!

17...d4

41
17...Bxc3 18.Bxc3 d4 19.Qh7+ Kf7 20.Qg6+ Ke7 [20...Kg8 21.Ne5+–] 21.Nxd4! cxd4 22.Qxg7+ Rf7
23.Bb4++–

18.Ng5!!

The show must go on: White prevents the black king from escaping to f7.

18...hxg5 19.hxg5 dxc3

Position after: 19...dxc3

20.Bf4!?

The nicest way to win.


20.Rh8+ Kf7 21.Qg6+ Ke7 22.gxf6+ gxf6 23.Rh7+ Kd6 24.Qd3+ Kc6 25.Qxc3 also wins, according
to the engine.

20...Kf7

20...Bd4 21.Qg6!! with the idea Rh8+ and Qh7# wins: 21...Bxf2+ 22.Ke2+–.

21.Qg6+ Ke7

42
Position after: 21...Ke7

Now it is just a matter of clean calculation.

22.gxf6+ Rxf6 23.Qxg7+ Rf7 24.Bg5+ Kd6 25.Qxf7 Qxg5 26.Rh7! Qe5+

26...Qd2+ 27.Kf1 Qd1+ 28.Kg2 Bb7+ 29.Qxb7+–

27.Kf1 Kc6 28.Qe8+ Kb6 29.Qd8+ Kc6 30.Be4+!

43
Position after: 30.Be4+!

Mate was to follow: 30.Be4+ Qxe4 31.Qc7#.

1-0

With such games to his account, who wouldn’t dream of working with such a player?

44
Chapter 2
The Start of Our Cooperation

Meeting Topalov for the first time


In the middle of 2010 I was talking with the experienced GM and friend of mine Darko Anic, and he
suggested to me several ideas to boost my chess career. One of them was to assist a top player, and he
told me that Veselin Topalov was eager to find new young players to help him. Giving me what he
probably thought to be a random tip, Darko changed my chess career dramatically.
Shortly afterwards I sent Topalov’s manager, Silvio Danailov, a short email to which I got a friendly
answer, asking me to send a sample of my chess work. After checking it, Veselin called me to suggest I
come to Salamanca, where he lives, for a training week. We decided to set it for December of that year.
Our first training session was approaching; I didn’t know what to expect but was really looking forward
to it. I was playing a game somewhere in Belgium on a Sunday and flying to Madrid on the Monday in
order to reach Salamanca. For the Sunday night I had booked a hotel room next to Charleroi airport in
Brussels. Two hours before my flight, I left my room and went to look for the check-in desk. I couldn’t
find my flight! I had confused Charleroi and Zaventem airports, which are on two different sides of the
city! I could not believe that I was going to look such a fool for my first meeting with Topalov. I got
hold of a taxi driver who told me that, even if he had the streets to himself, he couldn’t guarantee that I
would catch my flight. I told him, “If I make it, I’ll pay you double!”. And it worked! There I was, first
on the plane, then in Madrid and finally in Salamanca.
Veselin was waiting for me at the bus station. Like many people, I admired him as a player, not just for
his strength but also for his style. However, also like many people, I had never exchanged a word with
him. He smiled, helped me with my luggage and asked if all had gone smoothly. I answered “Yes!”. He
drove me to his second apartment, a beautiful flat where I stayed for the whole week.
The plan was generally the following: coffee or sport in the morning, chess work in the afternoon, and
in the evening I would recheck or tidy up our daily work. We would regularly go for a walk, or spend
an hour over coffee with his friend and manager, Silvio Danailov. Until then, my only contact with
Silvio had been limited to a short conversation and a few emails. Given all the gossip about him on the
internet, I really didn’t know what to expect of him as a person. I was very pleasantly surprised. I
immediately saw him as someone with strong opinions, but could also spot in him a true love for chess.
Even today, many people tend to ignore – or forget about – that second part. I had spoken to many
‘chess politicians’ during my career, and this was the first time I was hearing about chess! One of his
main preoccupations was clearly to make the game livelier and more combative, and I’m sure this
played a role in Vesko being such a courageous player. Remember, Silvio is the inventor of the famous
‘Sofia rules’, which have ended up being not such a bad idea after all!

45
Silvio Danailov and Veselin Topalov, Dortmund... 1996! (photo Dagobert Kohlmeyer)
Back to Veselin. Our work started and the direction was generally clear: he wanted to sacrifice
something! I tried to count how many times he suggested moves like h4 or g4 in any kind of position,
but gave up! As I mentioned earlier, he didn’t care about risks: what mattered was to find rich new
ideas, and to make life difficult for his opponents. Only a handful of Vesko’s inspirations were good
enough to be played, but all of them made sense. Once I had understood that, we quickly started
working in the same direction.

46
After just one day, en route to a training session, Veselin told me something I was not expecting at all.
He said that if there were any particular openings I wanted to analyze with him for myself, he would be
very happy to do so. Not only that, but he gave me some tips that would improve my own chess, and
even gave me a database of theoretical endgames he had created for himself over several months. “You
should learn it all, then recheck it from time to time, when you’re on the train or plane for example”, he
said. I was amazed. Many people had told me that working with/for a top player was like being a slave
– well, apparently it depends on the player!
After a few days we had got to know each other better and started to talk about several things besides
matters on the chess board. On one occasion when we were having lunch I could not prevent myself
from asking him about his match with Kramnik in Elista in 2006. Of course I knew how much Vesko
had been destroyed by the media at the time.
I was really shocked by how tortured he still was by this episode in his life, four long years later. His
explanation of the whole situation was that the Bulgarian team felt that there were too many strange
things about Kramnik’s behaviour, which were confirmed when watching videos of the games. Not
clear cheating, just things which aroused suspicion. They started to ask questions away from the scene.
When too many contradictory answers were given by the opposing team, when videos disappeared, and

47
zero action was taken by the organisers, he had to make the whole story public not to let it continue.
Even today, Veselin hasn’t changed his opinion that there were too many coincidences, for reasons
which I think are very respectable.
However, so that there will be no misunderstanding, I would like to make it clear that this book is not
meant to accuse anyone, or to give my version of events which I didn’t witness. Actually, the only time
I’ve spoken to Vladimir Kramnik up to now was a friendly chat about football, during the Paris Grand
Chess Tour 2018. We were sharing a car and I even had to cheer him up for Russia losing heavily to
Uruguay! But if it’s about judging whether Topalov just made a fuss because he was losing some
games, well, anyone talking to him for a few minutes will never think this way again. Could things
have been managed in a better way? Possibly, although that’s always easier to say from the outside.
The good news is that although they haven’t shaken hands for many years, it hasn’t stopped either of
them from making beautiful contributions to chess.
Anyway, after discussing stories of Vesko’s past, and sharing our opinions about many other things, it
was clear: we had become friends.
Evolution of openings
Over the years, the level of preparation and working methods at the top level have developed a lot.
‘Inspired’ ideas had much greater effect when computers were not as powerful as they are now.
Nowadays, you really need to combine creativity with many hours of collaboration with your chess
engine, otherwise you might just get outprepared and lose the game!
Within just a few years, the strength of computers has improved dramatically.
Big holes, nearly unthinkable today, could appear in any player’s opening preparation. Perhaps the
most famous example occurred in 2004, when Vladimir Kramnik lost the eighth game of his World
Championship match as White against Peter Leko.

48
White to move

In the above position, he went for a complex move sequence prepared by his team from computer
analysis: 21.a4 Qg6 22.axb5 Bd3 23.Qf2? Re2 24.Qxe2 Bxe2 25.bxa6. Now they expected his
opponent to sacrifice a piece to stop his a6-pawn from queening, after which White would have more
than enough material for the queen. Instead, over the board, Peter Leko found 25...Qd3!!,

which turned out to be a forced win for Black: 26.Kf2 (26.a7 Qe3+ 27.Kg2 Bxf3+! 28.Nxf3 Qe2+
29.Kg1 Ng4 30.a8=Q+ Kg7 also win for Black) 26...Bxf3! 27.Nxf3 Ne4+ 28.Ke1 Nxc3!–+.
Soon after, it was already a different story. See, a few years later, what deep computer preparation
could be like. This was the first game of the Topalov – Anand match in Sofia in 2010. It could have
been decided in White’s favour without having to play a single move besides his opening preparation if
Veselin had remembered the (barbaric!) totality of his home files. It was played a few months before
we started working together.
Topalov, Veselin (2805)
Anand, Viswanathan (2787)
Sofia 2010

1.d4 Nf6 2.c4 g6 3.Nc3 d5 4.cxd5 Nxd5 5.e4 Nxc3 6.bxc3 Bg7 7.Bc4 c5 8.Ne2 Nc6 9.Be3 0-0 10.0-
0 Na5 11.Bd3 b6 12.Qd2 e5 13.Bh6 cxd4 14.Bxg7 Kxg7 15.cxd4 exd4 16.Rac1 Qd6 17.f4 f6

49
Position after: 17...f6

18.f5

All this is just... theory! Actually one of the main lines in the Grünfeld.

18...Qe5?! 19.Nf4 g5 20.Nh5+ Kg8?

Here it seems that 20...Kh8! is necessary.

21.h4 h6 22.hxg5 hxg5

And here let’s take a bet: how many years would be needed for anyone to find the winning move? Give
it a try...

50
Position after: 22...hxg5

23.Rf3?

23.Bb5!! At that time very few people on earth (only some top players, to my knowledge) knew about
this move – as it took a lot of effort even for the strongest engines to find it. While writing this book, I
decided to give it a new try – and my computer found it in a second! How humiliating for us poor
humans.

Position after: 23.Bb5!!

51
Some sample lines:
A) 23...a6 24.Bc6 Bb7 25.Bd7 and the bishop gets to e6, while if Black plays 25...Bc8 White answers
26.Be6+ anyway: 26...Bxe6 27.fxe6 Qxe6 28.Qxd4+–.
B) 23...Rf7 24.Nxf6+! Qxf6 [24...Rxf6 25.Qxg5+ Kf7 26.Qh5+ Kg8 27.Rf3+–] 25.e5! Qxe5
26.Qxg5+ Qg7 27.Qd8+ The nuance with the rook on f7 and not f8! 27...Rf8 28.Qd5+ Qf7
29.Qxa8+–.
C) 23...Kh7 24.Rf3 …Rh3+–
D) 23...Kf7 24.Bc6 Bb7 25.Bxb7 Nxb7 26.Rc6+–
E) 23...Qxb5 24.Nxf6+! [24.Rc7 Qe5 25.Rg7+ Kh8 26.Rg6 also wins.] 24...Rxf6 25.Qxg5+ Kf7
26.Rc7+ [26.Qh5+!? Ke7 27.e5 also wins.] 26...Bd7 27.a4 Qxa4 [27...Qd3 28.e5+–] 28.e5+–

23...Kf7??

23...Bd7! was fine for Black.

24.Nxf6!

Position after: 24.Nxf6!

Now White is winning again.

24...Kxf6

24...Qxf6 25.Rh3 is overwhelming: both rooks are coming in. [Funnily enough, the engine again gives
25.Bb5 as winning!]

52
25.Rh3 Rg8 26.Rh6+ Kf7 27.Rh7+ Ke8 28.Rcc7 Kd8 29.Bb5!

Position after: 29.Bb5!

Finally this move! Black is paralyzed – and loses lots of material!

29...Qxe4

29...a6 30.Rce7 Qxe7 31.Qxd4++–

30.Rxc8+! 1-0

Black resigned in view of 30.Rxc8+ Kxc8 [30...Rxc8 31.Rd7+ Ke8 32.Rxd4++–] 31.Qc1+ Nc6
32.Bxc6+–.

53
Topalov – Anand (photo Anastazia Karlovich) World Championship Match (Sofia 2010) – Game 1
This game says two things, namely that top players need prodigious powers of memory, and that
nowadays a second cannot afford to just think by himself – the engine is going to laugh at him.
Combining human ideas and engine strength is an interesting challenge, and that’s what a second is
hired for, not just to press the computer button.
The remaining part of the job is to present the final work to the player in clean and well-organized
fashion, so he can remember the maximum number of things in the minimum time, without missing any
important information. In particular, if analysis has been prepared only the night before the game, you
can’t throw ‘your’ player masses of new information in the morning: not only might he not be able to
assimilate it, but it would take up a lot of his energy.
In other words, a second must be able to deliver a concise summary of his discoveries, so that the
player has all the important ideas and key moves in mind, without having to remember too many long,
sometimes wild, lines. How many times per tournament does one hear, “I forgot my preparation”?
First experiences!
In March 2011, Veselin was playing in the ‘Amber’ tournament in Monaco, the last edition of a
prestitigous event consisting of rapid, blitz and blindfold games.
He asked me if I could come to Salamanca just before the event, so we could have some training games

54
and look at some more openings. Of course I went with great pleasure.
Vesko really wanted to try his best, and there he used two of our best novelties, one of them in a rapid
game against Nakamura, which he won convincingly, and the other in a blindfold game against
Aronian, which he lost despite holding a winning advantage.
These two ideas were quite representative of Veselin’s character over the board. Let’s have a look.
Topalov, Veselin (2775)
Nakamura, Hikaru (2774)
Monaco (rapid) 2011

1.e4 e6 2.d4 d5 3.Nc3 Nf6 4.e5 Nfd7 5.f4 c5 6.Nf3 Nc6 7.Be3 cxd4 8.Nxd4 Qb6 9.Qd2 Qxb2
10.Rb1 Qa3 11.Bb5 Nxd4 12.Bxd4 a6 13.Rb3 Qe7 14.Ba4!N

Position after: 14.Ba4!N

This genius idea was a typical ‘Topalov try’ when we were having our training sessions! Since then, it
was been played seven times, mostly by very good players. Basically, the idea is not to release the
pressure (as with the usual 14.Bxd7+) and to answer a possible ...b5 with a sac on b5.

14...b5?

We considered this a huge mistake, probably rightly so.


14...Qd8! was the main move in our notes. Note that Black may first give a check on h4, but it will just
transpose. One of our lines was played twice in real top-level games: 15.Bb6!? Qh4+ 16.g3 Qg4

55
Position after: 16...Qg4

17.Ne4! This move (which we had in our analysis) was found by Grischuk. [Funnily enough, he later
reached the same position with colours reversed against... Veselin! Sadly, Vesko confused two lines
and after 17.0-0 Be7 18.f5 0-0! 19.Rf4 Bg5! was just much worse. 0-1 (68) Topalov, V (2749) –
Grischuk, A (2761) Paris 2017] 17...Be7! [17...dxe4? loses to 18.Rc3! Bb4™ 19.Rxc8+ Rxc8 20.Bxd7+
Ke7 21.Qxb4+ Kxd7 22.Qd6+ Ke8 23.0-0+–] 18.Nd6+!N [Now it was Grischuk who went wrong as
White! 18.Nf2? Qg6 19.Rc3 0-0µ 1-0 (52) Grischuk, A (2763) – Morozevich, A (2770) Astana 2012]
18...Bxd6 19.exd6 0-0 20.0-0 It’s not like White is winning, but he has an easier position: the black
pieces are quite paralyzed.

15.Bxb5 axb5 16.Nxb5 Qd8

It is debatable whether Black should have played 16...Qh4+ first, in order to provoke g3 or Bf2.

17.Qc3!

56
Position after: 17.Qc3!

The following sequence is forced.

17...Qa5™ 18.Nc7+ Kd8 19.Nxa8 Qxa8 20.0-0 Qa4

With this move, our preparation ended. Our conclusion was that the position was clearly better for
White, whatever Black plays.
We had briefly checked 20...Rg8 (with the idea ...g5), but after 21.Rfb1!

57
Position after: 21.Rfb1!

White is doing great, as 21...g5?? loses to 22.Rb8! Nxb8 23.Bb6++–.

21.Kh1!

The best move. At some point the d4-bishop will disappear and the king on h1 will be far from all kinds
of tactics. Now Black needs to react.

21...Be7 22.Rfb1 f6

Position after: 22...f6

23.exf6?

Missing the cute winning move 23.Rb6!, targeting the e6-pawn. For example: 23...Qc4 [23...Nxb6?
24.Bxb6+ Kd7 25.Qc7++–] 24.Rc6! Qxc3 25.Bxc3+– (…Ba5+).

23...gxf6?

23...Bxf6! and the game would have become very decent for Black.

24.Rb6!

This time he saw it!

24...Qxa2 25.Re1

58
25.R1b3!+– preventing ...Qc4 was probably even stronger.

25...Qc4 26.Rbxe6

Position after: 26.Rbxe6

26...Bb4?

26...Qxc3™ 27.Bxc3 Ne5™ was the only way to stay in the game: 28.fxe5 Bxe6 29.exf6 Kd7 30.fxe7
with nevertheless a relatively easy win for White.

27.Bxf6+ Nxf6 28.Qxf6+ Kc7 29.Re7+?

29.Qe5+! would have put an end to the game:

59
Position after: 29.Qe5+!

29...Kb7 30.Qg7+! and Black loses the whole house.

29...Bd7

White is still better, but the position has become a mess. Fortunately, Vesko played the rest of the game
perfectly.

30.Qe5+ Kc6 31.Qxh8 Bxe7 32.Qa8+ Kc7 33.Qa7+ Kd8 34.Qb8+ Bc8 35.Qe5

60
Position after: 35.Qe5

35...Bc5?!

35...Qc5 was a tougher defence, the idea being to meet 36.f5? with 36...Bd6!∞.

36.f5!

The f-pawn will make the difference.

36...Qxc2?

The final mistake.


The very difficult 36...Kd7! with the idea ...Bd6 was the only way to keep on fighting.

37.f6 Qf5 38.Qe8+ Kc7 39.Rc1 Kb6 40.f7 d4 41.Qd8+ Kb5 42.Qc7

Position after: 42.Qc7

Black can’t hold everything: 43.Qxc5+ and 43.Rxc5+ are deadly threats, while if the c5-bishop moves,
Black will lose the one on c8 (if he doesn’t get mated right away!).

1-0

Topalov, Veselin (2775)


Aronian, Levon (2808)
Monaco (blindfold) 2011

61
1.d4 Nf6 2.c4 e6 3.Nf3 d5 4.Nc3 Bb4 5.cxd5 exd5 6.Bg5 h6 7.Bh4 c5 8.e3 g5 9.Bg3 c4 10.Be2 Ne4
11.Rc1 Qa5 12.Ne5

Position after: 12.Ne5

By now, this move has been tried by many GMs, but at the time of the game it was a new idea,
involving quite a few sacrifices! Black has several possibilities already, and I will give some samples of
the lines we had on each of them.

12...Nc6

A) 12...0-0!? 13.Bf3! [13.0-0?! Bxc3 14.bxc3 f6! 15.Ng6 Rf7 proved fine for Black in Williams, S
(2465) – Sanikidze, T (2514) Saint – Quentin 2014, 0-1 (28).] 13...Nxg3! Other moves are worse.
[13...Nxc3?! 14.bxc3 Bxc3+ 15.Kf1 Nc6 16.h4‚; 13...f6? 14.Bxe4! dxe4 15.Qh5+– ½-½ (46) Guo,
Q (2351) – Tan, Z (2438) China 2012; 13...Bxc3+?! 14.bxc3 Nxc3? 15.Qd2+–] 14.hxg3 [We even
considered the very aggressive 14.fxg3!? Nc6 15.h4 to be quite unclear.] 14...Kg7 15.Bh5

62
Position after: 15.Bh5

Our main move. Seven years later, it also appeared in practice, although White became overoptimistic
and soon lost! The most interesting options for both sides lead to crazy lines:
A1) 15...Nc6 16.Nxf7!N [16.f4?! was played in the only existing game: 16...f6! 17.Be8? Nxe5
18.fxe5 Rxe8 19.exf6+ Kxf6 20.Qf3+ Ke7 21.0-0 Be6–+ 0-1 (31) Vocaturo, D (2620) – Sanikidze,
T (2501) Batumi 2018] 16...Be6 [16...Rxf7? 17.Bxf7 Kxf7 18.Rxh6+–] 17.Nxh6!? [17.Ne5 Nxe5
18.dxe5 Qxa2 19.Qd2 Qa6 20.0-0 Rad8 21.Ra1 Qb6∞] 17...Qxa2™ Black needs to collect some
pawns as well! [17...Kxh6?? 18.Bf7++–] 18.Rc2!

63
Position after: 18.Rc2!

18...Rh8™ [18...Qb3? 19.Bg4±] 19.f4! [19.Ng4 Rad8©] 19...Rxh6 [19...gxf4? 20.Nf7!! fxe3 21.0-
0+–] 20.fxg5 followed by 0-0 and Rf6, with a crushing attack.
A2) 15...Bf5!? 16.Bxf7!? [16.Nxf7 Nd7 17.Nxh6!? Kxh6 18.g4 Bd3 19.Be8+ Kg7 20.Bxd7 Rh8©]
16...Nc6

Position after: 16...Nc6

17.Bg6! [17.Qh5 Nxe5 18.dxe5 Bxc3+ 19.bxc3 Qa3™ and it is White who should be careful.]
17...Nxe5 18.Bxf5 Rxf5 19.dxe5 Rxe5 20.0-0 and White’s position is easier.
B) Two years later 12...Nxc3 was played in two GM-level games, but on both occasions Black failed
to solve his problems: 13.bxc3 Bxc3+ 14.Kf1 Nc6 15.h4!

64
Position after: 15.h4!

15...g4 [15...Bb2 16.Rb1 c3 17.Nxc6 bxc6 18.hxg5 Bf5 19.Bd3 Bxd3+ 20.Qxd3 Qxa2 21.Ke2 Qc4
22.Be5± 1-0 (101) Ragger, M (2655) – Sanikidze, T (2580) Germany 2013] 16.Bxg4 Nxe5 17.Bxe5
Rg8 18.Bxc8 Rxc8 19.Qc2 Bb4 20.Rh3± ½-½ (56) Gagunashvili, M (2601) – Mchedlishvili, M
(2637) Tbilisi 2013.

13.0-0

Position after: 13.0-0

65
13.Kf1 was tried later, but not so successfully: 13...Nxg3+ 14.hxg3 Nxe5 15.dxe5 Be6 16.a3 Bxc3
17.Rxc3 Qc5 18.Bg4 0-0-0= ½-½ (39) Kacheishvili, G (2597) – Kulaots, K (2564) Las Vegas 2014.

13...Bxc3

13...Nxg3 is also critical.

Position after: 13...Nxg3

A) We had concluded that 14.fxg3 didn’t offer White anything, and the reason was shown at top
women’s level one year later: 14...Nxe5 15.dxe5 Be6 16.Bg4 0-0-0! 17.Rxf7 Qb6! 18.Bxe6+ Qxe6∞
0-1 (42) Danielian, E (2476) – Kosintseva, T (2530) Turkey 2012.
B) 14.hxg3 Nxe5! [We considered 14...Be6 15.Nxc6 bxc6 16.Bg4 as pretty annoying for Black.]
15.dxe5 Be6

66
Position after: 15...Be6

16.a3 [The reason why we didn’t like 16.Bg4 was shown the following year: 16...h5! Black plans to
castle long, but wants to avoid Bxe6 followed by Qg4. Of course, taking the h-pawn and opening the
h-file would be very dangerous for White. 17.Bxe6 fxe6 18.Ne2 0-0-0 19.Nd4 Qa6!? 20.b3 Kb8!
21.Qc2 Ba3 22.Rb1 Bc5∞ 0-1 (63) Ivanisevic, I (2645) – Grandelius, N (2562) Turkey 2012.]
16...Bxc3 17.Rxc3 [We also considered 17.bxc3, but it became clear to us that White should keep the
possibility of the b2-b3 break.] 17...0-0-0 [17...h5?! is too slow: 18.f4 g4 19.f5 Bd7 20.Qd4 0-0-0
21.b4 Qc7 22.Qxd5± 1-0 (33) Vitiugov, N (2694) – Kulaots, K (2587) Gibraltar 2013] 18.Qd4!? This
is what we had considered the best try for White, preventing ...Qc5.

67
Position after: 18.Qd4!?

Additionally, he wants to play g3-g4, f2-f4, or possibly b2-b3 next, depending on Black’s next move.
Since then, it has been played only once, by WGM Batsiashvili in 2014. 18...Kb8! The best move,
which Veselin and I had underestimated. We thought Black should react immediately on the kingside,
but new engines disagree! [18...h5 19.f4! is a mess, but definitely easier for White.] 19.b3 [19.f4 gxf4
20.gxf4 Rhg8!? 21.f5 Bc8∞; 19.g4 Qb6!=] 19...h5∞ ½-½ (47) Batsiashvili, N (2417) – Danielian, E
(2458) Plovdiv 2014.

14.bxc3

Position after: 14.bxc3

14...Nxc3

Another option is to ignore the pawn gift: 14...0-0 15.Bf3! Nxg3 [15...Nxc3?? 16.Nxc6 bxc6
17.Qd2+–] 16.fxg3! Now that White has castled, he opens the f-file, not the h-one! Of course, taking
with the f-pawn is against all positional rules, but here White plays strictly for an attack against the
black king. For example, if castling long was still possible for Black, fxg3 could hardly be a good move
(look at the 13...Nxg3 14.fxg3 line above, for instance!). 16...Nxe5 17.dxe5 Be6 18.Bh5 Qxa2 19.Rf6
Qb2!

68
Position after: 19...Qb2!

Veselin and I considered this move a very important resource for Black, aiming for ...Qb6 and
temporarily paralyzing the white queen, which has to defend the c1-rook. Shortly after, that position
arose in two top-level women’s games: Koneru – Hou and Gunina – Kosintseva.
A) 20.Rxh6 was preferred by Koneru, but the game proved our assessment that it is not the best try:
20...Bf5 21.Rf6 Be4 22.Bf3 Bd3 Black is more than fine, and eventually won the game: 0-1 (37)
Koneru, H (2600) – Hou, Y (2578) Albania 2011.
B) 20.h4! was our choice. 20...Qb6 [20...Kg7 21.Kh2 with the idea Rc2 is also terrible for Black.]
21.Qf3 Qc7 22.Rf1!+–

69
Position after: 22.Rf1!

22...Qxe5 [22...Rad8 23.hxg5 hxg5 24.Bg6!! with the idea Qh5 could have been a delightful finish.]
23.Bxf7+ Bxf7 24.Rxf7 Rfe8 25.Qh5 Re6 26.R1f5 Qxe3+ 27.Kh2+– 1-0 (35) Gunina, V (2507) –
Kosintseva, N (2524) Moscow 2012.

15.Rxc3!

Position after: 15.Rxc3!

70
Of course! White is close to winning.

15...Qxc3 16.Bh5 0-0

A) 16...Rf8 17.Qf3! [17.Nxf7? Rxf7 18.Qf3 Nd8 19.Bc7 Qd3! 20.Bxd8 Qf5 holds for Black.]
17...Nxe5 [17...Be6 18.Qf6+– …Qxe6] 18.Bxe5 Be6 19.Qf6 Kd7 20.e4 Qb4™ 21.exd5 Bxd5
22.Qf5+! Be6 [22...Kc6 23.Bf3! crushes.] 23.Qc2! followed by Rb1, with a winning attack.
B) 16...Nxe5 17.Bxe5 would not help: White has Qf3 next anyway, and now even Qb1 becomes a
strong option; he has an extra threat in Qb5+.
C) 16...Be6 17.Qf3 0-0-0 18.Bxf7+–

17.Qf3!

Position after: 17.Qf3!

17...Nd8

17...Kg7 18.Qxd5 Be6 19.Qe4±

18.Qf6 Qc2 19.Ng4

A) 19.Qxh6?! Qh7 20.Qxg5+ Qg7 is less clear.


B) 19.f4!? was killing as well.

19...Bxg4 20.Be5! Qh7 21.Bxg4 Ne6

71
Position after: 21...Ne6

22.f4?

Suddenly the position is not so clear.


22.Bf5! was correct: 22...Qg7 23.Qe7! Rfe8 24.Qxb7 Qf8 25.Qxd5+–.

22...Qg7 23.Qe7 Rae8 24.Qd7?!

24.Qxb7 would have led to a forced, but very complicated, draw: 24...f6 25.Qxd5 fxe5 26.Bxe6+ Kh8

72
Position after: 26...Kh8

27.f5!? [27.fxe5 Rxf1+ 28.Kxf1 Rf8+ 29.Ke2 Qg6 30.Qxc4 Qb1=] 27...exd4 28.exd4 Rd8 29.f6!
Qc7™ 30.Qe4 Qb6™ 31.Qg6 Qxd4+ 32.Rf2 [32.Kh1?? Rxf6–+] 32...Qa1+ 33.Rf1 Qd4+=.

24...Qh7 25.fxg5 f5!

Position after: 25...f5!

A great move, forcing a queen exchange. Now the risk is gone and it is White who has to be very
careful.

26.Qxh7+ Kxh7 27.Bxf5+ Rxf5 28.Rxf5 c3!?

A good practical try.


The simple 28...Nxg5 was perhaps objectively stronger. White has problems: 29.Kf1 [29.Bf4 c3
30.Rxd5 Rc8³] 29...Re6!³ …...Kg6.

29.gxh6 b5 30.Bd6 a5 31.Ba3 b4 32.Bc1 a4

73
Position after: 32...a4

33.Rf7+!

33.Rxd5 Nf4! and White is in trouble. [33...b3? 34.axb3 axb3 35.Rb5 b2 36.Bxb2 cxb2 37.Rxb2±]

33...Kxh6 34.Rb7 Nf4!

Again! Otherwise White would be better. However, it is not enough for Black to be better.

35.exf4™

35.Kf1? Nd3–+

35...Re1+ 36.Kf2 Rxc1 37.Rxb4 Ra1 38.Rb6+ Kg7 39.Rc6 Rxa2+ 40.Ke3!

40.Kf3 would not be smart, as after 40...Rb2! the c3-pawn is untouchable.

40...Rxg2 41.Rxc3 Rxh2 42.Rc6 a3

74
Position after: 42...a3

43.f5?!

Allowing the only trick. White should just have played Ra6 first and the players could have shaken
hands.

43...Rh6! 44.Rc1??

44.Rc7+! was still a draw: 44...Kf6 45.Rc5!= [or even 45.Ra7 Kxf5 46.Rf7+ Kg5 47.Rg7+ and then
Ra7].

44...Ra6–+

Now it’s too easy for Black.

45.Kd3 a2 46.Ra1 Kf6 47.Kc3 Kxf5 48.Kb2 Ke4 49.Re1+ Kxd4 50.Ka1 Kc3 51.Rc1+ Kd2 52.Rc2+
Kd3 53.Rc3+ Ke4 54.Rh3 d4 55.Rh4+ Kd3 56.Rh3+ Kc4 57.Rc3+ Kd5 58.Rh3 Rb6 59.Kxa2 Kc4
60.Rh1 Kc3 0-1

This loss was a pity, but nevertheless the positions resulting from our training sessions looked
promising for the future!

75
With Veselin in Wijk aan Zee, about a year after we first met (photo Jean-Michel Péchiné)

76
Chapter 3
Learning the Job

In early 2012 Veselin asked me to join him at a tournament for the first time: Wijk aan Zee! Helping a
player during a tournament is a very different – and much tougher – job than finding ideas during a
relaxed training session, for the following reasons:
1) Due to the limited time between games, a second generally needs to work at night, quite often all
night.
2) There is the pressure of quality: a mistake can cost ‘your’ player the game. It is easier to take your
responsibilities when working for yourself; watching the game live and seeing your idea being
completely refuted is, I think, the nightmare of all seconds!
3) As I explained in the previous chapter, the idea behind having a second is that the player does not
have to expend a huge amount of energy before playing a game. That means your conclusions must be
clear and concise, but without accidentally omitting a key move from your summary.
4) Finally, what usually results in the high productivity of a training session is that both players push
each other in the right direction. To find good novelties, one needs to try risky moves, generally
disliked by the engine, and prove them playable after great effort. When you work alone, it is very
common to give up too early; when another player is sitting in front of you he will often tell you, “No,
let’s try longer!”. When analyzing a variation alone in your room, you must do without that extra push.
The routine with Vesko was that he would give me instructions after dinner and I would give him my
conclusions at breakfast. If everything was fine, I could go to sleep! If anything was missing, I had to
do a little more work until the game, which was at 1.00 pm every day.
Sadly, the tournament didn’t go well, as Vesko scored ‘-3’ (meaning three half points below 50%: 5 out
of 13). However, our collaboration and work developed in a very positive direction, and I learnt an
amazing number of things about the job, as you will see throughout the chapter.
Veselin started the tournament with two draws, against David Navara and Loek van Wely. A slightly
disappointing start, but nothing dramatic obviously. In the third round, he was facing Boris Gelfand
with the black pieces. I was amazed how the preparation started: “Gelfand plays this line, and I know
how to make a forced draw, but please check this idea as I want to have a game!”, Veselin told me. I
was speechless, as normally everyone should be happy to make a forced draw against Boris with Black,
especially in such a long event (13 rounds in 16 days!). But Topalov just wants to play chess...
Gelfand, Boris (2739)
Topalov, Veselin (2770)
Wijk aan Zee 2012

1.d4 Nf6 2.c4 e6 3.Nf3 d5 4.Nc3 c6 5.e3 Nbd7 6.Qc2 Bd6 7.Bd3 0-0 8.0-0 dxc4 9.Bxc4 b5 10.Be2
Bb7 11.Rd1 Qc7 12.Bd2

77
Position after: 12.Bd2

12...a5

12...b4 13.Na4 c5 14.dxc5 a5! was what Veselin considered to be a forced draw; later it was shown in
practice that this assessment is probably correct.

13.Rac1

The main move. We considered 13.e4 e5 as tricky, but fine for Black. There were some fun lines like
14.dxe5 Nxe5 15.g3 [15.Nd4 Neg4! 16.h3?! Bh2+ 17.Kf1 Nxf2! 18.Kxf2 b4 19.Na4 c5–+] 15...Neg4
16.h3 Nxf2! 17.Kxf2 Bxg3+ [17...b4!?] 18.Kg2 b4 19.Na4 c5 with huge compensation.

13...Rac8

78
Position after: 13...Rac8

This is the idea (starting with 12...a5) which Veselin wanted to play.
We had considered 14.Qb1 as critical here, and missed the easiest move of the position... played a
tempo by Boris.

14.Ne4!

14.Qb1 Qb8 and here we checked several options for White. One that wasn’t too relevant was 15.Ne4
after which Black has easy play: 15...Nxe4 16.Qxe4 c5 17.Qh4 and now just 17...cxd4 and Black is
fine. But pause for a minute and think: if White wants to enter this position (playing Nc3-e4), shouldn’t
he do it with the black queen on c7 and not on b8, to take advantage of the rook opposing the queen on
the c-file? A crazy miss by me!
We also considered 14.e4 (to which Black answers 14...e5) and 14.a3 (to which Black answers 14...b4),
and both were fine. But these were pointless!

14...Nxe4 15.Qxe4 c5 16.Qh4

79
Position after: 16.Qh4

Following the game live was no fun: it was clear Black’s position was extremely difficult to play and I
wasn’t feeling too smart.
Fortunately, Veselin played perfectly and was able to hold this slightly worse position.

16...b4

A) 16...Qb6 doesn’t solve all the problems: 17.dxc5 Nxc5 18.Bxa5! [18.Bc3!? is also very interesting,
forcing 18...f6] 18...Qxa5 19.Rxd6 Qxa2 20.Rd2. The position looks very dry, but is really difficult to
play for Black.

80
Position after: 20.Rd2

20...b4!? [20...Qa8 21.Rdd1²; 20...Nb3? 21.Rxc8 Bxc8 22.Rd8±] 21.h3! Threatening Rd2-c2.
[21.Qxb4? Na6–+; 21.Rdc2 b3 22.Rxc5 Rxc5 23.Rxc5 Qxb2 24.Bf1 Qa1 25.Nd2 b2=] 21...b3
[21...h6 22.Ne5‚] 22.Ne5 Bd5 23.Bh5!

Position after: 23.Bh5!

And Black may have serious problems soon, for example: 23...g6 [23...Qa8 24.Qe7+–; 23...Qa7
24.Bxf7+! Rxf7 25.Nxf7 Qxf7 26.e4! Bxe4 27.Rd8++–] 24.Bf3 Bxf3 25.gxf3! with the idea Ng4.

81
25...Qa8 26.Kg2 with a clear advantage for White. Note that provoking ...g7-g6 was important so that
26...f6 loses to 27.Ng4+–.
B) 16...c4 17.e4 is no fun neither. Here 17...Rfe8 nearly works, but fails to 18.a4 Be7 19.Bg5 Bxg5
20.Qxg5 Bxe4 21.Nd2 Bd5 22.axb5 Nb6 23.Nxc4! Bxc4 24.Bxc4 Nxc4?! 25.b3±.

17.e4

17.Bd3 h6 18.e4 scared me, as White has an extra option of sacrificing the bishop on h6, but it seems
Black holds: 18...Rfd8! [18...Qd8? 19.Bg5!±] 19.Be3 [19.e5 Bf8 20.Qg3 Qb8!? 21.Bxh6 Bxf3 22.gxf3
cxd4∞] 19...Bf8 and White is no more than symbolically better.

17...Rfe8™

Position after: 17...Rfe8™

18.Bb5

18.e5 Be7 19.Bg5 Bxg5 20.Nxg5 h6 21.Ne4 Bxe4 22.Qxe4 Qb8 and Black should hold.

18...Be7!

18...Bc6 19.Bd3 Be7 20.Qh3 is very dangerous for Black: there is an extra threat d4-d5.

19.Qf4

19.Bg5 Bxg5 20.Nxg5 h6=

82
19...Qxf4 20.Bxf4 Red8 21.d5

21.dxc5 Nxc5 22.Rxd8+ Rxd8 23.Bc7 Ra8=

21...exd5 22.exd5 Bf6

Position after: 22...Bf6

Black has managed to solve his problems. The endgame is equal.

23.b3 h6

23...Nb6!? was also fine.

24.d6 Bc6 25.Bxc6 Rxc6 26.Nd2 Be5 27.Bxe5 Nxe5 28.Ne4 f5 29.f4 Nd3 30.Nf6+ Kf7 31.Rxd3
Kxf6 32.d7 Rc7 33.Rcd1 Ke7 34.Re1+ Kf7 35.Red1 Ke7 ½-½

It was funny that when the players analyzed afterwards, Veselin showed Boris how he could have made
an early draw! The mistake I made in preparation was huge, missing a simple idea I would have seen if
I had looked at the position afresh with my own eyes. I had started my work based on a wrong
judgement which I had never bothered to question. I felt fortunate that it didn’t have serious
consequences, and made sure never to make it again.
In game four Veselin had to face one of the biggest Azeri talents ever, the sadly missed Vugar
Gashimov. We found a couple of new ideas in a well-known line of the Slav Defence. We surprised
Vugar, and Veselin was able to get an ending with an extra pawn, albeit very difficult to convert. This
game illustrates perfectly how the level of preparation has improved between 2012 and now.
Topalov, Veselin (2770)

83
Gashimov, Vugar (2761)
Wijk aan Zee 2012

1.d4 d5 2.c4 c6 3.Nf3 Nf6 4.Nc3 dxc4 5.a4 e6 6.e3 c5 7.Bxc4 cxd4 8.exd4 Nc6 9.0-0 Be7 10.Qe2!

Position after: 10.Qe2!

Nowadays everyone understands this is the best plan, but at the time of the game, though, we were just
happy to discover new ideas.

10...0-0 11.Rd1 Nb4

The main move. Another option is 11...Nd5, but it’s not generally the choice of good players.

12.Bg5 Bd7?!

This move leads by force to a drawish pawn-down ending, not the sort of thing anyone would be happy
to play as Black.
The analysis of the critical move 12...h6 is a perfect example to show the impact of the computer
nowadays. Let’s see how theory has developed lately: 13.Bxf6 Bxf6 14.Ne4 b6 15.Ne5

84
Position after: 15.Ne5

A) 15...Bb7 is one of the human moves, but is tactically refuted: 16.Nxf6+ Qxf6 17.Nd7 Qg6 18.d5!²
0-1 (45) Edouard, R (2686) – Ernst, S (2556) Wijk aan Zee 2013.
B) 15...Be7 16.Ra3! Bb7 17.Qg4 Nd5 18.Rg3 Bg5 would be the second best defence, and after
19.Nxg5 Qxg5 20.Qe2 for example, Black has to be very careful.
C) Now see what kind of equalizer has been found by recent engines... 15...Bh4!

Position after: 15...Bh4

85
16.Ra3 [16.g3!? Be7 17.Nc3!? was definitely an interesting try by Nakamura. Note that the next five
moves are forced: 17...Bf6 18.d5 Qc7! 19.d6 Qc5 20.Ng4 Bxc3 21.bxc3 Nc6 22.Rab1 a6 23.d7 Bb7
24.Bd3 Rfd8 25.Qe4 Kf8∞ ½-½ (64) Nakamura, H (2790) – Svidler, P (2757) Moscow 2016.]
16...Bb7 17.Rh3 Nd5

Position after: 17...Nd5

18.g3 [18.Qg4 Bg5 would be a worse version for White than 15...Be7, since there we reached the
same position with the white rook on g3 and not h3. And that is the whole point!] 18...Be7 19.Ng4
Nf6! and the position is a forced draw. White needs to force events or he will end up slightly worse.
20.Nexf6+ Bxf6 21.d5 Bxd5 22.Bxd5 exd5

86
Position after: 22...exd5

23.Rxh6 [White could also play something like 23.Rh5 but after 23...Re8 or 23...Qe7 there is no way
he can be better.] 23...gxh6 24.Nxh6+ Kg7 25.Nf5+ Kg8 26.Qg4+ Kh7 27.Qh5+ Kg8 28.Nh6+ Kg7
29.Nf5+ Kg8 30.Rxd5 Qxd5 31.Qg4+ Kh7 32.Qh5+ ½-½ (34) Bluebaum, M (2642) – Svane, R
(2583) Dresden 2017. All this is forced, and was played à tempo by Bluebaum. Imagine that six years
ago, it was still possible to (nearly) surprise 2750+ players after 12.Bg5.

13.d5!

87
Position after: 13.d5!

Before Black plays ...Bc6 and equalizes.

13...exd5 14.Nxd5 Nbxd5 15.Bxd5 Nxd5 16.Rxd5 Bxg5 17.Nxg5 h6 18.Qd2 hxg5 19.Rxd7 Qf6
20.Rxb7

Position after: 20.Rxb7

Definitely an opening success for White, who has won a pawn. However converting it into a win is not
an easy job.

20...Rab8 21.Rxb8

21.Qd7 Rxb7 22.Qxb7 Qd4 is no improvement for White.

21...Rxb8 22.Rb1 Qe6 23.Qxg5

23.b4?! Qe4 24.Rb3 a5=

23...Qa2 24.Rd1 Qxa4 25.Rd8+ Rxd8 26.Qxd8+ Kh7 27.Qd3+ g6 28.b3 Qa1+ 29.Qf1

88
Position after: 29.Qf1

29...Qd4

29...Qb2 30.Qd1±

30.g3 Qd5 31.Qb1 a5 32.h4 Kg8 33.Qc2 Kh7 34.Kh2 Qf3 35.Kg1 Qd5 36.Kf1 Kg8 37.Ke2 Qe5+
38.Kf3 Qd5+ 39.Ke3 Qe5+ 40.Qe4 Qc3+ 41.Qd3 Qe5+ 42.Kf3 Qf6+ 43.Kg2

Position after: 43.Kg2

89
Black has defended precisely and given White no chance to bring his king to the queenside. And now,
he has a typical drawing blow...

43...a4! 44.bxa4 Qc6+ 45.f3 Qxa4

Such a 3-vs-2 queen endgame is an easy draw for the defending side.

46.Qe4 Qa2+ 47.Kh3 Qa1 48.Qc4 Kg7 49.Qd5 Qh1+ 50.Kg4 Qa1 51.Qd6 Qc3 52.Qd5 Qc8+
53.Kf4 Qc1+ 54.Ke5 Qe1+ 55.Kf4 Qc1+

½-½
Games 5 and 6 were two boring draws against Teimour Radjabov and Vassily Ivanchuk. Then in round
7, Veselin lost a tough game with Black to Sergey Karjakin. Sergey played the Grand Prix Attack in the
Sicilian. Veselin got a nice position and was even pushing for a win at some point, but after White
played a couple of good moves, Black overpressed and ended up losing the game.
That loss was very tough for Vesko psychologically as it also took him to -1 (3 out of 7). The next
game was against Levon Aronian. We played a new idea in the Queen’s Gambit, with many dangerous
hidden ideas, but Aronian knew it and responded with the best answer. Let me show you the game and
some of our ideas.
Topalov, Veselin (2770)
Aronian, Levon (2805)
Wijk aan Zee 2012

1.d4 d5 2.c4 e6 3.Nc3 Be7 4.Nf3 Nf6 5.Bf4 0-0 6.a3 b6

Position after: 6...b6

90
The critical answer to 6.a3. We had prepared a relatively new idea against it.

7.Qc2 c5

The first unpleasant surprise. We knew this was the best move!
A) 7...dxc4 8.e4 Nc6 Critical. 9.Rd1 Na5 10.Be2!?

Position after: 10.Be2!?

White plans to act on the kingside, but it’s good to complete development first. 10...c6 This is what he
considered the main move. Black wants to play ...b6-b5. [10...Bb7 11.h4! is similar.] 11.h4! b5
[11...h6 12.Ne5 b5 13.g4!‚] 12.Ng5

91
Position after: 12.Ng5

12...g6 [We considered the prophylactic 12...Nd7?! a mistake: 13.e5! g6 14.Nce4! (…h5) and for
example 14...h6 15.Nxe6! fxe6 16.Nd6 with a crushing attack.] 13.Qc1! We need the white queen in
the kingside action! [We could not make 13.Be3 work. For example: 13...Nb3 14.h5 Nxh5! (the
simplest) 15.Nxh7 Kxh7 16.g4 f5 17.gxh5 g5!.] 13...Nb3 This move isn’t forced right now, especially
as White is probably going to play Qc1-e3 whatever Black does, but it will have to be played soon
anyway in order to create counterplay on the queenside or in the centre. [13...h6 14.Be5! would be
horribly annoying, e.g. 14...Nd7 15.Nf3 Kh7 16.Bf4 h5 17.g4! hxg4 18.h5‚.] 14.Qe3

92
Position after: 14.Qe3

A1) Again 14...h6 15.Be5 is difficult for Black. Let’s see some of the lines we had: 15...h5 [15...Nd7
16.Nf3 Kh7 17.Bf4 h5 18.g4! is hardly survivable for Black.; 15...hxg5? 16.hxg5+–] 16.g4! Nxg4
[16...hxg4 17.h5 Nxh5 18.Nxe6 fxe6 19.Qh6 Rf6 20.Bxf6 Bxf6 21.Qxg6+ Ng7 22.Qh7+ Kf7
23.e5+–; 16...Nd7 17.gxh5±] 17.Bxg4 hxg4 18.h5! Bxg5™ 19.hxg6 fxg6™ [19...f6? 20.Qxg5+–]
20.Qg3 with huge compensation.
A2) 14...h5 15.g4 [We also liked 15.Kf1!?, first improving the king (just like playing Kc1-b1 after
you’ve castled long), followed by g2-g4.] 15...hxg4 [A fun line was 15...Nxg4 16.Bxg4 hxg4 17.h5
gxh5 18.Rxh5 f6 19.e5! fxg5 20.Qe4! Rf5

Position after: 20...Rf5

21.Qh1! Kf7™ 22.Rh7+ Ke8 23.Qh5+ Kd7 24.d5! cxd5 25.Nxd5 exd5 26.Rxd5++– Beautiful!]
16.e5!

93
Position after: 16.e5!

16...Nd5 [16...Nh5 17.Bxg4 Nxf4 18.Qxf4‚] 17.Nxd5 Qxd5 [17...exd5 18.h5 Bf5 19.hxg6 fxg6
20.Rh6‚ isn’t any fun either.] 18.Rg1 c5 19.dxc5 Qxc5 20.Qe4! Qc7 21.Nxf7 Rxf7 [21...Kxf7
22.Bxg4‚] 22.Rxg4 Bb7 23.Qxg6+ Kh8

Position after: 23...Kh8

Here I spotted something amazing looking at our old file with a recent computer. Such a line is a
good illustration why we had tried 15.Kf1 instead of the immediate 15.g4, hiding the white king

94
from checks! Here, the f7-rook is not hanging because of ...Bb4+ followed by ...Qxf7. However, the
engine shows 24.Rd6!! and it’s game over: 24...Rh7 25.Bh6+–.
B) Even before that game was played, I won one myself in similar fashion to the above: 7...Bb7!?

Position after: 7...Bb7!?

8.Rd1 dxc4 9.e4 b5 10.h4! h6 11.Be2!? [11.Rh3!?N was another good try, but is probably only a
draw: 11...a5!? 12.e5 Nd5 13.Bxh6 gxh6 14.Rg3+ Kh8 15.Qd2 Kh7=] 11...b4?! [11...a5!? was surely
stronger.] 12.axb4 Bxb4 13.e5!

95
Position after: 13.e5!

13...Nd5? [13...Be4 was necessary: 14.Qd2!? Nd5 15.Rh3©] 14.Bxh6! Winning by force! 14...gxh6
15.Ng5! hxg5 [15...f5 16.Nxe6 Qd7 17.Nxf8 Kxf8 18.Bxc4+–] 16.hxg5

Position after: 16.hxg5

16...f5 [16...Re8 was the only move, but losing anyway: 17.Qh7+ Kf8 18.g6! Qd7 19.g7+ Ke7
20.Qh4+ f6 21.Qh7! Kd8 22.Bh5+–] 17.gxf6 Rxf6 18.Qh7+ Kf8 19.Qh8+ 1-0 Edouard, R (2608) –
Van Der Wiel, J (2459) Antwerp 2011; Black resigned in view of 19...Ke7 20.Rh7+ Rf7 21.Rxf7+
Kxf7 22.Qxd8+–. I could never have added such a game to my account without Topalov’s influence!

8.Rd1

96
Position after: 8.Rd1

8...Nc6!

Another nasty surprise. This move is a powerful equalizer.


8...Bb7 is not as good: 9.cxd5 Nxd5 10.dxc5

Position after: 10.dxc5

10...Qc8!? [10...bxc5 11.e4 Nxf4 12.Rxd8 Rxd8 is similar to the game (if White played 10.e4 to win

97
the queen), except this time White has an improved version with 13.Qb3! and ...Bb7 appears to be a
much worse move than ...Nc6.] 11.Nxd5 Bxd5 [11...exd5 12.e3 bxc5 13.h4!? (…Ng5) or 13.Bd3 is
great for White.] 12.e4!? Bb7 13.Rc1

Position after: 13.Rc1

13...bxc5 [13...Qxc5? 14.Qb3 Qh5 15.Rc7+–; 13...Bxc5?! 14.b4 Be7 15.Qb1 Qd8 16.Bb5±] 14.Be2!?
Qc6 15.Nd2 and we assessed the position as better for White.

9.cxd5 Nxd5!

Now it was clear that Aronian simply knew about our new idea. His preparation was obviously too
good, meaning that our whole new idea was spoiled. In general you hope to play something new
against someone who doesn’t know it at all!
9...Nxd4 is nevertheless a very suitable alternative. 10.Nxd4 cxd4

98
Position after: 10...cxd4

A) 11.d6 Bxd6 12.Rxd4 Bxf4 13.Rxd8 Rxd8 14.e3 Be5 15.Be2 Bd7 16.0-0 Rac8 should be an easy
draw for Black.
B) White should probably play 11.Rxd4, after which it could be that Black doesn’t fully equalize:
11...Nxd5 12.Nxd5 exd5 13.e3 with the idea Be2, 0-0, and Rd2 against ...Bf6.
C) 11.dxe6 Bxe6 12.Be5 [12.e3 Nd5!∞] 12...Bc5! [12...Nd5 13.Bxd4 Nxc3 14.Bxc3 Qc7 15.e4²]
13.e3 dxe3!

Position after: 13...dxe3!

99
Fire on board! 14.Rxd8 exf2+ 15.Kd2 Raxd8+ 16.Kc1 Ng4 17.Bc7 Be3+ 18.Kb1 Rd2 and Black has
huge compensation!

10.dxc5 bxc5

Position after: 10...bxc5

This is the whole point. Without preparation one would be afraid of 11.e4, forcing Black to give up his
queen, but it simply leads to a very decent position for him. Aronian clearly knew what he was doing,
and we had made it clear with Veselin that if this sad situation arose, he should just play a quiet
drawish move here, which he did.

11.e3

11.e4 Nxf4 12.Rxd8 Rxd8 13.g3 Ng6 and Black has nice compensation.

100
Position after: 13...Ng6

Levon, who likes to ‘tease’ a bit at press conferences, said something like, “I didn’t know anything, but
I thought the ending a queen down looks much better for Black”. A funny comment, as he knew for
sure what he was doing, and ‘better for Black’ is very exaggerated (it should be equal). No
misunderstanding then – I’m a big fan of Levon both as a player and a fun person! A sense of humour
is something very positive.

11...Qa5 12.Ne5 Nxe5 13.Bxe5 Rd8 14.Bd3 Ba6

101
Position after: 14...Ba6

And with the following exchanges our lovely, rich preparation fizzles out to a boring, short draw.

15.0-0 Bxd3 16.Qxd3 Nxc3 17.Qxc3 Qxc3 18.Bxc3 Kf8 19.Rxd8+ Rxd8 20.Rc1 g6 21.Kf1 Ke8
22.Ke2 a6 23.g4 Rd6 24.Be5 Rd5 25.f4 f6 26.Bc3 f5 27.h3 Kd7 28.a4 ½-½

An informal get-together after the game (photo Fred Lucas)

102
In the next game Vesko faced Hikaru Nakamura with the black pieces. We somehow guessed the
opening and brought a slightly forgotten system back to life. Let’s have a look!
Nakamura, Hikaru (2759)
Topalov, Veselin (2770)
Wijk aan Zee 2012

1.c4 e5 2.Nc3 Nf6 3.g3 c6 4.Nf3

4.d4 is the other main move. Both options have the same objective: to disturb Black’s centre before he
plays ...d7-d5.

4...e4 5.Nd4

Position after: 5.Nd4

5...Qb6!?

Initially, Veselin had asked me to check the line 5...d5 with the idea ...Qb6, but during the night I found
out it was dubious: 6.cxd5 Qb6 [6...cxd5 7.d3 Bc5 8.dxe4 dxe4 9.Be3²] 7.Nb3! cxd5 8.Bg2 Nc6
[8...Be7 9.d3 exd3 10.Qxd3²] 9.d3! exd3. After some investigation, I understood ...Qb6 before ...d5 to
be the right path! 10.Bg5!²

6.Nb3

Critical. Other moves aren’t too dangerous.


A) 6.e3 d5 7.cxd5 cxd5 8.d3 Nc6 9.dxe4 Nxe4! is just fine for Black, as after 10.Nxd5 Qa5+ 11.Nc3

103
Bb4 12.Bd2 Nxd2 13.Qxd2 Nxd4 14.Qxd4 0-0© he has tremendous compensation for the pawn.
B) 6.Nc2 d5 7.Bg2 [7.cxd5 cxd5 8.Bg2 Nc6=] 7...Be6 is also a good line for Black. [7...dxc4!?]

6...a5!

Position after: 6...a5!

Chasing the knight away from b3 before going ...d7-d5.

7.d3

A) The natural 7.Na4 is answered by the unnatural 7...Qa7!N [and not 7...Qb4?! 8.Nd4²]. Black
wants to push ...b7-b5. 8.c5 d5!? Now a sample line goes 9.d4 Nbd7 10.Qc2 [10.Bg2 b5„] 10...Be7
11.Bf4 0-0 12.Bc7 Bd8. Black will end up pushing ...b7-b5 and will be fine.
B) 7.a4 avoids the threat but creates a big hole on b4. Black may sac a pawn: 7...d5 8.cxd5 Nxd5
9.Nxe4 Be6 with nice compensation, 10.Bg2?? being a horrible blunder due to 10...Nf4! 11.gxf4
Bxb3 trapping the white queen!

7...a4

104
Position after: 7...a4

8.Nd2

8.Be3!? Another interesting move which Nakamura played a few months later. However it doesn’t
bring White any advantage. 8...Qd8 [Our plan was actually 8...Qb4!? when, for example, 9.Nd2 a3
10.Ncxe4 Nxe4 11.bxa3 Nxf2 is pretty unclear.] 9.Nd2 [9.Nd4 a3∞; 9.Nc5 Ng4!∞] 9...exd3 10.Nxa4
d5 11.Bb6 Qe7 12.Bc5 Qc7 13.Bxf8 Kxf8∞ 0-1 (41) Nakamura, H (2778) – Wang, H (2739) Biel
2012.

8...exd3

8...d5 doesn’t equalize: 9.cxd5 cxd5 10.dxe4 dxe4 11.Bg2 e3 12.fxe3 Be7 13.Nde4 Nbd7 14.0-0 0-0
15.Qd4².

9.exd3?!

I had pinpointed this move as definitely dubious during my work.


9.Bg2! is critical. I’ve played a few games here, with both colours in fact! Herewith some extracts from
my analysis for Veselin, combined with two of my games.

105
Position after: 9.Bg2!

A) 9...a3 is not the best: 10.Rb1!? Be7 [10...d5?! 11.cxd5 Qa6 12.0-0 Bb4 13.exd3 0-0 14.dxc6 Nxc6
15.Nde4 Nxe4 was my own game Edouard, R (2622) – Warakomski, T (2518) Warsaw 2017, ½-½
(32), and here 16.Bxe4! would have given me a great advantage.] 11.0-0 0-0 12.exd3 axb2 13.Bxb2
Qd8 14.Re1ƒ and White has some pressure.
B) 9...Bb4!? 10.exd3 0-0 11.0-0 Qa5 is also a decent line.
C) 9...d5 10.0-0! [10.exd3 Bg4! is fine for Black e.g. 11.Bf3 Bh3 12.cxd5 Be7 13.Nc4 Qd8 14.d6
Bxd6 15.Nxd6+ Qxd6 16.Qe2+ Qe6 17.Qxe6+ fxe6= ½-½ (40) Maze, S (2573) – Edouard, R (2652)
Pau 2012.] 10...d4 [This time 10...Bg4 11.Bf3! poses problems.] 11.Nce4 [11.Nxa4? Rxa4 12.Qxa4
dxe2 13.Re1 d3µ] 11...Nxe4 12.Nxe4 Be7 [Not 12...dxe2?? 13.Qxe2 Be7 14.Nd6++–.] 13.Qxd3
[13.exd3 0-0 14.Re1 Be6=] 13...f5!N 14.Nd2 0-0 15.Nf3 Bf6∞

9...Bc5!

106
Position after: 9...Bc5!

A nasty surprise. Black is better!

10.Qe2+ Kd8 11.Nde4 Re8 12.Be3!?

From now on, Veselin was playing on his own. Instead, my ChessBase file went 12.Bg5 h6 13.Bxf6+
gxf6 14.Bh3 d5 15.Bxc8 dxe4 16.Nxe4 Kxc8 17.Nd6+ Bxd6 18.Qxe8+ Qd8³.

12...Bxe3

107
Position after: 12...Bxe3

13.fxe3

13.Qxe3 Qxe3+ 14.fxe3 Nxe4 15.Nxe4 d5³

13...a3!

The best move.


13...Nxe4 would force 14.dxe4, but opening the d-file is not something Black really wants when is king
is standing on d8. However the engine prefers Black after 14...d6 15.0-0-0 Kc7 followed by ...Bd7,
...Qc5 and ...Ne5.

14.Rb1 d5 15.c5 Qb4 16.Nxf6 gxf6 17.Qd2

Position after: 17.Qd2

17...axb2?!

Natural, but not best.


A) 17...d4! was more convincing: 18.Ne4 Na6! followed by ...f5 was close to a decisive advantage.
B) 17...Qxc5 18.d4 Qe7 was also a clear advantage for Black, similar to the game.

18.Nd1!

18.d4 Bf5 19.Rxb2 Rxe3+! was probably what Veselin was hoping for.

108
18...Qxc5 19.d4 Qd6 20.Rxb2

White is clearly worse, but he just about holds the position. Black’s doubled f-pawns are likely to cost
him half a point at the end of the day.

20...b5 21.Bd3 Bh3 22.Nc3 Nd7 23.Kf2 Nb6 24.a4 Nxa4 25.Ra1

Position after: 25.Ra1

25...Re7?!

25...Ke7!, to move the king to the other side of the board and link rooks, was the best move: 26.Nxa4
Rxa4 27.Rxa4 bxa4 28.Qd1 Qe6 29.Qxa4 Kf8µ. Black is still far from the full point, but he can press
forever with an extra pawn.

26.Rba2 Rea7 27.Qc2 h6 28.Nxa4 bxa4 29.Rxa4 Rxa4 30.Rxa4 Rxa4 31.Qxa4

109
Position after: 31.Qxa4

Without rooks on the board, Black has no real winning chances.

31...Ke7 32.Qa7+ Kf8 33.Qc5

And without queens even fewer!

33...Qxc5 34.dxc5 Bg4 35.Bc2 Ke7 36.Ba4 Kd7 37.Ke1 Bf5

½-½
In the next game he took on Fabiano Caruana with the white pieces. Fabiano played a great game,
Veselin overpushed, ended up slightly worse and lost a pawn. He still had drawing chances, but made a
wrong defensive queen sacrifice and lost.
Topalov, Veselin (2770)
Caruana, Fabiano (2736)
Wijk aan Zee 2012

110
White to move

23.Bb3?

A blunder.

23...Ng4!

Black has three deadly threats: ...Rxe4, ...Qh4, and even ...Qxf4. The first on the list was overlooked by
Veselin.

24.g3

The best reaction, but of course Black wins a pawn.

24...Rxe4 25.Rxe4 Nf2+ 26.Qxf2??

A terrible reaction! White could have played a pawn down and just a slightly worse position.
26.Kg2 Nxe4 27.Qf4 and obviously White does not have full compensation, but Black would have had
a lot of work to win.

26...Bxf2 27.Kg2 Bc5 28.h4 Qf5 29.Re2 Qg4 30.Rde1 g5! 31.Re4 Qf5 32.R1e2 gxh4 33.Nf4 Qg5
34.Kh1 d5 35.Bxd5 hxg3 36.c4 Qh4+ 37.Kg2 Qh2+ 38.Kf3 Qh1+ 39.Kxg3 h4+

0-1

After this game, we had to go to the official tournament dinner. Hundreds of people were there: the
organizers, all the people who supported the tournament in one way or another, the players and their

111
seconds. Vesko had to give a public interview, answering questions from Ivan Sokolov, a strong player,
great author, and sometimes apparently even a brilliant commentator!

Ivan and Veselin during the interview (photo Fred Lucas)


The interview was very lively and I was impressed with the positive mood Vesko showed. As a player,
I know how difficult it is to smile and be sociable an hour after a loss. In fact the first question he had
to answer was how he could take losing a game so easily. He replied honestly, saying once you have
played World Championship matches in your career, the rest appears less important. To be more
precise, he even said that when you lose a World Champion title on the last day, that costs you at least a
million euros. Of course he is still motivated for the rest, but it puts things into perspective.

112
We had some fun that evening! (photo Fred Lucas)
Then he was asked about his match with Kramnik in 2006. He answered very simply, saying that
especially in a match you should be able to ask questions when you feel something wrong is happening.
He more or less said the same as he did to me (see previous chapter), but talked more in generalities
than precise examples, as it had to be brief! He responded to all these questions with an open heart and,
whatever percentage of people in the room were on his side, his comments were received with
applause.

113
A little dark humour...
Once the interview and dinner were over, Veselin tried to focus on the next game, the toughest of all,
Black against Magnus Carlsen. Magnus played the Moscow variation of the Sicilian but was outplayed
by Vesko up to a particular point. Black was completely winning, but Magnus always finds so many
resources that any position against him is war, and the final result wasn’t satisfactory for us... Let’s
have a look at this amazing game!
Carlsen, Magnus (2835)
Topalov, Veselin (2770)
Wijk aan Zee 2012

1.e4 c5 2.Nf3 d6 3.Bb5+ Nd7 4.c3 Ngf6 5.Qe2 a6 6.Ba4 Qc7 7.0-0 e5 8.d4 b5 9.Bc2 cxd4 10.cxd4
Be7 11.Nc3 0-0 12.Bg5 h6 13.Bh4 Bb7 14.Rad1 Rac8 15.Bb3 Rfe8 16.dxe5 dxe5 17.Bg3 Bf8 18.h3
Nb6

114
Position after: 18...Nb6

Veselin has reached a very comfortable position and is going to play ...Nc4 next. If White doesn’t want
to just sit and be worse, he needs to counter-attack at any price – which is what Magnus did!

19.Nh4 Nc4 20.Nf5 Nxb2 21.Bh4!

The position is becoming completely crazy: everything is hanging!

21...Nxd1

21...g5 was a tempting defence, just preventing Bxf6 and Qg4 as in the game. Now the best
continuation for White by far is just stunning...

115
Position after: 21...g5

22.Rd6!! [After 22.Rc1 Nc4 White does not have enough compensation.] 22...Bxd6 23.Nxh6+
Probably both critical answers lead to a draw here.

Position after: 23.Nxh6+

A) 23...Kh7 24.Bxg5 [24.Qf3?! is tempting but doesn’t work: 24...gxh4 25.Qf5+ Kxh6 26.Qxf6+
Kh7 27.Bxf7 Rg8! 28.Qf5+ Kg7 29.Qg6+ Kf8 30.Qxg8+ Ke7 31.Qh7 Rf8! and Black is better.]
24...Re6! A key defensive move, as we will see very soon in the game. [24...Be7 25.Nf5 would be

116
very dangerous for Black, as would 24...Rg8 25.Nxg8 Nxg8 26.Nd5 Bxd5 27.Bxd5‚.] 25.Bxe6 fxe6
26.Bxf6 Kxh6 27.Qg4 Qf7 28.Qg5+ Kh7 and White has nothing better than a draw: 29.Qh4+ Kg6
30.Qg5+ Kh7=.
B) 23...Kf8 24.Bxg5 Be7! 25.Nf5 [25.Bxf6? Bxf6 26.Qg4 Bg7 isn’t convincing for White.]
25...Qxc3 Taking all the material. White has a draw, but no more. 26.Bxf6 Bxf6 27.Qh5 Re6!
28.Bxe6 fxe6 29.Qh6+ Ke8 [29...Kf7 30.Qh7+=] 30.Qxf6 exf5 31.Qe6+=.

22.Bxf6 Nxc3 23.Qg4

Position after: 23.Qg4

This was the whole idea.

23...Bxe4?!

A) 23...Re6!! was the strongest move. The position is insanely complicated, but Black obtains a
decisive advantage against any white answer. 24.Nxg7 [24.Bxe6 fxe6 25.Ne7+ Kh8™ 26.Qg6 Bxe4
27.Qxh6+ Bh7–+; 24.Nxh6+ Kh7 25.Nxf7 Qxf7 26.Bxe6 Qxf6–+; 24.Bxg7 Kh7! 25.Bxf8 Rg6–+]
24...Rxf6 25.Ne8+

117
Position after: 25.Ne8+

25...Kh8 [Of course not 25...Rg6?? 26.Qxg6++–.] 26.Nxf6 [In case of 26.Nxc7 Rxc7–+ Black has far
too many pieces for the queen.] 26...Bg7 27.Qf5 Bxf6 28.Qxf6+ Kg8 29.Qg6+ Kf8 30.Qxh6+ Ke8–
+
B) Just like the game move, 23...Kh7?! isn’t quite precise. 24.Nxg7

Position after: 24.Nxg7

24...Qc6!? Not the best move objectively, but definitely the best practical try. [24...Ne2+ 25.Kh2 Nd4

118
26.Nh5! is very unclear; 24...Bxe4 25.Nxe8 Rxe8 isn’t so clear due to 26.Rd1! Bg6! 27.Rd7 and now
to be slightly better Black must find 27...Ne2+! 28.Kf1 Ng3+! 29.fxg3 Qc1+ 30.Ke2 h5 31.Qg5
Qxg5 32.Bxg5 Rc8³.] 25.Qf5+ [25.Nxe8? fails to 25...Qxe4 26.f4 Nd5µ] 25...Kg8 26.Nxe8 Qxe4

Position after: 26...Qxe4

At first it looks like what White just did was totally dumb, but he actually draws with the magnificent
27.Bxf7+! Kxf7 28.Nd6+! Bxd6 29.Qd7+ Kxf6 30.Qxd6+ and the black king can’t escape the
checks!

24.Nxh6+?

24.Nxg7 was required, and after 24...Kh7 would transpose to the line 23...Kh7 24.Nxg7 Bxe4 above,
where White might have missed that he could get a relatively unclear game by playing 25.Nxe8 Rxe8
26.Rd1!„.

24...Kh7

119
Position after: 24...Kh7

25.Bxf7?

Just throwing all his pieces at Topalov’s king and hoping he will collapse. When following the game
online, I wanted to shout at Vesko, “No, don’t sac your queen like you did against Fabiano!”. I will
always wonder, if I could have, whether he would have listened...
25.Nxf7 was the least damaging, but – and this is the other thing Magnus might have missed – Black
has the killer 25...Bd5!, winning for him: 26.Bc2+ Ne4 27.Ng5+ Kg8 28.Bxe4 Bxe4 29.Nxe4 Qf7–+.

25...Qxf7??

I was really afraid to see that coming! In reality, White’s threats were mostly illusions. I switched my
engine on to ten moves, and the first ten are totally winning! The top three moves were 25...Bd3,
25...Qa7 and 25...Qb7, all giving at least a +5 advantage in Black’s favour. Otherwise 25...Rb8
(…...Re7) or 25...Kh8 were also great options.
Someone willing to choose a concrete solution could have chosen 25...Re7 26.Bxe7 Qxe7 27.Qxc8
Ne2+ 28.Kh2 Kxh6 29.Qxa6+ g6–+ with a decisive advantage for Black anyhow.

26.Nxf7 gxf6 27.f4!

120
Position after: 27.f4!

A pity! Now the position is a mess and becomes much easier to play for White.

27...Bg6

A fun draw was 27...exf4 28.Rxf4 f5 29.Qh5+ [29.Rxf5 Bxf5 30.Qxf5+ Kg8 31.Qg6+ Bg7 32.Nh6+
Kh8=] 29...Kg7 30.Ng5 Ne2+! 31.Kh2 [31.Qxe2? Rc2–+] 31...Nxf4 32.Qf7+ Kh6 33.Qf6+ Ng6
34.Nf7+ Kh7 35.Ng5+ Kh6=.

28.Qh4+ Kg7 29.fxe5 Ne4?!

29...Nd5! was the most flexible defensive square, so that after 30.Rxf6 Bc5+ 31.Kh2 Black has the
extra option of playing 31...Bxf7 which actually forces a draw:

121
Position after: 31...Bxf7

32.Qg5+ Kf8 33.Rh6 Bg8 34.Rh8 [34.Rg6 Bf7=] 34...Kf7 35.Rh6 Kf8=

30.Rxf6 Bc5+ 31.Kh2 Nxf6™

31...Bxf7?? 32.Qxe4+–

32.Qxf6+ Kh7 33.Ng5+ Kh6 34.Ne6!? Rxe6™ 35.Qxe6 Re8 36.Qf6

122
Position after: 36.Qf6

Of course Black should still be fine, but in practice White’s position is much easier, not to mention the
psychological effect of what has just happened.

36...Be7?!

36...a5, not allowing Qxa6, was a good start not to lose the game. 37.h4 Be7 38.Qf4+ Kg7=

37.Qxa6 b4 38.Qc4 Bf8?!

38...Kg7 was a tougher defence, when it will be difficult for White to make progress. If 39.g4, as in the
game, Black can now play 39...Bf7 [39...Rd8!?; 39...Ra8!?] 40.e6 Bd6+=.

39.g4 Kh7?!

A) Coming back with 39...Be7 was the least damage.


B) Of course 39...Rxe5?? loses to 40.Qf4++–.

40.e6 Bd6+ 41.Kg2 Be7 42.Qc7 Kg8 43.Kg3 Kf8 44.Qf4+ Kg7

Position after: 44...Kg7

Things are going downhill for Black, but White should still take a little care technically.

45.Qd4+?

45.h4! Rd8 (…...Bd6) 46.Kg2! was simple and strong.

123
45...Kg8 46.h4 Rd8 47.Qc4?!

Giving Black the chance of a forced draw.


47.Qa7! was the last opportunity to create problems.

47...Bd3?

47...Rd3+! was a draw: 48.Kf4 [48.Kg2 Rd2+ 49.Kf3 Bxh4=] 48...Bd6+ 49.Kg5 Kg7! and White can’t
avoid perpetual check (...Be7+, ...Bd6+).

48.Qc6 Bb1?

48...Kg7! was the last hope, willing to answer 49.h5? with 49...Kf6!„.

49.h5

Now it’s all over; White’s pawns are too advanced and the black king will soon be in trouble.

Position after: 49.h5

49...Bxa2 50.Qe4 Kh8

50...b3 51.h6 with mate to come; if 51...Kh8 then 52.Qe5++–.

51.h6 Bf6 52.e7 Re8 53.Qf4 Bg7 54.hxg7+ Kxg7 55.g5 Kg8 56.Qf6

A tough loss, but a beautiful game from both sides. The quality of a chess show doesn’t depend on the
perfection of the game!

124
1-0

Magnus Carlsen vs Veselin Topalov (photo Fred Lucas)


In the following game, Vesko was facing Anish Giri with the white pieces.
The story behind this game is quite funny. I normally received the to-do work the day before each
game, but from the beginning of the tournament Veselin had given me two things to check when I had
some extra time: the variation that he used against Nakamura, and Anish’s beloved Petroff. That meant
that I had spent several hours every day for a week looking at it – and the game went straight into the
preparation!
Topalov, Veselin (2770)
Giri, Anish (2714)
Wijk aan Zee 2012

1.e4 e5 2.Nf3 Nf6 3.Nxe5 d6 4.Nf3 Nxe4 5.Nc3 Nxc3 6.dxc3 Be7 7.Be3 Nc6 8.Qd2 Be6 9.0-0-0
Qd7 10.Kb1 Bf6

125
Position after: 10...Bf6

This line is the favourite of top Chinese players. If you look up all the recent games, you will find Wei
Yi, Li Chao, Bu Xiangzhi and Ni Hua largely dominating the spreadsheet!

11.Bg5!?

At the time of the game, this move was very unexplored.


11.h4 and 11.Ng5 were the main moves. Anish had several games there!

11...Qe7

The best option should be 11...Bxg5 although after 12.Nxg5 0-0-0 13.Nxe6! I believe Black does not
equalize:

126
Position after: 13.Nxe6!

A) 13...fxe6 14.g3! e5 15.Bg2 Rhf8 16.Qe3 Black’s position is solid but unpleasant. White will play
Rhe1, Rd2 (to protect f2 if attacked), h4 and c4, aiming for c4-c5. Black’s structure is inflexible and
White’s bishop is superior to Black’s knight.
B) 13...Qxe6 14.Bb5! This was the plan, and was played by Hou Yifan in... 2018! 14...Qf5 15.Bxc6
bxc6 16.Rhe1 Rhe8 17.f3 and Black is suffering; he has a weaker king and White will slowly push the
kingside and queenside pawns. In the long term, the a7-pawn is a serious problem, and a pawn ending
could be lost! Hou Yifan (2680) – Wei Yi (2743) Wijk aan Zee 2018, ½-½ (45).

12.Bb5

127
Position after: 12.Bb5

12...a6

12...0-0 13.h4! h6 14.Qf4 Ne5 15.Bxf6 Qxf6 16.Qxf6 gxf6 17.Nd4²

13.Bxc6+!

A novelty at the time.


13.Ba4 had been played by Ivanchuk but did not convince me. For example after 13...b5 [13...0-0!?]
14.Bb3 0-0N [14...Ne5 15.Bxf6 Nxf3 16.Qe3! Qxf6 17.Bd5! was slightly better for White in
Ivanchuk, V (2779) – Gashimov, V (2746) Monaco 2011, 0-1 (28)] 15.h4 Rae8 with the idea ...h6 or
...Qd8 I did not see any perspectives for White – the bishop on b3 is misplaced.

13...bxc6 14.Qf4!

Threatening to weaken Black’s pawn structure.

14...Bxg5 15.Nxg5 0-0

128
Position after: 15...0-0

16.Rde1?!

Enough to apply pressure, but worse than the suggestion in my analysis file, the far more natural
16.Rhe1. When Veselin picked the other rook, I thought he probably did not remember my suggestion,
but immediately after the game he told me, “Over the board I thought I was improving on your
analysis, but my choice was dumb!”.
My analysis continued 16.Rhe1 and I considered White to be better after 16...Qd7 17.Nxe6 fxe6
18.Qc4! d5 [18...Rf6 19.g3²] 19.Qe2 Rae8 20.f3².

129
Position after: 20.f3²

The a6-pawn is hanging, and White will play c3-c4 next, with a more flexible pawn structure.

16...Qd7 17.f3!?

17.Nxe6 fxe6 18.Qc4 is no longer clever without a rook on d1: 18...d5 19.Qe2 Rae8! 20.f3 a5 and
Black is just fine, as White will be too slow to pose questions in the centre with c3-c4.

17...h6?!

The hard-to-find 17...Bf5! would have been close to a draw:

Position after: 17...Bf5!

18.h4! [18.g4 f6!=] 18...h6 [18...f6 19.Ne4 a5 20.c4 Rfe8„] 19.g4 Bg6 20.h5!? [20.Nh3 Rfe8„]
20...hxg5 21.Qxg5 Qd8!=

18.Nxe6 fxe6 19.Qg4 Rf5 20.Re2 e5 21.Rd1²

Finally the rook is back on d1, and we more or less got what we wanted! Veselin poked fun at himself
after the game.

130
Position after: 21.Rd1²

About the position itself: why is White better?


At first, the position looks quite dry, but White’s pawn structure is much better. It is more flexible, as
White can aim for two breaks in the middle/long term: c4-c5 and f3-f4. Black lacks such goals, plus the
a6-pawn is a long-term weakness. Topalov will demonstrate these problems very well.

21...Qf7 22.Qb4 Qd7 23.Qa4 Kh8 24.c4 Qe8 25.Rd3!?

A clever move: often, the rook will be able to shift to a3, especially if the queens are gone.
25.c5!? was possible right away. If Black answers 25...dxc5 or 25...d5, White will continue with
26.Rde1 and target the e5-pawn.

25...Rf4

25...c5 26.Qxe8+! Rxe8 27.Ra3 Ra8 28.Ra5 would be terribly unpleasant for Black. Remember the a6-
pawn!

26.Qa5 Qe7 27.c5!

131
Position after: 27.c5!

27...Rf6?!

Black tries to sit tight, but it was probably not the most resistant. He remains paralyzed and White will
soon be dominating both sides of the board.
A) 27...d5 28.Rde3 e4 29.b3 is definitely difficult for Black.
B) In case of 27...dxc5 White can choose between b2-b3 and a2-a3, and then target the e5-pawn. Of
course it is important to avoid back rank mate problems. Usually the first suggested move is more
harmonious, but with the extra possibility of ...c5-c4 counterplay, the second makes a lot of sense too.

28.Qc3 Qe8 29.b3 Rb8 30.a4 Ra8 31.Re4 Qc8 32.Qd2 Qf8 33.Qa5 Qe7 34.Rde3 Qd7 35.Re1 Rf5
36.Qc3 Rd8

132
Position after: 36...Rd8

As you can see, White has taken his time to organize all of his pieces the way he wants. Now it’s time
to act!

37.g3! Rf7 38.cxd6 cxd6 39.f4!

Finally the break! White wants Re4-e6 and to target the black king. As mentioned earlier, Black is
much worse on both sides.

39...exf4 40.gxf4 c5

40...d5 41.Re6 d4 42.Qxc6 should be an easy win for White.

41.Re6! Rxf4

41...Kg8 42.f5! is crushing, since 42...Rxf5? loses to 43.Re7+–.

42.Rxh6+ Kg8 43.Qd2 Rf7 44.Rg1

133
Position after: 44.Rg1

44...c4?!

After this move, Black loses right away, but the engine’s best suggestion, 44...Qf5, loses immediately
anyway: 45.Rxd6+–.

45.Qg5 Rdf8 46.Qh5 Rf1+ 47.Kb2!

And Rh6-h8 is unavoidable.

1-0

Anish, who is a good friend of mine since we played for many years for the same European teams, told
me after the game, “Come on, I’m sure you were targeting me from the beginning, otherwise how could
you find this idea!”, and then added, “I knew that after your novelty, White was a bit better, but even
2750+ players don’t find it because it’s unusual”. I initially took it as a big compliment until he added a
typical Giri-joke: “I should have known that having a fresh look at the position without any knowledge,
you would find it!”. It actually took me months before I could tell him that all his suspicions were right!
In the last round, Vesko faced Gata Kamsky with the black pieces. He was a little better but could not
find his way, after which the position was equal. The only thing I remember from that day is that at
some point they started to repeat moves. Seeing that, I headed for the playing hall to join Veselin.
When I was half-way, I remembered Veselin never repeats a position three times, and went back to the
hotel! He did decline a draw but eventually lost, a final twist of fate, but hardly any change in what had
been an unsuccessful event anyway.
Veselin finished the tournament on a dismal -3, a very sad result given the number of games he should

134
at least not have lost. But, as he always says: Life Goes On!
If you are wondering what Topalov’s reaction towards me may have been after such a bad result, he
congratulated me on my work and said he was aware how many excellent positions he hadn’t handled
in the best way. He immediately offered me more training sessions.
Having many friends who have helped top players in the past, I can assure you that this is far from
usual when the final result has not been satisfactory!

135
Chapter 4
London: First Success!

For readers not familiar with the rules of the FIDE Grand Prix series in 2012-2013, they were as
follows: each player would participate in four of the six Grand Prix tournaments; for every tournament,
each player would be awarded a certain number of points depending on his place (from 170 points for
first to 10 points for twelfth). For each player, only the best three results would count in the final
ranking; the worst one would be discounted. At the end of the series, the top two players in the general
ranking would qualify for the next Candidates tournament.
In September 2012, the adventure started with the first Grand Prix taking place in London.
In the first round, Veselin faced Grischuk with the white pieces. He got some advantage out of the
opening, but lacked precision at a later stage and had to agree a draw.
There was a weird story about Veselin’s second round game when he faced Gelfand with the black
pieces. Read about it below!
Gelfand, Boris (2738)
Topalov, Veselin (2752)
London 2012

1.d4 Nf6 2.c4 e6 3.Nf3 d5 4.Nc3 Be7 5.Bf4 0-0 6.e3 Nbd7 7.Be2 dxc4 8.0-0 c5 9.dxc5 Nxc5
10.Bxc4 Qxd1 11.Rfxd1 b6 12.Nd4 Bb7 13.Rac1 a6 14.b4 Nce4 15.Nxe4 Bxe4 16.a3

Position after: 16.a3

All this had already been played before (actually by Gelfand as White) and is considered theory. This

136
position also gave birth to a surprising story which I’ll recount right now.
Before the tournament I had told Vesko I wouldn’t be able to work much during the first two days as I
was busy in Paris, and asked him to tell me upfront what he would need done. He sent me a list of
questions one week prior the event, one of which was how to solve the black problems in this endgame.
I answered by email, telling him to play 16...a5, and explained why it is a draw. As I was watching the
game live I saw that he played all the moves until here à tempo. I was expecting him to continue
playing fast and use my novelty 16...a5, but after a very long time (I think around half an hour), he
played differently, and actually played even worse than Kramnik did in his game against Gelfand one
year earlier.
I was shocked and thought Topalov might have found a big hole in my idea, but after the game he told
me, “I initially planned to go for another opening, then changed my mind and forgot to read your
email!”.
Luckily, he was able to hold this game, but, very soon in this book, you will encounter the second part
of the story, which is even more amazing!

16...Rfc8 17.f3 Bd5?!

Worse than 17...Bb7, as played earlier, after which Black suffers a bit, but seems to hold: 18.e4 a5

Position after: 18...a5

19.Nxe6 [19.Nb5 axb4 20.Bd6 Kf8 21.axb4 Ne8=] 19...axb4 20.axb4 b5 [20...fxe6 21.Bxe6+ Kf8
22.Bxc8 Bxc8 23.Bd6 is terrible for Black: with such excellent coordination and centre, a rook and two
pawns are much stronger than two pieces.]
A) 21.Nc7 bxc4 22.Nxa8 Bxa8

137
Position after: 22...Bxa8

At first it may look like Black has an easy life, but he must be careful. Although he’s doing OK with
material, the f3-e4 chain is making a fool out of the a8-bishop, and the c4-pawn is weak. 23.Bd6
[23.b5!? was possible, but should probably end up in similar fashion.] 23...Kf8 24.Bxe7+ Kxe7
25.Rd4 c3 26.Rd3 c2 27.Rd2 Nxe4! 28.Rdxc2 [28.fxe4 Bxe4=] 28...Rxc2 29.Rxc2 Nd6= ½-½ (48)
Gelfand, B (2744) – Kramnik, V (2800) Moscow 2011.
B) I believe Gelfand was planning to improve on his game with Kramnik by playing 21.Bb3, although
it is probably a draw if Black is precise: 21...Rxc1 22.Rxc1 fxe6 23.Rc7

138
Position after: 23.Rc7

23...Bxe4!? 24.Bxe6+ Kf8 25.fxe4 Ra1+ 26.Kf2 Nxe4+ 27.Ke2 Bxb4 28.Rf7+ Ke8 29.Rxg7 Re1+
30.Kf3 Nd2+ 31.Bxd2 Bxd2 32.Bd7+ Kf8 33.Rxh7 b4©.

18.Be2 Bb7 19.e4 h6

19...a5 20.Rxc8+ Rxc8 21.bxa5 bxa5 22.a4 is similar to the game.

20.Be3

Position after: 20.Be3

At first it may look like the position is about equal, but the g2-f3-e4 chain is powerful against the b7-
bishop. Black gets similar endgames in the Queen’s Gambit Accepted, and always has to be very
careful: without a concrete solution he is worse.

20...a5 21.Rxc8+?!

Unnecessary.
21.bxa5! bxa5™ [21...Rxc1?! 22.Rxc1 Rxa5? 23.Rc7+–; 21...Rxa5? 22.Rxc8+ Bxc8 23.Nc6+–] 22.a4
This is really no fun for Black.

21...Rxc8 22.bxa5 bxa5 23.Rb1 Ba8 24.a4 Bb4

Natural, and it seems Black is holding, but White’s advantage is much greater than it looks.
24...Nd7! with the idea ...Nc5 was a tougher defence.

139
25.Kf1?!

Too slow. Now Black holds relatively comfortably.


25.Ba6! was surprisingly strong:

Position after: 25.Ba6!

25...Rc7 [25...Rc3? 26.Bd2+–; 25...Rd8 26.Bb5 Rc8 27.Nb3 and the a5-pawn won’t survive for very
long.] 26.Bf4! Chasing the black rook away from the c-file. 26...Re7 27.Rc1 Kh7 28.Be3± Black is
paralyzed.

25...e5

140
Position after: 25...e5

26.Nb3

26.Nf5 Kh7= followed by ...Bc6.

26...Nd7 27.Rd1 Bc6 28.Rc1 Kf8 29.Ba6 Rc7 30.Bb7 Bb5+!

Position after: 30...Bb5+!

141
A nice move, forcing a draw in an otherwise slightly worse position.

31.axb5 Rxb7 32.Nc5 Nxc5 33.Bxc5+ Ke8 34.b6 Bxc5 35.Rxc5 f6 36.Rxa5 Rxb6

½-½
Veselin felt a little stupid that he hadn’t gone for 16...a5, but he had drawn anyway, and theoretically
couldn’t have got any more out of my new idea.
Anyway, despite its ups and downs, this was a rather boring game! Soon you will understand why it
was important to mention it.
In the third round, Veselin faced Rustam Kasimdzhanov with the white pieces. They entered a
complicated Meran and ‘Kasim’ played what is nowadays a well-known, surprising move that we
weren’t aware of at the time.
The game quickly headed towards a draw...
Topalov, Veselin (2752)
Kasimdzhanov, Rustam (2684)
London 2012

1.Nf3 d5 2.d4 Nf6 3.c4 c6 4.Nc3 e6 5.e3 Nbd7 6.Bd3 dxc4 7.Bxc4 b5 8.Bd3 Bb7 9.a3 Bd6 10.0-0 0-
0 11.Qc2 Rc8 12.b4

Position after: 12.b4

12...c5!N

When ‘Kasim’ played this move, which we didn’t know, à tempo, it somehow felt like a bad news!

142
13.bxc5 Bxf3

After this move, everything leads to a draw.

14.gxf3

After the game I found out that 14.cxd6 wasn’t that simple for Black – but it was too late!
Later, there were quite a few stunning victories for White: 14.cxd6 Nd5 15.gxf3 [15.Bb2 Qg5 16.g3
Qh5 17.Rfc1 N5b6∞]

Position after: 15.gxf3

A) 15...Nxc3 16.f4 Nf6 17.Qb2! [17.Bb2 is less convincing: 17...Na4 18.Qe2 a6 19.e4 Nxb2
20.Qxb2 Nh5 21.e5 Nxf4 22.Be4∞ but even there White won! 1-0 (29) Sasikiran, K (2680) – Najer,
E (2646) Bilbao 2014.] 17...Qd7! The best defence. (…...e5) [17...a6?! 18.Bd2 Na4 19.Qb1 Nc3
20.Qb2 Na4 21.Qb1 Nc3 22.Qb3 Nfe4? 23.Be1+– 1-0 (48) Ganguly, S (2619) – Zherebukh, Y
(2627) Fujairah 2012; 17...Qxd6 18.Bd2 Ncd5 19.Qxb5±] 18.Bd2 e5

143
Position after: 18...e5

19.f3! exd4 20.e4 a5 21.Rae1 Rc5 22.Bc1 Nh5 23.e5‚ Wojtaszek played all the best moves in this
game! Probably superb preparation. 1-0 (51) Wojtaszek, R (2734) – Shirov, A (2715) Skopje 2015.
B) 15...Qg5+ 16.Kh1 Qh5 17.Be2! [17.Bxh7+ Kh8 18.Be4 Nxc3 with the idea ...f5 is unclear.]
17...Nxc3 18.Rg1

Position after: 18.Rg1

18...Nf6 [According to my post-analysis, 18...Rfd8! was the best defence, and has been tried in

144
correspondence games: 19.e4 e5 20.Be3 Nxe2 21.Qxe2 exd4 22.Rg5 Qh6 23.Bf4 Qf6 24.Rf5 d3 and
after many only moves Black seems to be OK.] 19.Bb2! Nce4 [19...Na4 20.Qd2 Rfd8 21.e4 Rxd6
22.e5 1-0 Sethuraman, S (2653) – Shirov, A (2682) Edmonton 2016; 19...Nxe2 20.Qxe2±] 20.fxe4
Rxc2 21.Bxh5 Rxb2 22.Bf3 Rxf2 23.Rgf1± 1-0 (37) Bacrot, E (2711) – Ter Sahakyan, S (2580)
Jerusalem 2015.

14...Nxc5 15.dxc5 Rxc5

Position after: 15...Rxc5

Now any normal continuation ends in a draw. All the games played in this line, and there are quite a
few, ended that way.

16.f4

16.Bb2 Bxh2+! 17.Kxh2 Rh5+ 18.Kg2 Rg5+ 19.Kh3 Rh5+ 20.Kg2 Rg5+= ½-½ (22) Malakhatko, V
(2558) – Khismatullin, D (2638) St Petersburg 2012.

16...Nd5 17.Bb2 Nxc3 18.Bxc3 Qc7 19.Rfc1 Rc8

145
Position after: 19...Rc8

20.Bxh7+

20.Bb4 Rxc2 21.Rxc2 Qxc2 22.Bxc2 Bxb4 23.axb4 Rxc2 24.Rxa7 g5 25.Rb7 gxf4 26.exf4 Rc4
27.Rxb5 Rxf4 28.Kg2 Kg7 All this has been played twice, by two different players (GMs Iturrizaga
and Antoniewski), against Alexei Shirov. Two draws, obviously!

20...Kh8 21.Bd3 Rxc3 22.Qxc3 Qxc3 23.Rxc3 Rxc3 24.Bxb5 Bxa3

146
Position after: 24...Bxa3

The rest of the game isn’t too interesting.

25.Kg2 g6 26.Rd1 Rc7 27.Rd7 Rxd7 28.Bxd7 Kg7 29.e4 Kf6 30.Kf3 a5 31.e5+ Ke7 32.Ba4 Bc5
33.h3 Bb6 34.Bb5 Bc5 35.Ba4 Bb6 36.Bb5 Bc5 37.Ba4

½-½
In round four Veselin played Nakamura with the black pieces. The American player surprised us with
an Alapin opening, and the game was a boring draw.
In round 5, Veselin faced Peter Leko with the white pieces. The game was a fighting draw, where
Topalov sacrificed an exchange and Leko handled the compensation relatively easily, making it enough
to draw, but not to win. We can’t look at every game!
In the sixth round, Veselin had White again, this time against the best Cuban player Leinier
Dominguez. It looked like the game would be a draw, but after an inaccuracy by Black, Veselin broke
through his opponent’s fortress in a spectacular way. Let’s have a look.
Topalov, Veselin (2752)
Dominguez, Leinier Perez (2725)
London 2012

White to move

Black is about to play ...Bf6 and hold the position. White has two options, and Topalov went for the
one involving a piece sacrifice.

29.Bxe7!?

147
This is all about practical decisions: Topalov judged this move to be a draw, but felt that Black’s
chances of making a bad decision were greater.
29.Qa7 Bf6 [29...Qe8 30.Nd7 Qd8 31.Nb6 Bf6 is a much worse version for Black: 32.Bxf6 exf6
33.f5! gxf5 34.Qd7! Qxd7 35.Nxd7 Kg7 36.Kg3 with an easy win.] 30.Bxf6 exf6 31.Nd7 Kg7
32.Nb6 Qd8 and Black is slightly worse, but holding the position.

29.Bxe7!? Qe8 30.Qc7!

30.Qd8 Qxd8 31.Bxd8 Bxe5 would obviously lead to a draw.

30...f6 31.Qd7 Bf7

Winning a piece – but everything was under control!


31...Qxd7 32.Nxd7 Kf7 33.Bd8 Ke6 34.Nb6 h4 was probably holding for Black relatively easily.

32.Bxf6! Bxf6 33.Qxc6

Position after: 33.Qxc6

Queens are going to be swapped, but Black has to make a very critical decision: to exchange two minor
pieces or not.

33...Bxe5??

Played with little time left on the clock, this natural move loses the game. It may look like the black
bishop is able to handle the white pawns, but that’s an illusion!

148
33...Qxc6! was the right choice: 34.Nxc6 Bc4 and the black king runs towards the centre. Black isn’t
better and the position is a draw.

34.Qxe8+ Bxe8 35.fxe5 h4

Stopping the invasion, but now White can push his d-pawn and later come to the other side with his
king.
Black probably planned 35...Bc6, but after 36.Kg3 he is not able to hold the position:

Position after: 36.Kg3

36...Kf7 [36...g5 37.h4+–] 37.Kf4 Ke6 38.Kg5 Be8 39.g3 and Black is in zugzwang: he has to let the
white passed pawns advance if he isn’t willing to give away his own pawns!

36.d5 Kf7 37.Kg1 Ke7 38.Kf2 g5 39.Ke1 Bf7 40.e6 Bh5 41.Kd2

149
Position after: 41.Kd2

41...Kd8

Luckily for White, Black’s counterplay is too slow: 41...g4 42.hxg4 Bxg4 43.Kc3! Bf3 44.Kd4! Bxg2
45.Ke5

Position after: 45.Ke5

45...h3 46.d6+ Ke8 47.d7+ Ke7 48.c6 h2 49.c7 h1=Q 50.d8=Q#.

150
42.d6 g4 43.hxg4 Bxg4 44.e7+ Ke8 45.c6 Be6 46.Kc3 Bd5 47.c7 Bb7 48.Kd4 Kd7 49.Kc5

The white king goes to b8, and Black won’t be able to stop all the pawns.

1-0

A few days later, I was again impressed by another fantastic ‘break’, not by Veselin, but by Hikaru
Nakamura.
Let’s have a look at what I consider a very instructive example of a very important strategic topic.
Nakamura, Hikaru (2783)
Giri, Anish (2730)
London 2012

White to move

47.g5! hxg5 48.h6! gxh6 49.Rxe5! fxe5 50.f6

White has sacrificed an exchange and two pawns just to unblock the position – not something you see
every day! And he’s winning.

50...Bd7 51.f7+ Ke7 52.Bxd7

151
Position after: 52.Bxd7

52...Kxd7

52...Kxf7 53.Bf5 is an easy win as well, due to the weakness of the b7-pawn.

53.Bxc5 h5 54.f8=Q Rxf8+ 55.Bxf8 h4 56.Bh6 g4 57.Bg5 h3 58.Bh4 Kd6 59.Bg3 Ke6 60.Ke2 Kd6
61.Kd2

Position after: 61.Kd2

152
61...Kc5

Sitting and waiting wouldn’t have helped: 61...Ke6 62.Kc2 and the white king goes to c5, e.g. 62...Kd6
[62...Kf5? 63.c5 Ke6 64.c6+–] 63.Kb3 Ke6 64.Kb4 Kd6 [64...e4 65.dxe4 d3 66.Kc3+–] 65.Kb5 Ke6
66.Kc5 e4 67.Kxd4+–.

62.Bxe5 Kb4 63.Kc2 Kxa5 64.Kb3 Kb6 65.Bxd4+

1-0

In the following game Veselin faced Wang Hao with the black pieces. We went for a Grünfeld Defence
and the Chinese player opted for the tricky 4.Bg5 line. Vesko sacrificed half of his pawns in this game!
Let’s have a look.
Wang, Hao (2742)
Topalov, Veselin (2752)
London 2012

1.d4 Nf6 2.c4 g6 3.Nc3 d5 4.Bg5 Bg7 5.Bxf6 Bxf6 6.cxd5 c5

Position after: 6...c5

A double pawn sacrifice! When Topalov played it, there had only been five games – and never at top
level. Needless to say he fell in love with it! Since then, hundreds of games have been played. Let’s
look a little into the possible lines...

7.dxc5

153
A) 7.dxc6 Bxd4 8.cxb7 Bxb7©
B) 7.Nf3 cxd4 8.Nxd4 Qb6 9.Nb3

Position after: 9.Nb3

9...0-0 [9...a5!? 10.e3 a4 11.Bb5+ Bd7 12.Nd4 Bxd4 13.Qxd4 Qxd4 14.exd4 Bxb5 15.Nxb5 Ra5
16.Rc1 0-0 is also close to a draw: ½-½ (27) Bluebaum, M (2605) – Swiercz, D (2665) Moscow 2016]
10.e3 [10.Qd2 Rd8 11.Rd1 Na6 12.e4 Bd7 13.Bd3 Nb4 14.Bb1 Bb5 15.Nxb5 Qxb5© ½-½ (38)
Wojtaszek, R (2745) – Vidit, S (2693) Linares 2017] 10...Rd8 11.Be2 e6 and the position will become
very dry soon, for example 12.Rc1 Bxc3+ 13.Rxc3 exd5 14.0-0 Nc6 15.Rc5 a6 16.Qc1 [16.Rxd5
Be6©] 16...Be6 17.Nd4 Nxd4 18.exd4 Rac8= ½-½ (36) Mamedyarov, S (2754) – Vachier-Lagrave, M
(2762) Shamkir 2015.

7...Nd7

154
Position after: 7...Nd7

8.Qa4

A) 8.c6 isn’t dangerous: 8...Nc5! 9.Rc1 Qb6 10.Nf3 0-0 11.e3 Qxb2 12.Nd4 bxc6 13.Be2 cxd5
14.Nxd5 Bxd4 15.Nxe7+ Kg7 16.Qxd4+ Qxd4 17.exd4 Re8© ½-½ (18) Vallejo Pons, F (2710) –
Nepomniachtchi, I (2751) Moscow 2017.
B) 8.e3 0-0! [A few years later, Veselin went wrong in a blitz game against Magnus: 8...Nxc5?!
9.Bb5+! isn’t fun for Black: 9...Kf8 10.Nge2 Qb6 11.a4 Qa5 12.0-0 Kg7 13.Nd4 Rd8 14.Rb1± 1-0
(36) Carlsen, M (2855) – Topalov, V (2761) Leuven 2016] 9.Bc4 [9.Nf3 Nxc5 10.Nd4 Qa5 11.Bc4
Qb4 12.Bb3 Bf5 13.Nxf5 Bxc3+ 14.bxc3 Qxc3+ 15.Ke2 gxf5 16.Rc1 was a game Mamedyarov –
Giri, and here 16...Qa5! would have equalized: 17.Qd4 Rac8 18.g4 Nxb3 19.axb3 Rxc1 20.Rxc1
Rd8=] 9...Nxc5 10.Nge2 Qa5

155
Position after: 10...Qa5

B1) 11.Qd2 b5! This is why good players tend to prefer putting their queen on a5 and not b6 in this
exact line! 12.Nxb5 Qxd2+ 13.Kxd2 Rb8 14.Rhb1 Bf5 15.Nbd4 Bxb1 16.Rxb1 Ne4+ 17.Ke1

Position after: 17.Ke1

And now there are two games with Tomashevsky as White: 17...Rb6 [17...Rfc8!? 18.Nc6 Rb6 19.f3
Nd6 20.Bd3 e6 21.e4 exd5 22.exd5 Rxb2 23.Rxb2 Bxb2 24.Ne7+ Kf8 25.Nxc8 Nxc8 was very
close to a draw in Tomashevsky, E (2758) – Vachier-Lagrave, M (2744) Baku 2015, ½-½ (54).]

156
18.b4 Rc8 19.Nc6 e6 20.Rc1 and here I believe 20...Nd6! was right, with an unclear game.
[20...Rbxc6? 21.dxc6 Rxc6 22.b5 Rc5 23.f3 Nd6 24.Bd3± ½-½ (101) Tomashevsky, E (2749) –
Dominguez, L (2734) Khanty-Mansiysk 2015]
B2) 11.0-0 Qb4 12.Bb3 Bf5

Position after: 12...Bf5

Black gets sufficient compensation, for example 13.Bc2 [13.Nd4 Bd3 14.Re1 Rfd8 15.Qd2 a5 16.a3
Qb6 17.Ba2 Rac8 18.Rac1 Ba6 19.Rb1 Bd3 20.Rbc1 Ba6 and the players repeated moves in Svidler,
P (2745) – Nepomniachtchi, I (2740) Moscow 2016.] 13...Qxb2 [13...Bg4!? is also possible: 14.Rb1
Bxe2 15.Nxe2 Bxb2 16.Nf4 Qa3 17.Nd3 Nxd3 18.Bxd3 Rab8= ½-½ (29) Vitiugov, N (2725) –
Grischuk, A (2774) Skopje 2015] 14.Bxf5 Bxc3 15.Rb1 Qd2= ½-½ (21) Kuzubov, Y (2635) –
Inarkiev, E (2730) Turkey 2016.

8...Qc7!?N

Better than the previously played 8...0-0 which allows 9.Qa3! as played by Moiseenko.

9.Ne4?!

9.Rc1 gave birth to a very famous game, due to a brilliant (missed!) tactic: 9...Qxc5 10.e4 0-0 11.Nf3

157
Position after: 11.Nf3

11...Qb6 [11...Ne5 looks more precise to me, when I think Black has sufficient compensation after
12.Nxe5 Bxe5 13.Be2 Qd6 14.h3 a6∞.] 12.Qa3 Nc5 13.e5 Bg7 14.Nd1 Nd7 15.Qxe7 Nxe5 16.Nxe5
Bf6

Position after: 16...Bf6

This position was in my book Chess Calculation Training! 17.Nd7 [Missing the superb 17.Qxf8+!
Kxf8 18.Rxc8+! winning material, although after 18...Kg7 19.Rxa8 Qa5+! Black would keep some

158
practical chances.] 17...Bxe7 18.Nxb6 axb6© ½-½ (30) Nguyen, D (2503) – Negi, P (2651) Manila
2013.

9...Bxb2 10.Rb1 Bg7

Position after: 10...Bg7

Black’s position is already a little more pleasant.

11.Nf3

11.c6 bxc6 12.Qxc6 Qxc6 13.dxc6 isn’t problematic for Black, for instance 13...Nb6 14.e3 Bf5 15.f3
0-0-0©

11...0-0 12.Qa3 Ne5 13.Ned2?

The engine thinks it was better to put the other knight here, and suggests, for example, the following
drawing line: 13.Nfd2 Bf5 14.e3 Rfd8

159
Position after: 14...Rfd8

15.d6 exd6 16.cxd6 Qc6 17.Rc1 Qb6 18.Rb1 Qc6=.

13...Nxf3+ 14.gxf3 Rd8 15.e4 Bd4 16.Rc1 Qe5 17.Nc4 Qf6 18.Rc2 Bd7 19.Be2 e6 20.Ne3

Position after: 20.Ne3

20...exd5

160
20...Bxe3 was strong as well: 21.Qxe3 [21.fxe3 Qh4+ 22.Kd2 exd5 23.exd5 Ba4!–+] 21...exd5 22.e5
[22.exd5? Re8–+] 22...Qe7 23.Qd4 Rac8µ.

21.Nxd5 Qh4 22.Bc4 Be6?!

Giving away a huge part of the advantage.


The exchange sac 22...Rac8! was a killer: 23.c6 [23.Bb3 Be6–+ is a huge improvement on the game.]
23...Bxc6 24.Ne7+ Kg7 25.Nxc8 Bxe4! Wow! Now White has to find a couple of only moves not to
resign immediately.

Position after: 25...Bxe4!

26.Rd2™ [26.fxe4 Qxe4+–+] 26...Qf4! 27.Be2™ 27...Qxd2+ 28.Kxd2 Bc5+ 29.Bd3 Bxa3 30.fxe4
Rxc8µ.

23.Rd2 Be5 24.Ke2 b5!?

161
Position after: 24...b5!?

After sacrificing the d5- and c5-pawns early on, why not another one on b5?

25.cxb6?!

A) White needed to find a very difficult series of moves in order to hold the balance: 25.Bb3! a5
26.c6! (…Ne7+) 26...Rd6!? 27.Rhd1! a4

Position after: 27...a4

162
28.Nb6! Rxd2+ 29.Rxd2 b4! 30.Qxb4 axb3 31.Nxa8 bxa2 32.Rxa2 Bxa2 33.c7 Be6 34.Qb8+ Kg7
35.c8=Q=
B) 25.Bxb5? Bxd5 was Black’s dream, opening two dangerous files, the ‘b’ and the ‘e’: 26.Rxd5
[26.exd5 Bd4µ] 26...Rxd5 27.exd5 Rb8µ.

25...axb6 26.Qb3 b5!?

Position after: 26...b5!?

And a fourth one! As the first gift on this square was declined, Topalov offers another one. The aim is
just to open lots of files for the black rooks. The bishop pair and the white king position ensure full
compensation.

27.Qxb5

Challenge accepted!

27...Kg7 28.Qb3 Rab8 29.Qe3 h5 30.Bb3 Bxd5 31.Bxd5 Qf6 32.Rc2 Rdc8 33.Rxc8 Rxc8 34.Rd1 Bf4
35.Qd4 Rc2+ 36.Kf1 Be5 37.Qe3 Qh4

163
Position after: 37...Qh4

The previous ten moves by both players were correct, but here Wang Hao committed a mistake.

38.f4?

38.Rd2!³ was necessary.

38...Qxh2! 39.Qf3 Rc3!

Perhaps the move White overlooked or underestimated.


39...Bxf4 40.Bxf7! would be fine for White, 40...Kxf7?? being met by 41.Qb3++–.

40.Rd3 Rc1+ 41.Rd1 Rxd1+ 42.Qxd1 Qxf4?

42...Qh3+! was much stronger:

164
Position after: 42...Qh3+!

43.Ke2 [43.Ke1? Bc3+ 44.Ke2 Qg4+ 45.f3 Qxf4 is even worse for White, as he was forced to weaken
his dark squares even more with f2-f3.] 43...Qg4+ 44.Kd2 [44.f3? Qxf4–+] 44...Qxf4+ 45.Ke2 h4
46.Qd3 Keeping the pawn on f2 was crucial for White: now he is able to hold the position a bit thanks
to the free third rank. 46...Qg4+ 47.Qf3 Qd7 48.Kf1™

Position after: 48.Kf1™

The king is needed to defend against the h-pawn. 48...h3 49.Kg1 Qc8 [49...h2+ 50.Kh1 Qe7 51.Bb3

165
Qg5 52.Qd1 Qf4 is also close to winning.] 50.Bb3 Qc1+ 51.Bd1 Here it’s a question of whether Black
can break through White’s defence, but the position definitely looks like a nightmare for White, even
after finding all these only moves!

43.Qd3

Position after: 43.Qd3

Now White holds, despite still being slightly worse.

43...h4 44.Bb3 g5 45.Bd1! g4 46.Qd7 Qxe4

46...g3 47.Qg4+=

47.Qxg4+ Qxg4 48.Bxg4 ½-½

The next game wasn’t very interesting. Veselin had the white pieces against one of my favourite
players, Shakhriyar Mamedyarov. In an open Caro-Kann, we overestimated one idea for Black and the
position was equal, although there was still quite a fight. Once again, we can’t examine every game!
Game 9 was versus Ivanchuk with the black pieces. Vesko showed great technique to win a very
slightly better ending. Here it is.
Ivanchuk, Vassily (2769)
Topalov, Veselin (2752)
London 2012

166
White to move

On move 40, Ivanchuk played what seemed to be a natural move – but is actually losing!

40.Kd3??

40.exf5+ gxf5 41.Be1 was a tougher defence.

40.Kd3?? f4!

Black is winning already! The h4-pawn has to be defended, while Black will wait for the right moment
to push ...g6-g5 and create another passed pawn (the other being the ‘a’ one).

41.Be1

41.gxf4 exf4 42.Bd2 just loses a pawn: 42...Ne5+ 43.Ke2 Nxc4 44.Bxf4 Bxh4 should be a relatively
easy win for Black.

41...fxg3 42.Bxg3 Nc5+!

167
Position after: 42...Nc5+!

An instructive move. The white bishop won’t be able to hold both sides of the board alone. Or, to put it
another way, it won’t be able to stop the two black passed pawns on each side of the board!

43.Nxc5+ Bxc5 44.Be1

44.f4 looked like a natural try, but it still loses: 44...exf4 45.Bxf4 Bf2 46.Bg5 a5 47.Kc3 Ke5

Position after: 47...Ke5

168
48.Bd8 [48.Kd3 a4 49.Be7 Ke6 50.Bg5 a3 51.Kc3 Ke5 52.Kb3 Bc5–+] 48...Kxe4 The white king
won’t be able to stay on c3 for very long. Once Black collects the c-pawn, he will take the h-one.
49.Be7 [49.Kb3 Kd3 50.Be7 a4+–+] 49...Bg3 50.Bg5 [50.Kb3 Kd4 51.c5 Kd5 52.Ka4 Bf2–+]
50...Be5+ 51.Kb3 Kd4–+

44...a6!?

44...Be7 was also fine, but I assume Veselin wanted to avoid the following complications: 45.Bf2 a5
46.c5 g5 47.hxg5 Bxg5 48.Kc4 h4 49.c6

Position after: 49.c6

49...a4! [49...h3? 50.Bg3 Bf4 is only a draw: 51.Kc5!! Bxg3 52.Kb6 h2 53.c7 h1=Q 54.c8=Q+ Kf6
55.Qf5+=] 50.Kb4 h3 51.Bg3 a3 52.Kxa3 Bf4–+.

45.Ke2 Be7

169
Position after: 45...Be7

46.f4

A desperate try.
A) 46.Ke3 g5 47.hxg5 Bxg5+ 48.Ke2 Bd8! and Black will push both of his passed pawns with an
easy win. The white king can choose one side to defend, but then Black will win the game on the
other side of the board.
B) Now 46.Bf2 is far too slow: 46...a5 47.c5 a4 48.Kd3 g5 49.hxg5 Bxg5 50.Kc4 h4–+.

46...exf4 47.Kf3 Bd6 48.Ba5 Kd7

Black needs his bishop on c7 in order to push his a-pawn.

49.Bd2 Bc7 50.Bc3 Ke6 51.Bh8?!

170
Position after: 51.Bh8?!

Accelerating the process.

51...Be5! 52.Bxe5 Kxe5

52...Kxe5 53.c5 a5–+

0-1

171
172
Discussing the game with Vassily Ivanchuk (photos Fred Lucas)
In game ten Vesko faced Michael Adams with the white pieces. We tried to avoid forcing lines, going
for the sideline 1.d4 Nf6 2.c4 e6 3.Nf3 d5 4.Bg5. It worked decently, as Vesko got a slightly
favourable ending. However the top English player defended well and the game was drawn.
The eleventh and last round was against Anish Giri with the black pieces. In order to share first place
and earn lots of Grand Prix points, Vesko had to win this game. We considered many aggressive
openings, until we agreed that, given the situation, the biggest surprise would be to play... the Queen’s
Gambit!

A crucial game: Anish Giri vs Veselin Topalov (photo Anastazia Karlovich)


Giri, Anish (2730)
Topalov, Veselin (2752)
London 2012

1.d4 Nf6 2.c4 e6 3.Nf3 d5 4.Nc3 Be7

173
Position after: 4...Be7

We toyed with a sharper opening for a long time, as Veselin needed a win to share first in the
tournament, but ended up deciding it was easier to repeat his normal repertoire and wait to see how
aggressively Anish would play, as a draw wasn’t a good result for him either.

5.Bf4

Anish was in this period when he liked to do something.

5...0-0 6.e3 Nbd7 7.Be2 dxc4 8.0-0 c5 9.dxc5 Nxc5 10.Bxc4 Qxd1 11.Rfxd1 b6 12.Nd4 Bb7
13.Rac1 a6 14.b4 Nce4 15.Nxe4 Bxe4 16.a3

174
Position after: 16.a3

Does this position remind you of something? (See Gelfand – Topalov, p.87).

16...a5!N

Finally!

17.f3 Bd5 18.e4

18.Bxd5 Nxd5 19.Nc6 Bf6 20.Bd6 Rfc8 is fine for Black.

175
Position after: 20...Rfc8

When summarizing the ideas of this variation to Veselin, I told him that the position was equal, but to
be careful and after 21.bxa5 bxa5 22.Kf2 play a move like 22...h5=. [And not 22...Nc3?? 23.Rxc3
Bxc3 24.Ne7+ Kh8 25.Nxc8 Rxc8 26.Rc1 winning for White.]

18...Bxc4 19.Rxc4 axb4 20.axb4

Position after: 20.axb4

20...Rfc8 21.Rdc1

All this was prepared. Black solves his problems here, as 21.Nc6 allows 21...Bxb4 22.Rxb4 Rxc6
23.Be3=.

21...Rxc4 22.Rxc4 h6

22...Kf8 was also fine.

23.Kf1 Kf8 24.Nc6 b5 25.Rd4 Ra6 26.Nxe7 Kxe7 27.Rd2 Nd7 28.Rc2 g5 29.Bc7 f6 30.Ba5 h5

176
Position after: 30...h5

31.h4?!

What we were hoping for happened: Anish plays for a win!


A) 31.Rc8 Ne5 32.Rb8 Rc6 33.Rxb5 Rc1+ 34.Kf2 Rc2+ is a draw, as 35.Kg3? h4+ 36.Kh3 Nd3
would lead to disaster for White.
B) 31.Rc7 Kd6 32.Rb7 Kc6 33.Rc7+ Kd6= was one natural way to finish the game.

31...gxh4 32.Rc8?!

32.Rc7! was still a draw: 32...Kd6 [32...Rd6 33.Rb7 Rd1+ 34.Kf2 Rd2+ 35.Kg1 h3 36.gxh3 Kd6
37.Rxb5 Ne5©] 33.Rb7 Kc6 34.Rc7+ Kd6=.

32...Ne5 33.Rc7+ Kd6

177
Position after: 33...Kd6

34.Rb7?!

Now White is definitely worse.


34.Rh7! was required, but even here, after 34...Nc4! [34...h3?! 35.Bc7+ Kc6 36.Bxe5=] 35.Bc7+
[35.Rxh5 Nxa5 36.bxa5 Rxa5 37.Rxh4 Kc5 is likely to go wrong for White.] 35...Kc6 36.Bd8 Ra2
White is suffering.

34...h3 35.gxh3 Nxf3 36.Rxb5 Nd2+

36...Ng5!? was another good option.

37.Kg2

178
Position after: 37.Kg2

37...Ra8!

Black needs his rook in the ‘attack’, if you can call it an attack with so few pieces left! Instead,
37...Nxe4 38.Rxh5 Ra8 39.Rh7 is an improved version of the game for White.

38.Rxh5

38.e5+ fxe5 39.Kf2 was perhaps a tougher defence. Unlike in the game, White is a pawn down, but he
should be able to draw since the two central black pawns are doubled.

38...Rg8+ 39.Kf2 Nxe4+

179
Position after: 39...Nxe4+

40.Ke3?!

A natural move. It didn’t seem important where to move the king, but it was!
40.Kf3! was better, with the idea 40...f5 41.h4 Kd5 42.Bc7 Nf6 43.Rh6! and it isn’t easy for Black to
make progress. Of course, with the white king on e3, this move is impossible due to ...Ng4+. Computer
stuff!

40...f5

Now Black has a huge – probably winning – advantage.

41.Rh7

41.h4 may have been best: 41...Kd5 42.Bc7 Nf6 43.Rg5 Rh8µ.

41...e5

The two pawns are just too strong!

42.Bc7+

42.h4 Rg3+ 43.Ke2 f4–+

42...Ke6 43.Kd3

180
Position after: 43.Kd3

Trying to activate the king, but the game is over.

43...Rg3+ 44.Kc4 Rc3+ 45.Kb5 f4 46.Rh6+ Kd5 47.Bb6 f3 48.Rh8 Ra3 49.Bg1 f2 50.Bxf2 Nxf2
51.Rd8+ Ke4 52.Kc6 Rc3+

0-1

The novelty 16...a5 was only supposed to bring half a point, but sometimes life smiles at you!
A truly amazing finish, with a slight touch of luck, enabling Vesko to share first in London. Actually,
he even won the tournament according to the tiebreaks, but it did not matter very much as the Grand
Prix points were shared between people with the same tournament points.
The final ranking was as follows:

181
The points for the top three being 170-140-110, to be shared between players on equal points, meant
that Veselin, Gelfand and Mamedyarov shared the early lead in the Grand Prix series with 140 points
each. The adventure had begun very well, and the road to the Candidates tournament was open to
Veselin.

182
Winner in London: Veselin Topalov (photo Anastazia Karlovich)

183
Chapter 5
Zug: More Success!

Technnical meeting in Zug: “I have a question!”.


(photo Anastazia Karlovich)

The tournament in the Swiss town of Zug was the third in


the Grand Prix series. The second had taken place in
Tashkent with Morozevich, Karjakin and Wang Hao sharing
first place.
Mixed fortunes for the two players who shared first with
Veselin in London: Mamedyarov had a decent result (eighty
points) and Gelfand a bad one (thirty points). Therefore
Mamedyarov had become the biggest contender to win the
Grand Prix series, but several other players would join the
party. Exactly who was still unclear!
The shared first place in London had been a good result, but
the road to the Candidates was still very long, so Veselin’s
winning spirit was undiminished.
Nevertheless, he started Zug with a rather boring draw
against Anish Giri with the black pieces. Sadly for us, Anish
didn’t make a ‘present’ this time as he had done in London
by pushing too hard! The game didn’t provide much to look
at.
In the second round, Vesko was up against Peter Leko with White. He won a fascinating game, where
all three results were possible throughout! Let’s have a look.
Topalov, Veselin (2771)
Leko, Peter (2744)
Zug 2013

1.d4 Nf6 2.c4 e6 3.Nf3 d5 4.Nc3 Bb4 5.cxd5 exd5 6.Bg5 h6 7.Bh4 c5 8.e3 c4

184
Position after: 8...c4

9.Nd2

Changing from his game with Aronian (see p.43) where he played 9.Be2.

9...g5

A solid option is 9...Be6.

10.Bg3 Bf5 11.Be5

11.h4!? is definitely another try.

11...Bxc3 12.bxc3 Nbd7 13.Bd6!?

185
Position after: 13.Bd6!?

Other moves aren’t problematic for Black.

13...Qb6 14.Bb4?!

A strange decision, allowing Black to win an important tempo with his next move.
14.Ba3 looked more natural. I assume Veselin’s idea in provoking ...a7-a5 was to weaken Black’s
queenside in case he decided to castle long.

14...a5

In case of 14...0-0-0?! White would play 15.a4 or 15.Qa4, and his last move would be totally justified.

15.Ba3 Qc6!

186
Position after: 15...Qc6!

Black delays castling in order to push ...b5.


Giving priority to the queenside attack is the correct plan; any other would let White remove his bishop
from a3 and push a2-a4.

16.Be2 b5 17.0-0 Rb8

Black is simply too fast on the queenside. Veselin is always happy if complications arise, but in this
case, I believe he underestimated Black’s position.

18.f3

White needs to react in the centre, otherwise he is clearly worse.

18...b4 19.Bb2

187
Position after: 19.Bb2

A really interesting position. The stakes are clear: Black has a huge advantage on the queenside, but
White is going to break in the centre with e3-e4. Black’s dominance will remain; for White it’s make or
break!

19...0-0

Instead of castling, Black could have continued in the same direction: 19...Nb6! 20.e4 [20.Rc1 is too
slow: 20...Na4 21.Ba1 bxc3 22.Bxc3 Nxc3 23.Rxc3 0-0 24.e4 Bg6 followed by ...Qb6, and Black is
doing great.] 20...Na4 21.Qc1 Be6³ Although White will soon create a mess on the kingside, I believe
he is slightly worse.

20.Rc1 Nb6!

Solidifying the central pawn chain.


The only existing game continued with the worse 20...Rfe8 which was nevertheless unclear: 21.e4!
dxe4

188
Position after: 21...dxe4

22.cxb4 [22.fxe4 Bxe4 23.Nxe4 Nxe4 24.cxb4 Rxb4 25.Ba3?! Ra4µ; 22.Nxc4?! bxc3 23.Bxc3 exf3
24.Rxf3 Bg4³] 22...exf3 23.Rxf3 Rxe2 24.Qxe2 Bg4∞ 0-1 (60) Ly, M (2460) – Ni Hua (2689) Sydney
2015.

21.Ba1

Position after: 21.Ba1

189
21...Qe6

Natural, but stronger was 21...Na4 22.cxb4 [22.e4 Nxc3 23.Bxc3 bxc3 24.Rxc3 Bg6³ transposes to
19...Nb6 20.Rc1.] 22...axb4 23.e4 Be6! 24.exd5 Nxd5 25.Rxc4 Qa8‚ followed by ...Nac3, with huge
compensation.

22.e4

Not only was it time for this, it had become forced anyway.

22...dxe4

Not a bad move, but keeping the tension with 22...Bg6! may have been stronger. Now if 23.e5 Nh5
24.g3 f6 25.f4 Ng7 Black is slightly better.

23.cxb4

Around here, Peter Leko was already in time trouble, and Veselin did all he could to take advantage of
it.

23...axb4 24.fxe4

Position after: 24.fxe4

24...Nxe4?!

Not the best. Two other moves made sense:


A) 24...Bxe4! 25.Nxc4 [25.Rxf6 Qxf6 26.Nxe4 Qf4 27.Nf2 Ra8µ] 25...Bd5ƒ

190
B) 24...c3 25.Rxf5 [25.exf5? Qe3+] 25...cxd2 26.Rc2 Nxe4 [26...Nbd7!?] 27.Re5 Qg6 28.Bd3 Nf2
29.Kxf2 Qxd3 30.Rxd2 Qg6³

25.Bxc4 Nd5!?

The natural 25...Nxc4 led to an acceptable position as well:

Position after: 25...Nxc4

26.Nxc4 Nc3 27.Bxc3 bxc3 28.d5 Qe4! 29.Rxc3 Rb1 30.Qd2 [30.Qxb1?? Qd4+–+] 30...Rxf1+
31.Kxf1 Rb8©

26.Nb3

26.Re1 Rbc8 27.Bb3 Rxc1 28.Qxc1 Bg6 29.h4 Rc8 30.Qd1 Kg7∞

26...Bg6 27.Nc5 Qd6 28.Qf3

191
Position after: 28.Qf3

28...Rfe8

Bblundering an exchange, but Black will have sufficient compensation.


28...Ndc3 led to a forced draw: 29.Nxe4 Qxd4+ 30.Kh1 Bxe4 31.Bxf7+ Kg7 32.Qg3 Rxf7 33.Rxf7+
Kxf7 34.Qxb8 Bxg2+ 35.Kxg2 Qd2+=.

29.Na6 Rb7 30.Bxd5 Qxd5 31.Nc7 Rxc7 32.Rxc7 Qxa2

192
Position after: 32...Qxa2

What a strange game! After all the weird things that have happened, the position is still about equal.
Top level chess!

33.Qh3!?

The engine suggests 33.d5 to open the a1-h8 diagonal, but it was probably too drawish for Veselin’s
taste: 33...Qxd5 and if White creates a mating threat on the a1-h8 diagonal at any time, Black will play
...Nc3 and draw the game due to the limited amount of material.

33...Qd5!?

There was nothing wrong with 33...Kh7 either.

34.Qxh6 Nc3

Position after: 34...Nc3

35.Kh1?

A huge blunder which could have lost the game, but fortunately didn’t!
35.Bxc3 bxc3 36.Kh1 Qf5 37.Rg1 c2 with equality.

35...Qf5! 36.Rg1 Ne4! 37.Rc2™ 37...b3

37...Nf2+? is a blunder, although after 38.Rxf2 Qxf2 39.d5 Black should be able to draw with

193
39...Qxg1+ 40.Kxg1 Re1+ 41.Kf2 Rxa1.

38.d5

In case of 38.Rb2?, 38...Nf2+ would work perfectly: 39.Rxf2 Qxf2 40.d5 b2–+.

38...f6 39.Re2

Position after: 39.Re2

And here it seems that Black doesn’t have a win, much as it might look like it.

39...Bf7?

A) 39...Kf7! was best. 40.d6™ 40...Ng3+ 41.hxg3 Rxe2 42.d7!

194
Position after: 42.d7!

How lucky! [42.Rc1? b2 43.Rc7+ Ke6–+] 42...Rd2 [42...Qxd7? 43.Bxf6!‚] 43.Kh2 Rxd7µ
B) 39...Re7 is a great try, but White holds. 40.Rc1™ [40.Rge1?? Nf2+ 41.Kg1 Rxe2 42.Rxe2 Ng4–+]
40...Rc7 [40...Rh7? 41.Rc8+! Qxc8 42.Qxg6++–] 41.Rce1 Rh7 42.Qxh7+ Bxh7 43.Rxe4=
C) 39...Ng3+?? is, however, a huge blunder: 40.hxg3 Rxe2 41.g4!+–.

40.d6

195
Position after: 40.d6

40...Qg6?

A) 40...Bd5! was required. Difficult on move 40! 41.d7 [41.Rge1 Ng3+! 42.hxg3 Rxe2 43.Rxe2 Qf1+
44.Kh2 Qxe2 45.Qh3 Be6 46.Qh6 Bd5=] 41...Rd8 42.Rc1 Bb7 43.Qh5 Rxd7 44.Qe8+ Kg7=
B) 40...Ng3+?? would still have been a blunder: 41.hxg3 Rxe2 42.Bxf6 Qh7 43.Qxh7+ Kxh7
44.d7+–.

41.Qxg6+ Bxg6 42.d7 Rd8 43.Rd1

Position after: 43.Rd1

43...Kf7

Threatening ...Bh5.
43...Bh5?? 44.Rxe4 Bxd1 45.Re8++–

44.g4!?

44.Rd5 preventing ...Nc5 was good as well, while 44...Ke6 45.Rd4 wins material: 45...Rxd7 [45...f5
46.Re3 Rxd7 47.Rxd7 Kxd7 48.Rxb3+–] 46.Rdxe4+ Bxe4 47.Rxe4+ Kf7 48.Re1+–.

44...Nc5

44...f5 doesn’t seem to be a solution: 45.Kg1!? fxg4 46.Re3 Ke7 47.Red3 Nc5 48.Re1+ Ne4 [48...Kf7
49.Rd6+–] 49.Rd5 Rxd7 50.Rxg5 Bh7 51.Rxg4 Ke6 52.Rgxe4+ Bxe4 53.Rxe4+ Kf5 54.Re5+ Kf4

196
55.Re1+–.

45.Rd6 Ne4 46.Rd4 Nc5 47.Re3 Bc2 48.Rc3

Position after: 48.Rc3

48...Nxd7

48...Ne6 loses without much of a fight: 49.Rc8 Nxd4 [49...Ke7 50.Rxd8 Nxd8 51.Kg2+–] 50.Rxd8
Ke7 51.Rf8! Ne6 52.d8=Q+ Nxd8 53.Rxf6+–.

49.Re3 f5?

49...Kg6! was the toughest defence: 50.Bb2! [50.Re7? Ra8µ] 50...Ra8 51.Rxd7 Ra2 52.Bd4 b2 53.Bxb2
Rxb2 54.Re6 Rb4 55.h3 And I am not sure that White is winning here.

50.Bb2! fxg4

197
Position after: 50...fxg4

51.Rd5!

Basic but instructive prophylaxis, preventing ...Bc2-f5! The game is over.


A) 51.Rxg4? Nc5=
B) 51.Kg2? Bf5„

51...Kg6

51...Bg6 52.Ba3+–

52.Re7 Ra8 53.Rdxd7 Ra2 54.Rd6+ Kh5

198
Position after: 54...Kh5

55.Bg7

55.Ree6 was even more effective: 55...g3 56.h3+–.

55...g3 56.h3 Bg6 57.Ree6 b2 58.Bxb2 1-0

The third round was against Gata Kamsky with the black pieces. The American went for a Bb5+
Sicilian. Vesko played his beloved 3...Nd7 and obtained a very decent position, where at the very least
Black was not worse.
In the middlegame he outplayed Kamsky and obtained a great advantage thanks to an exchange
sacrifice...
Kamsky, Gata (2741)
Topalov, Veselin (2771)
Zug 2013

1.e4 c5 2.Nf3 d6 3.Bb5+ Nd7 4.c3 Ngf6 5.Qe2 a6 6.Ba4

199
Position after: 6.Ba4

6...b5

6...Qc7 was Veselin’s choice against Magnus in Wijk aan Zee, remember! This time he decided to go
for a similar plan, but with the queen staying on d8.

7.Bc2 e5 8.0-0 Be7 9.d4 cxd4 10.cxd4 0-0 11.Nc3 Bb7 12.a3 Rc8 13.Rd1 exd4 14.Rxd4 Re8 15.Bg5
Qc7 16.Re1 Nf8 17.Qd1 Ne6 18.Rd2 Nxg5 19.Nxg5 Qc5 20.Nf3 Bf8 21.h4 h6 22.Rd4 Qa7 23.Bb3
Rc5 24.Qd2 Qa8 25.Qf4 Qc8 26.Red1 a5

200
Position after: 26...a5

After some inspired moves by Kamsky (14.Rxd4, 21.h4) we have reached a position that is about equal.
It was time for White to try to go for a dry position, but that isn’t Kamsky’s style. Here he decided to
allow an exchange sac that Veselin was surely dreaming of.

27.Nh2?!

Any normal move (27.Nd5, 27.Re1, 27.Ba2 etc.), would be about equal.

27...Rxc3!

No need to be asked twice! Especially now that the white knight is way out of the centre, it is definitely
time for this sac.

28.bxc3 Rxe4 29.Rxe4 Bxe4 30.Qd2 d5 31.a4 Bc5!

Position after: 31...Bc5!

An excellent move, with the deadly threat of ...Ng4.

32.Kf1?

A blunder in an already bad position.


A) 32.axb5? Ng4! just wins.
B) 32.Qf4 was perhaps best, although after 32...bxa4 33.Bxa4 Bb6 Black is much better. The c3-pawn
is weak, the white pieces are slightly paralyzed, and Black’s bishop pair is hideous for White.

201
32...bxa4 33.Bxa4 Qc7 34.g3

Position after: 34.g3

Here Black missed two wins. One of them is just a cleverer way to achieve what Vesko tried to get with
the game move. The other, to my taste, is simply outstanding!

34...Qc8?

A) 34...Qe5 is the first one. Compared to the game move, Black does not allow f2-f3, and will access
the h3-square through f5 (still preventing f2-f3) instead of c8. 35.Qb2 [35.f3 Qxg3–+; 35.Qe2 Bg2+!
36.Ke1 Qxc3+ 37.Qd2 Qe5+ 38.Qe2 Bb4+–+] 35...Qf5 [35...Bf5!? threatening ...Bh3+, but also
...Qa4 (attacking the a4-bishop), also wins.] 36.Ke1 Bg2–+ followed by ...Ne4 or ...Qh3.
B) I fell in love with the move 34...Bh1!! Black has two threats: the obvious one ...Ne4, and the less
obvious ...Qc8, after which f2-f3 will no longer close the diagonal of the black light-squared bishop.
35.Kg1 [35.Bc2 Qc8! 36.Ke1 and now that the white king has run from the safety zone, Black has
many good moves, for instance 36...Be4 37.Ba4 Qh3 38.Nf1 Bg2 39.Qe2 Ne4–+] 35...Qxg3+!
36.Kxh1 Qxh4

202
Position after: 36...Qxh4

It is important that the a4-bishop is hanging! 37.f4 [37.Bc2 Ng4–+] 37...Ne4 38.Qxd5 Nf2+ 39.Kg1
[39.Kg2 Qh3+ 40.Kg1 Ne4+–+] 39...Qg3+ 40.Kf1 Nxd1 41.Qxc5 Ne3+! 42.Ke2 Qxh2+–+. Of
course the e3-knight is untouchable due to ...Qg1+.

35.f3

Position after: 35.f3

203
Now White can breathe.

35...Bf5?!

35...Qh3+ was the last try to create serious problems: 36.Qg2 Qf5 37.Qe2 Nh5!

Position after: 37...Nh5!

38.Ke1™ [38.g4? Ng3+ 39.Kg2 Nxe2 40.gxf5 Bxf5 and the c3-pawn is undefendable, while
41.Rxd5?? is met by 41...Nf4+–+.] 38...Nxg3 [38...Qe5 39.fxe4 Qxc3+ isn’t as strong: 40.Rd2 Nxg3
41.Qd3 Qc1+ 42.Bd1 Nxe4 43.Qc2 Qxd2+ 44.Qxd2 Nxd2 45.Kxd2 and White should survive.]
39.fxe4 Qe5 40.Rxd5 Qxc3+ 41.Qd2 Bf2+!

204
Position after: 41...Bf2+!

42.Kd1 [42.Kxf2? Nxe4+–+] 42...Qa1+ 43.Kc2 Qxa4+ 44.Kb2 Bb6 [44...Nxe4 45.Rd8+™ 45...Kh7
46.Qd5™ and somehow, Black is only slightly better.] 45.Nf3 Qxe4 46.Qd3 Qf4 White has found
many only moves – and is still definitely not having any fun.

36.g4

Position after: 36.g4

205
Black remains slightly better, but White survives.

36...Be6

A) 36...Bb6! was the best try. White gets an unpleasant position whatever move he plays: 37.gxf5
[37.Bb3 a4 38.Ba2 Be6³; 37.Kg2 Qc4 38.Ra1 Bg6 39.h5 Bd3³] 37...Qc4+ 38.Qd3 Qxa4 39.Rb1 Nd7
40.Ng4 h5 41.Nf2 Qxh4 42.Rb2 Bxf2 43.Rxf2 Nf6³.
B) 36...Bxg4 37.fxg4 Nxg4 38.Nxg4 Qxg4 39.Bb3 is a draw.

37.Kg2 Bd6 38.Qd4 Qc7 39.Nf1 Be5

39...Bc5 was the last aggressive try, with the funny idea 40.Qd2 Qc8! (back!). White’s position isn’t
too pleasant due to the possible sacrifices on g4, but he can just play a move like 41.Nh2, when Black’s
position is a little better, but nothing dramatic.

40.Qd3

Position after: 40.Qd3

40...Qxc3

Basically agreeing to a draw.


40...Bxc3 41.Rc1 d4 42.Qxd4 Nd5!?

206
Position after: 42...Nd5!?

was a good try, but if White finds the forced 43.Qe4, it doesn’t seem that Black can win. For instance
43...Qb8 44.Rd1 Nf4+ 45.Kh1 Nh3 46.Kg2 Nf4+ with a draw.

41.Qxc3 Bxc3 42.Ng3 Kf8 43.g5 hxg5 44.hxg5 Ne8 45.Ne2 Bb4 46.Nf4

Position after: 46.Nf4

White will collect one of the two pawns which Black has for the exchange, but the position is so

207
simplified that there is no question about whether it’s a draw or not.

46...Nd6 47.Kh1

47.Nxd5? was a nice last trick: 47...Nc4! and Black is better again due to ...Ne3/...Nb2 problems.

47...Nc4 48.Bc6 Bd2 49.Bxd5 Bxf4 50.Bxe6 fxe6 51.Rd4 Bxg5 52.Rxc4 Ke7 53.f4 Bf6 54.Rc5 a4
55.Ra5 a3 56.Rxa3 g5 57.fxg5 Bxg5

½-½
The next round saw Vesko take on Rustam Kasimdzhanov with the black pieces. He decided to go for a
side-line of the King’s Indian and was unpleasantly surprised by an unknown piece sacrifice. A
fascinating game, as the following analysis shows.
Kasimdzhanov, Rustam (2709)
Topalov, Veselin (2771)
Zug 2013

1.d4 Nf6 2.c4 g6 3.Nc3 Bg7 4.e4 d6 5.Nf3 0-0 6.Be2 e5 7.0-0 exd4 8.Nxd4 Re8 9.f3 c6 10.Kh1
Nh5?!

Black wants ...Qh4 and ...Be5. Of course, given what happened next move, playing 10...Nbd7 would
have been better, when White has just a slight edge.

11.g4! Nf6 12.Bf4 h5?!

Position after: 12...h5?!

208
Nowadays it is official: this whole line is way too risky for Black, not to say totally refuted.

13.Nf5!N

We have started to get used to shocking bad news when facing Kasim.
After the 12...c5! novelty in London, this time he surprises Veselin with a powerful piece sacrifice. At
the time when Vesko had analyzed this position (many years before this game!), engines were not too
strong. Nowadays, a good one immediately shows +1 (an almost winning advantage) for White.
13.g5 Nh7 14.Qd2 Qe7 was what Veselin was hoping for.

13...gxf5

Sadly, you can’t decline a gift from a friend, especially if the only other option is to jump from a
bridge.

14.gxf5 d5!

The best practical chance. Others are just too easy for White, with Rg1 coming and the d6-pawn falling.
A funny computer line goes 14...Nbd7 15.Rg1 Kh8 16.Bxd6 Nh7 17.c5 b5 18.a4 b4

Position after: 18...b4

and now the engine humiliates us by giving away its second knight: 19.Nd5! cxd5 20.Qxd5 with a
huge advantage for White. I gave up trying to find a solution other than 14...d5!.

15.cxd5 cxd5

209
Position after: 15...cxd5

16.e5?

Objectively a bad move, but in practice a good one; you will soon see how difficult it was for Black to
defend against it.
16.Rg1! was a monster! The lines are quite nice:

Position after: 16.Rg1!

210
A) 16...Kh8 17.Qe1 dxe4 Now White has a plethora of winning moves, for example 18.Rd1 followed
by either Qh4, Qg3, or even Rxg7 (followed by Qg3+ and Bxb8) is winning. [18.Qg3!? Rg8 19.Qh3
…Nxe4 is also powerful, as is 18.Qh4!? or 18.fxe4!?]
B) 16...Nc6 17.Qf1! followed by Qg2-Qg5 or Qh3-Ne4, or simply Rd1, is crushing (above +3 for
White).
C) 16...dxe4 17.Nxe4! [17.Bh6?? Ng4µ] 17...Qxd1 [17...Nxe4 18.fxe4 Qxd1 19.Raxd1 Kh8 20.f6!
Bxf6 21.Rd5+–] 18.Nxf6+ Kh8 19.Raxd1 Bxf6 20.Rd6 Nd7 [20...Bg7 21.f6 Bf8 22.Rd5+–] 21.Bb5
Rg8™

Position after: 21...Rg8™

22.Re1! [22.Rgd1 Bg5 23.Bxg5 Rxg5 24.Bxd7 Bxd7 25.Rxd7 Rxf5 isn’t so clear.] 22...Bg5 23.Bxg5
Rxg5 24.Re8+! Rg8 25.Bxd7 Rxe8 26.Bxe8 Bxf5 27.Bxf7 and White should win this endgame.

16...Bxf5

16...Nfd7 17.f6 would definitely be bad for Black.

17.exf6 Qxf6 18.Nxd5 Qxb2 19.Ne7+!

211
Position after: 19.Ne7+!

The point! Otherwise Black would be better.

19...Rxe7 20.Qd8+ Kh7?

20...Bf8!! would have been an amazing save: 21.Rg1+ [21.Bc4? Nd7! 22.Qxa8

Position after: 22.Qxa8

22...Re4!! Double attack on the white bishops! 23.Rg1+ Kh7 and Black is fine!] 21...Bg4! [21...Bg6?

212
22.Bc4!+–] 22.fxg4 [22.Rxg4+ hxg4 23.Rg1 Qxe2 24.Rxg4+ Kh7 25.Rh4+ Kg6 26.Rg4+=; 22.Bc4
Qf6!? 23.fxg4 h4„] 22...Nd7!!

Position after: 22...Nd7!!

23.gxh5+ Kh8 24.Qxa8 Rxe2 [24...Qxe2?? 25.Qxb7+–] 25.Rae1 [25.Qd8? Qd4³] 25...Rxe1 26.Rxe1
Qf2 with a draw to come: 27.Bg3 Qf3+ 28.Kg1 Kh7 29.Bf2 Qg4+=.

21.Qxe7 Bg6

Position after: 21...Bg6

213
22.Rad1?

22.Qe8! was critical: 22...Nd7!? [22...Qb4 23.Bg3 Qf8 24.Qxf8 Bxf8 25.Rad1 Na6 26.Rd7 Nc5
27.Rc7 h4 28.Bf2 b6 29.Bxh4+–] 23.Qxa8 Qxe2 24.Rae1 Qxa2 25.Qxb7 Nc5 26.Qe7 Nd3 27.Re2
Qc4 28.Bd2

Position after: 28.Bd2

and Black isn’t without chances, even if the future isn’t too bright. A sample line would be 28...Ne5
29.Rff2 Nd3 30.Rg2 a5 31.Bxa5 Nf4 32.Qb4 Qxb4 33.Bxb4 Nxe2 34.Rxe2 and it is difficult to say
whether Black can hold or not.

22...Nc6 23.Qe3 Qxa2

Now Black can breathe.

24.Bd3 Rd8 25.Bxg6+ fxg6 26.Rxd8 Nxd8

214
Position after: 26...Nxd8

27.Be5?!

After this move Black is basically not in danger anymore.


White should have tried 27.Qe4 to play Rc1-c7 or Rd1-d7 and keep some pressure. Of course, Black
should be able to draw without too many problems anyway.

27...Ne6 28.Ra1 Qd5 29.Bxg7 Nxg7 30.Qe4

30.Rxa7 Nf5=

30...Qd7 31.Rxa7 Qd1+ 32.Kg2 Qd2+ 33.Kh3 Qd7+ 34.Kg2 Qd2+ 35.Kg3 Qd6+ 36.f4 Kh6

215
Position after: 36...Kh6

With such a weak white king and a knight coming to f5, Black is completely out of danger.

37.Rxb7 Nf5+ 38.Kg2 Qd2+ 39.Kg1 Ne3 40.Rh7+ Kxh7 41.Qe7+ Kg8 42.Qe8+ Kg7 43.Qe7+ Kg8
½-½

216
The next game was a smooth victory against Hikaru Nakamura. The game was purely technical and we
won’t be spending too much time on it. Still, I think a couple of Vesko’s manoeuvres are worth a look.
Topalov, Veselin (2771)
Nakamura, Hikaru (2767)
Zug 2013

1.e4 e5 2.Nf3 Nc6 3.Bb5 a6 4.Ba4 Nf6 5.0-0 Be7 6.d3 d6 7.c3 Bg4 8.Re1 Nd7 9.h3 Bh5 10.Be3 b5
11.Bc2 Nb6 12.Nbd2 0-0 13.Nf1 Rb8 14.Ng3 Bxf3 15.Qxf3 Bg5 16.Nf5 Bxe3 17.Nxe3 Ne7 18.d4
c5 19.dxe5 dxe5

Position after: 19...dxe5

White has got a great position out of the opening, and Veselin didn’t give his American opponent any
chance of counterplay.

20.h4!

Preventing all ...Ng6 ideas, as they would be answered by h4-h5. It is difficult for Black to know what
to do with his e7-knight.

20...Qd6 21.Red1 Qe6 22.a4

Opening the a-file.

22...Rfd8

22...bxa4 isn’t any good: 23.Bxa4 [23.Qe2!?] 23...Nxa4 24.Rxa4 Rxb2 25.Nc4!± followed by Rd6.

217
23.axb5 axb5 24.Rxd8+

24.Ra6!? right away also made sense.

24...Rxd8 25.Ra6!

Position after: 25.Ra6!

Paralyzing Black.

25...Qc6

A) In case of 25...Rd2, 26.b4! would be best [26.Nd5!? makes sense too, but better first to get rid of
the problems with the b2-pawn.] 26...cxb4 [26...c4 27.Nd5 Nexd5 28.exd5±] 27.cxb4 Rd6 28.Qe2±.
B) 25...c4 26.Nd5! was also highly unpleasant for Black.

26.c4! b4

Allowing White a sweet bishop manoeuvre.

27.Ba4!

27.Nd5 was strong as well: 27...Nexd5 28.exd5 Qb7 29.Ra1 Ra8 30.Rd1±. Of course the c4-pawn isn’t
hanging because of Qd3/e4.

27...Qb7 28.Bb5

218
Position after: 28.Bb5

28...Nbc8

28...Na8 to play ...Nc7 and get rid of White’s annoying pieces would be answered by the brilliant
29.Nf5! Nxf5 30.exf5 Qxf3 31.gxf3, and despite his tripled pawns, White is much better: Black’s
pieces are very passive and Rc6 is coming next.

29.Ng4 Nd6

29...Qc7, trying to hang on to the e5-pawn, would be strongly met by 30.h5! and if 30...h6 then
31.Nxh6+! gxh6 32.Rxh6 is crushing.

30.Nxe5

30.h5!? also made sense, with the idea of avoiding an immediate exchange of queens, since
30...Qxe4?? would be met by the nasty 31.Rxd6! Qe1+ 32.Kh2 Rxd6 33.Qa8++–.

30...Qxe4 31.Qxe4 Nxe4 32.f3 Ng3

219
Position after: 32...Ng3

33.Kf2?

Likely Topalov’s only mistake in the game!


33.Ra7! was much stronger. 33...f6 [33...Nef5 34.Bd7±] 34.Rd7! [34.Rxe7 fxe5 35.Rxe5 Rd2 isn’t so
clear.] 34...Ra8 [34...Rxd7 35.Nxd7 and the c5-pawn falls.] 35.Rxe7 fxe5 36.Kf2! The cleanest move.
36...Nf5 [36...Nh1+ 37.Ke3 Ra2 38.b3+–] 37.Rxe5 Nxh4 38.Rxc5 Ra2 39.Kg3 Ng6 40.b3+–.

33...Ngf5?

33...Nef5! would have given Black some chances: 34.Nd7 [34.Bd7 f6„] 34...Nh1+! 35.Kg1 [35.Ke1
Nxh4∞] 35...Nhg3

220
Position after: 35...Nhg3

and here the only chance for an advantage would have been to find 36.Bc6! [36.h5 g6„; 36.Rc6?!
Re8‚; 36.Kh2 h5∞] 36...h5 [36...Nxh4 37.Nxc5±] 37.Ra8! Now the only question is whether the
ending is winning or not. Objectively, it seems it isn’t if Black defends perfectly. 37...Rxa8 38.Bxa8
Nd6 39.Bd5 Ne2+ 40.Kf1 [40.Kf2?! Nc3!= with an extra threat of ...Nd1+.] 40...Nf4 41.Nxc5 Nf5
42.Kf2 And now the long engine line goes:

Position after: 42.Kf2

221
42...Nxd5 43.cxd5 Nxh4 44.Nd3 Kf8 45.Nxb4 Ke7 46.Nc2 Kd6 47.Ne3 g5! 48.Nf1 Kxd5 49.Ng3 f5
50.Nxh5 f4 51.Nf6+ Kc4 52.Ne4 Kb3 53.Nxg5 Kxb2 54.Ne6 Ng6 55.Ke2 Kc3

Position after: 55...Kc3

Just in time to make a draw!

34.h5 f6 35.Nd7 Rc8 36.Ba4

Position after: 36.Ba4

222
The c5-pawn is lost, this time without counterplay for Black.

36...Nd4

36...Kf7 37.Ra5+–

37.h6!

37.Ra5 Ne6 is less precise.

37...f5 38.Ke3 b3

38...gxh6 39.Nxc5 Nxf3 40.Nd3 Ng5 41.c5 should be an easy win.

39.hxg7 Kxg7 40.Kd3 Nc2 41.Kc3

Position after: 41.Kc3

All the black pawns are falling.

41...f4 42.Ra7 Kg6 43.Bxb3 Ne3 44.Nxc5 N7f5 45.Ra6+ Kh5 46.Ne6 Kh4 47.Nxf4 Kg3 48.Nd3
Kxg2 49.c5

223
Position after: 49.c5

49...Kxf3

Pushing the h-pawn right away doesn’t help: 49...h5 50.Be6 Rd8 51.Bxf5 Nxf5 52.Nf4+ Kxf3
53.Nxh5+–.

50.Be6 Rd8 51.Bxf5 Nxf5 52.Rf6 Ke4 53.Re6+ Kf3 54.b4 h5 55.Ne5+ Kf4 56.Nf7 Rf8 57.Nd6

Position after: 57.Nd6

224
Of course! These rook endings are winning, as even if Black manages to get the white rook for the h-
pawn, it will take so much time that the white pawns will be close to queening (if they haven’t queened
already).

57...Nxd6 58.Rxd6 Rh8 59.b5 h4 60.b6 h3 61.b7 h2 62.Rd1 Ke4 63.Kc2 Rg8 64.c6 Rg2+

64...Rg1 65.b8=Q h1=Q 66.Qb4+ Kf5 67.Qc5++–

65.Kc3 Rg3+ 66.Kc4 Rg1 67.b8=Q h1=Q 68.Rd4+

Position after: 68.Rd4+

It’s mate.

68...Kf5 69.Qf4+ Kg6 70.Rd6+ Kh7 71.Qf5+ Kh8 72.Qf8+

72.Qf8+ Rg8 73.Rh6+ Qxh6 74.Qxh6#

1-0

A very important point for the tournament!


In the next game, Vesko faced Teimour Radjabov with the black pieces. They played a long, very
theoretical Nimzo-Indian line which is a forced draw. Vesko had it all prepared somewhere in his files,
as Radjabov probably had as well. However, the game lasted several hours, as neither of them could
instantly remember it all!
Radjabov, Teimour (2793)
Topalov, Veselin (2771)

225
Zug 2013

1.d4 Nf6 2.c4 e6 3.Nc3 Bb4 4.Qc2 d5 5.cxd5 exd5 6.Bg5 c5 7.dxc5 h6 8.Bh4 g5 9.Bg3 Ne4 10.e3
Qa5 11.Nge2 Bf5 12.Be5 0-0 13.Nd4 Re8

Position after: 13...Re8

All this is big mainline theory.

14.Nxf5

14.Bxb8 is the other main move, known to lead to a draw if both sides play perfectly, for example
14...Nxc3 [14...Bg6!?] 15.Nxf5 Ne4+ 16.Kd1 Raxb8 17.f3 Rbc8!

226
Position after: 17...Rbc8!

18.fxe4 dxe4 19.Bc4 Qxc5 20.Bxf7+ Kxf7 21.Qxc5 Bxc5 22.Rc1 Red8+ ½-½ Laznicka, V (2703) –
Leko, P (2720) Porto Carras 2011.

14...Rxe5 15.Nxh6+ Kg7 16.Ng4 Re7

Position after: 16...Re7

17.0-0-0!?N

227
Radjabov’s novelty for the game, meaning everything up to now was known. Topalov had it prepared
in his home files, but to remember his lines until the end certainly took a while!
17.f3 had been tried, and after 17...Nxc3 18.Kf2 d4!?N 19.exd4 Nd5 the position would be quite
unclear.

17...Bxc3!

17...Nxc3? 18.bxc3 Bxc3 19.Rxd5 is just bad for Black.

18.f3 Qxa2! 19.fxe4 Qa1+ 20.Qb1 Bxb2+ 21.Kc2 Qa4+!

Position after: 21...Qa4+!

Drawing by force, otherwise Black would be worse.

22.Kxb2 Rxe4 23.Rd4! Rxd4 24.exd4 Qxd4+ 25.Ka3 Qxg4 26.Qxb7 Qd4 27.Qxa8 Qc3+

228
Position after: 27...Qc3+

With perpetual check.

28.Ka2

28.Ka4 Qc2+= doesn’t change anything.

28...Qc2+ 29.Ka3 Qc3+ 30.Ka2 Qc2+ 31.Ka3 Qc3+

½-½
Afterwards, the usual accusers came to the conclusion that the game had been pre-arranged, which was
of course total rubbish, as there would have been no point in the players showing such long preparation
if the game had been of no sporting interest. That was the beginning of couple of annoying accusations
which I will detail very soon in the book.
In the next game, Vesko had the white pieces against Ruslan Ponomariov. He misplayed the opening
and was worse. Eventually, solid play enabled him to save a draw, after his Ukrainian opponent had
pushed for more than seventy moves!
The next game was against Alexander Morozevich with the black pieces. ‘Moro’ played a very
interesting novelty on move nine, but an unpleasant surprise awaited him. Let’s see the game!
Morozevich, Alexander (2758)
Topalov, Veselin (2771)
Zug 2013

1.Nf3 c5 2.c4 Nf6 3.Nc3 Nc6 4.d4 cxd4 5.Nxd4 e6 6.g3 Qb6 7.Nb3 Ne5 8.e4 Bb4

229
Position after: 8...Bb4

9.c5!?

A novelty at the time. Morozevich was probably expecting to surprise Topalov with it, but there was
one small problem: Veselin and I had had the same idea for White a few weeks earlier, so he knew
exactly how to react.

9...Qc6!

We considered this move best, although we didn’t think White was necessarily better if Black accepts
the pawn sacrifice: 9...Bxc5 10.Nxc5 Qxc5 11.Be3 [11.Be2 Nc6 12.0-0 0-0 13.Be3 Qe7 14.Qa4 d5=]
11...Qb4 12.Rc1

230
Position after: 12.Rc1

12...0-0 [12...Nxe4!? 13.Bg2 d5 14.0-0 Nxc3 15.Rxc3 Nd7 16.Qc2 0-0 17.Rc1 h6 and White has
enough compensation for the two pawns, but probably only enough to draw.] 13.a3 [13.Bg2 b6„]
13...Qxb2! Leading to crazy lines. 14.f4 Nfg4 15.Bd2 Qb6 16.Qe2 Nc4!? [16...Nd3+!? 17.Qxd3 Nf2
18.Qf3 Nxh1 19.Qxh1 d5∞] 17.Qxg4 Nxd2 18.Kxd2 d5 and Black enjoys very interesting
compensation.

Position after: 9...Qc6!

231
10.f3

We also considered 10.Bg2, but after 10...Qb5 White should repeat moves. [However I’m pretty sure
Veselin would have opted for our second choice, 10...b6!?, leading to very sharp positions. For
example, one of our lines went 11.Bf4 d6! 12.cxd6 Bxc3+ 13.bxc3 Qxc3+ 14.Kf1 Nd3

Position after: 14...Nd3

15.e5 Nd5 16.Bd2 Qxe5 17.Qc2 Nc5 18.Bf4 Qh5 19.Nxc5 bxc5 20.Re1 Nxf4 21.Bxa8 Ba6+ 22.Kg1
Nh3+ 23.Kg2 Nf4+=.]

10...b6!

We didn’t like Black’s position after 10...Nxe4?! 11.fxe4 Qxe4+ 12.Qe2 Qxh1 [12...Nd3+? 13.Kd2
Qxh1 14.Kxd3 b6 15.Bg2! Ba6+ 16.Kc2±] 13.Qxe5 0-0 14.Bd2ƒ.

11.Bf4 Ng6 12.Bd6 bxc5 13.a3 Bxc3+ 14.bxc3

232
Position after: 14.bxc3

14...a5!

Just following our main line!


14...Nxe4? 15.fxe4 Qxe4+ 16.Kf2 Qxh1 17.Bg2 is bad for Black: the a8-rook is trapped.

15.e5

A) 15.Nxc5 is nicely met by 15...Nd5!ƒ.


B) 15.a4? now allows 15...Nxe4! as after 16.fxe4 Qxe4+ 17.Kf2 Qxh1 18.Bg2 Qxd1 19.Rxd1 (the
a8-rook is no longer trapped – obviously the idea behind 15...a7-a5!) 19...Ra6–+.

15...Nd5

One of the two choices we had – and the easier to remember.


15...Ne4!? was the other one:

233
Position after: 15...Ne4!?

16.fxe4 [16.Bb5 Qxb5 17.fxe4 Ba6‚] 16...Qxe4+ 17.Kf2 Qxh1 18.Nxc5 Bb7! 19.Nxb7 Qxb7 20.Rb1
and Black should be better, but the position is definitely not easy to play.

16.Bd3

Here we analyzed two other moves.


A) After 16.Qd2 f6 17.f4

234
Position after: 17.f4

Black has several ways to get a fine position, for example 17...a4 [17...fxe5 18.fxe5 a4 transposes;
17...Ndxf4!? 18.gxf4 Qxh1 19.0-0-0 Qd5 20.Qxd5 exd5 21.exf6 gxf6 22.Nxc5 Rg8∞] 18.Nxc5 fxe5
19.fxe5 Ra5 20.c4 Rxc5 21.Bxc5 Ndf4 22.gxf4 Qxc5 23.Be2 0-0 24.Rf1 Bb7©.
B) After 16.a4 Black has two funny ways to make a draw:

Position after: 16.a4

16...Nxc3 [16...Ba6 17.Nxa5 Qb6 18.Nc4 Qc6=; 16...Ne3?! 17.Bb5²] 17.Qd2 Ne4 18.fxe4 Qxe4+
19.Kf2 Bb7 20.Be2 Qg2+ [20...Qf5+ 21.Kg1 Qe4=] 21.Ke3 Qe4+=.

16...f6

235
Position after: 16...f6

Our analysis stopped here; we judged (correctly) that Black’s position was comfortable. Actually,
White has even no way to equalize.

17.Bxg6+ hxg6 18.Qd3 Kf7 19.Nd2

A) 19.c4 fxe5 20.cxd5 exd5!? 21.0-0-0 Bb7! 22.Bxe5 c4 23.Nd4 cxd3+ 24.Nxc6 dxc6 25.Rxd3 Ba6³/
µ
B) 19.Nxc5 Ba6 20.c4 Nb6 21.Nxa6 Rxa6 22.c5 Nc8³

19...c4 20.Qd4

20.Qxc4 Qxc4 21.Nxc4 Ba6 22.Nxa5 Nxc3µ

20...Qb6 21.Qxb6 Nxb6 22.Rb1 Nd5 23.Ne4

23.Nxc4 Ba6µ

23...g5 24.Kd2

The prophylactic 24.h3!?³ made sense.

24...Rh3!

236
Position after: 24...Rh3!

Black’s position is better and Topalov starts to turn things around, managing to create more and more
problems.

25.Nf2 Rh5 26.Rb5 a4 27.h3 Ba6 28.Rc5 Bb7 29.Rb1 Bc6 30.Rxc4

There was no reason to hang on to the c4-pawn. To do so it would have been necessary to keep a
passive bishop on a6. Better to have it on c6!

30...Ne7 31.g4 Rhh8 32.Ke3 Ra5 33.exf6 Nd5+ 34.Kd2 gxf6 35.f4™

Otherwise ...e5 and ...Nf4 were coming.

237
Position after: 35.f4™

35...gxf4 36.Bxf4 Rb5!?

36...e5!? 37.Bg3 Nc7 followed by ...Rd5+, ...Nb5, was strong as well.

37.Rxb5 Bxb5 38.Rd4 Bc6 39.Bg3 e5 40.Rc4 Rb8

Black’s centre is awesome and all his pieces are perfect compared to his opponent’s.

41.Nd3

238
Position after: 41.Nd3

41...Rb3

It might have been a good idea not to let White simplify the position: 41...Ke6 42.Nc5+ Ke7 43.Nd3
[43.Nxa4 Ra8 44.Nb2 Rxa3 followed by ...Ra2 is crushing] 43...Bb5! [43...Rb3?! 44.g5! would be a
worse version than the game, as White is taking on f6 with check!] 44.Rc5 Bxd3 45.Rxd5 [45.Kxd3
Ke6–+] 45...Bc4 46.Rc5 d5

Position after: 46...d5

239
and Black has powerful pieces and a powerful centre. Another important point is that the c5-rook needs
a lot of time to reach a decent square, so despite the opposite-coloured bishops, this should be an easy
win.

42.g5?

Trying 42.Bxe5! was a must.

Position after: 42.Bxe5!

In such a bad position, White should definitely take his chance to simplify the position in a
straightforward way, unless he sees that the chances of it working are hopeless.
A) 42...fxe5 doesn’t seem to win: 43.Nxe5+ Ke6 [43...Ke7 44.Nxc6+ dxc6 45.Rxc6 Rxa3 46.g5 and
for example 46...Ra2+ 47.Kd3 Rg2 48.Ra6 Rg3+ 49.Kc4 Rxg5 50.Kd4 Rh5 51.c4 with a draw.]
44.Nxc6 dxc6 45.Rxc6+ Ke5

240
Position after: 45...Ke5

Black activates his king, but I still don’t think it’s enough. 46.g5 Rxa3 [46...Rb2+ 47.Kd3 Rh2 48.Kc4
Rxh3 49.Kb5 Nxc3+ 50.Kb4=] 47.g6 Ra2+ 48.Kd3 Rh2 49.Kc4 Rxh3 50.g7 Rg3 51.Rc5 Rxc3+
52.Kb5 Rg3 [52...Rxc5+ 53.Kxc5 Ne7 54.Kb4=] 53.Kxa4=
B) 42...Rxa3! 43.Bd4 Ra2+ 44.Ke1 Bb5 [44...Rh2 45.Nf2 Rg2 46.Rc5 Nb6 47.c4 a3 48.Rxc6 dxc6
49.Bxb6 a2 50.Bd4 should also be a draw: White will sac his bishop for the a-pawn when the material
is too reduced.] 45.Rc5 Bxd3 46.Rxd5 Ke6

Position after: 46...Ke6

241
47.c4 This pawn must be given away so the white bishop can defend against the a-pawn. [47.Ra5 a3
looks winning: ...Bc4, ...Ra1, ...a2 is coming.] 47...Bxc4 48.Rc5 Bd5 49.Rc3 Rh2 and Black is clearly
better, but White has good drawing chances.

42...e4!

Position after: 42...e4!

Now Black wins.

43.Nf4

43.Rxe4 Nxc3 44.Re3 Nb1+!? [44...Ne4+ 45.Kc1 Nxg5 is also good] 45.Ke2 fxg5 with an easy win.

43...Nxf4 44.Bxf4 Rxa3 45.Rc5 fxg5 46.Bxg5 Ra2+ 47.Ke3 a3 48.Bf4 Ra1 49.Rf5+ Ke6 50.Re5+
Kf6 51.Ra5 d5 52.Kd4 a2 53.Bd2

242
Position after: 53.Bd2

53...Bb5!

A cute finish: getting the bishop from the passive c6-square to the optimal c4.

54.Kxd5

54.Rxb5 Rd1–+

54...Rd1 55.Kxe4

55.Rxa2 e3–+

55...a1=Q

0-1

This is how the ‘Morozevich Trilogy’ started, since Topalov beat him 3-0 in this Grand Prix series.
After the game, Alexander Morozevich said at the press conference that anti-cheating measures should
be reinforced. A strange coincidence, as there was already some ‘history’ between the players. To quote
The Guardian newspaper, “Alexander Morozevich believed there had been irregularities when Topalov
won the world title in San Luis, Argentina, in 2005”. He was right in any case: anti-cheating measures
are a plus, especially since they prevent people from becoming paranoid.
Nevertheless, from this moment on, some people started to launch accusations against Veselin on the
internet. Nobody would have cared if his accusers had been random, everyday internet trolls, but they
included several well-known Russian GMs whose opinions people listen to. Were they serious? Was it

243
brainwashing? Or revenge for the accusations against Kramnik in 2006? We’ll never know. But, in
order to add a little fun to this book, from now on we’ll label these people ‘the big brains’.
It was weird to read comments on the internet about ‘Vesko and his cheating team’, sometimes written
by people who shake my hand with a smile every time they see me! Well, they may be happy to learn
that his ‘preparation team’ was just me, and that we did work hard – but only in order to win fairly,
over the board! So far, I haven’t thanked any of these big brains for their kind words. As it would take
too long to do so individually, I thought the best thing would be to take this opportunity to thank them
all in my book.
What impressed me most was Topalov’s reaction to to all that. He simply said, “Look, when I became
World Champion the same people accused me; when I complained in Elista for much better reasons
they hammered me; these things happened during World Championships, when the outcome of the
tournaments had an importance that lasts a lifetime. Now I just ignore them, although it’s unpleasant to
read”. An impressive mentality! Actually, I even realized that the lack of such self-control was another
reason why I would never be a 2800 player myself.
The next game was a combative draw against Shakhriyar Mamedyarov. Vesko faced excellent
preparation in the Caro-Kann and the position was always around equal. It turned into a huge fight, but
was a draw anyway.
The tenth game was a big test for Vesko as regards winning the tournament, playing Caruana with the
black pieces. A draw would have been highly satisfactory, but Veselin continued in the same combative
spirit, playing his beloved Najdorf. The game was a huge mess, but in the end he managed to outplay
Fabiano.

244
Fabiano Caruana vs Veselin Topalov FIDE GP Zug (photo Anastazia Karlovich)
Caruana, Fabiano (2772)
Topalov, Veselin (2771)
Zug 2013

1.e4 c5 2.Nf3 d6 3.d4 cxd4 4.Nxd4 Nf6 5.Nc3 a6 6.Be3 e5 7.Nb3 Be6 8.f3 Be7 9.Qd2 0-0 10.0-0-0
a5!?

245
Position after: 10...a5!?

Black gives away the b5-square but threatens ...a5-a4, while a2-a4 will give away the b4-square.

11.a4!?

This is considered the best move.

11...Na6 12.Bb5?!

Not the critical move.


Later Veselin showed the way with the white pieces: 12.g4 Nb4 13.Kb1 Qc7 [13...Rc8 is also possible,
with minor differences.] 14.g5 Nh5 [14...Nd7 15.f4²]

246
Position after: 14...Nh5

A) 15.Rg1!? was played by Topalov’s former second, Ivan Cheparinov. I wonder if he had missed
15...d5!? [15...f5 16.g6 hxg6 17.Rxg6 Rf6 18.Rg2 f4 19.Bg1± 1-0 (50) Cheparinov, I (2678) –
Schandorff, L (2522) Helsingor 2013] 16.exd5 Bxd5 17.Nxd5 Nxd5 but after 18.Qxd5 Rfd8 19.Bb5
Rxd5 20.Rxd5© White has enough compensation for the queen; in fact, according to the engine, he is
even slightly better!
B) 15.Bb5 Rac8 16.Rhg1 f5 17.gxf6 Nxf6 18.Qg2 Nh5 19.Nc1! Bf6 20.Nd3 Nxd3 21.Rxd3 Nf4
22.Bxf4 exf4 23.Nd5 Bxd5 24.Rxd5± 1-0 (41) Topalov, V (2772) – Vachier-Lagrave, M (2768) Saint
Louis 2014. Black could definitely improve on the game somewhere, but that was the right path for
White.

12...Nc7 13.Bb6 Qc8

247
Position after: 13...Qc8

14.Bxc7

Not a piece you usually trade in the Sicilian, but, in this exact pawn structure, White should definitely
try to focus on the light squares, especially b5 and d5. Nevertheless, White has blown his opening a bit
and Black’s position is absolutely fine.

14...Qxc7 15.g4 Rfd8 16.g5 Nh5 17.Nd5 Bxd5 18.exd5 g6 19.Kb1 Rf8 20.Qe3 Qd8 21.h4

248
Position after: 21.h4

Black is slightly better and has two plans: opening the f-file and placing a rook on f4, or placing a
knight on f4 and playing ...h6 in order to make later use of his central majority. Topalov opted for the
second plan, and it worked very smoothly, but objectively I believe playing ...f6 was better.

21...Nf4

21...f6! 22.Rdg1 [22.gxf6 Rxf6 23.Rdg1 Rf4³] 22...fxg5 23.hxg5 Nf4 24.Nd2 Bxg5 25.Ne4 Be7
26.Rh6 Rf7 and White does not have sufficient compensation.

22.Qg1 h6

At first the engine doesn’t like this idea, underestimating the powerful pawn phalanx Black gets on
e5/f5/g5.

23.Nd2

23.gxh6 Kh7 [Even the brave 23...Bxh4!? makes sense.] 24.h5 g5 followed by ...f5 and the position is
much easier for Black. The assessment of the engine (+/=) is completely wrong to my eyes.

23...Kg7!?

Brave! Although objectively slightly dubious, it deserves a ‘!?’ mark, given how well it went in the
game. If Fabiano couldn’t react to it properly, who could?
23...h5 would be equal.

24.gxh6+ Kxh6

249
Position after: 24...Kxh6

Many players would reflexively have left the white pawn on h6, but White is never going to mate the
black king without a dark-squared bishop, so there is no reason not to take it!

25.Nc4

I believe 25.Ne4! was the most annoying: 25...Qc7 26.Ng3 (threatening h4-h5, while ...g5 would allow
Nf5+) 26...Rac8 [26...f5 27.Qe3! and White has a concrete problem with Ne2 coming; and if 27...Kg7
28.h5 g5 29.h6+ Kh7 30.Nh5 it is becoming too artificial for Black.] 27.c3 Qc5 28.Qh2 Kg7 29.h5
Rh8 30.Qd2 and Black can’t prevent a pawn exchange on g6 followed by Ne4. Only White can be
better.

25...Qc7 26.Qe3

Fabiano is definitely struggling to find a plan.


26.h5 g5 27.Ne3 f5 28.c3 would still be decent for White, although I would rather pick Black. The
short-term plan is to bring the bishop to b6.

26...Kh7 27.h5 g5

Despite what some idiots think, Vesko plays natural moves, not computer ones, but the engine has a
voracious appetite, and was ready to digest a full pawn with 27...gxh5.

28.h6

28.Qd2 f5 29.Ne3 was still about equal.

250
28...f5 29.Qd2 Rf6 30.Rdg1 Rg6 31.Ne3 Rf8

Position after: 31...Rf8

Now Black’s position is definitely preferable. It seems that everything has gone like a dream: both
rooks are perfectly placed and the bishop is soon coming to b6.

32.Nc4

32.Ng2 Nxg2 33.Qxg2 Bd8!³

32...Bd8! 33.Rd1 Rff6

251
Position after: 33...Rff6

34.Qf2?

34.Ne3 was necessary, not to let Black take the h6-pawn so easily.

34...Rxh6 35.Qa7 Rxh1 36.Rxh1+ Rh6 37.Qg1 Rh4!

Position after: 37...Rh4!

It has turned into a nightmare for White: he is a pawn down and Black’s majority on the kingside will
prove fatal in the endgame.

38.Ne3 Qf7 39.Ng2 Rxh1 40.Qxh1+ Qh5 41.Qxh5+ Nxh5 42.Ne3 Ng3 43.Kc1 Kg6 44.Kd1 g4!

252
Position after: 44...g4!

It’s time to push those pawns!

45.fxg4 f4 46.Nc4 Bc7 47.Bd7 Kg5 48.Ke1 f3 49.Ne3 Bb6

0-1

Topalov had won the tournament with a round to go, and was facing Sergey Karjakin with White in the
last round. In a complex Benoni pawn structure, Karjakin refused to repeat moves early on and
overplayed the position. Vesko took advantage and converted the game into another full point!
Here is the final ranking of the tournament.

253
254
255
Chapter 6
Tough Times in Thessaloniki

After the Zug Grand Prix, Vesko was in the sole lead in the Grand Prix series with 310 points. Second
was Mamedyarov with 240, but having played a tournament more. It was clear that Veselin only
needed one more decent result from the coming two tournaments to end up in the top two places in the
series.
The only top tournament Veselin played in during the Grand Prix series was Norway Chess, which took
place in May 2013. We travelled there together. Vesko’s tournament was pretty tough, with one loss –
to Vishy Anand – and eight draws. Not a complete disaster, but not a satisfactory result either. It is
always difficult to focus on a ‘regular’ tournament when you have the Candidates in mind. However,
our work there on the openings was a plus for the coming tournaments. Let’s switch straight away to
the next Grand Prix: Thessaloniki!
As I explained earlier, a good result there would ensure Vesko qualification to the Candidates
tournament. Going by usual Grand Prix Series final rankings, a positive score (at least 6/11) in
Thessaloniki would ensure a top two in the yearly standings.
Veselin started the tournament with three draws against Alexander Grischuk, Gata Kamsky and Ruslan
Ponomariov. In all three games, the evalua-
tions of the positions never fluctuated too much; he was never really worse or much better.
One thing I would like to mention about the first three rounds is the very high sporting class of two
players, Vassily Ivanchuk and Leinier Dominguez. Four years earlier, they had faced each other in
Linares. They were in time trouble, Ivanchuk was nervous, trying to play quickly and knocking pieces
over. “Nothing so terrible, besides it was absolutely clear that he wasn’t doing it on purpose”, Leinier
told me. Anyway, Ivanchuk got a completely winning position after move 40, but offered a draw as he
considered that he hadn’t behaved correctly during the time scramble. In Thessaloniki they were in
huge time trouble again, and Ivanchuk had dozens of ways to win the game. Instead, he blundered a
piece just before the time control. Dominguez didn’t take it immediately, and started to repeat moves,
however Ivanchuk lost on time on move 40. The Cuban player suggested that the game should be a
draw. To him, it was a fair result after what had happened four years previously, and he didn’t feel like
he should win such a game in this way. But the arbiter said it was too late once Ivanchuk’s flag had
fallen, and the Ukrainian player said, “Don’t worry, let’s leave it like that”. Not the sort of attitude you
meet every day. Let’s see the finish of this game:
Ivanchuk, Vassily (2755)
Dominguez, Leinier Perez (2723)
Thessaloniki 2013

256
White to move

31.Bc7

The easiest way to win was 31.Nxg7 Rxd6 32.Qe8+ Kxg7 33.Re7+ Kf6 34.Rf7#.

31.Bc7 Rc8 32.Qxc8?

Allowing some chances. Instead, 32.Nxg7 Rxc7 33.Ne8 was game over.

32...Qxe1+ 33.Kh2 Be5+ 34.Bxe5+ Qxe5+ 35.g3 Kh7 36.Qb7+ Kg8

257
Position after: 36...Kg8

37.f4??

Blundering the e8-knight.


37.Nc7 Nd7 38.Nxb5 was still a relatively easy win.

37...Qe2+ 38.Kg1 Qe1+ 39.Kg2 Qe2+

Position after: 39...Qe2+

258
Despite continuing to check the white king instead of taking the e8-knight, Black won here on time!

0-1

In the next game, Ivanchuk was playing Veselin. Clearly, the Ukrainian genius had one of those bad
days of the kind that might not have happened without that tough loss against Dominguez...
Topalov, Veselin (2793)
Ivanchuk, Vassily (2755)
Thessaloniki 2013

1.e4 c5 2.Nf3 Nc6 3.Bb5

Against Ivanchuk, guessing the opening is pretty impossible. Veselin decided over the board not to
enter the very main lines.

3...d6 4.0-0 Bd7 5.Re1 Nf6 6.c3 a6 7.Bf1 Bg4 8.h3

Position after: 8.h3

8...Bh5

Nowadays people tend to go for 8...Bxf3 9.Qxf3 g6, which is a solid set-up. If White doesn’t play
10.d3 Black may even try 10...Bh6.

9.d4 Bxf3 10.gxf3 cxd4 11.cxd4

259
Position after: 11.cxd4

11...d5?!

This move was known to be dubious, and despite what we might have thought when Ivanchuk played
it, he didn’t have any new idea to make it playable for Black.
11...e5 12.d5 Nd4 is supposed to be the most challenging: Black will develop with ...Be7 and ...0-0,
sacrifice a pawn on d4 (when White plays Be3) and hope for some compensation.

12.Nc3 e6 13.Bg5 Be7 14.Bxf6 Bxf6 15.exd5 Nxd4 16.Re4

260
Position after: 16.Re4

This is all theory.

16...Qb6!?

The alternative 16...Nf5 leads to some advantage for White: 17.dxe6 0-0 18.exf7+ Kh8 19.Qxd8 Raxd8
20.Rd1 and Black does not have full compensation.

17.dxe6 Qc5?N

A new move – but what a terrible one. As I said, this was definitely not a good day for ‘Chucky’.
17...Nxe6 is how Black normally plays, when he’s no more than slightly worse:

Position after: 17...Nxe6

A) 18.Qa4+ Kf8™ [18...Qc6?? 19.Bb5+–] 19.Rb4 Qa7 looks ugly, but isn’t so much for White.
B) 18.Nd5 Qd8 19.Qa4+ [19.Nxf6+ Qxf6 20.Qa4+ Kf8 21.Qb4+ Qe7 is close to equal] 19...b5
20.Nxf6+ gxf6

261
Position after: 20...gxf6

21.Qa3!N [21.Qb4 f5∞ 0-1 (89) Sadykov, R (2255) – Savina, A (2392) Russia 2010] 21...f5 22.Re5
and Black’s position is rather unpleasant, as the white queen on a3 prevents 22...Qg5+?! 23.Kh1 Rg8?
due to 24.Bxb5++–.

18.e7!

Position after: 18.e7!

262
White is just winning. The e7-pawn will be lost, but Black won’t have time to castle and will lose
material.

18...h5

18...Ne6 19.Nd5 Bxe7 20.Rc1 Qa5 [20...Qd6 21.Nc7+!+–] 21.Rc7!+–

19.Rc1 Rh6 20.Kh1 Rg6 21.Ne2

Black loses a piece: 21.Ne2 Qg5 22.h4+–. A nearly ‘free point’ that was definitely welcome in this
tough race.

1-0

I wasn’t able to follow this game live, but was relieved when I got a phone call from Veselin saying all
went great. After this ‘present’ it looked like nothing could prevent him from qualifying for the
Candidates tournament.
Game 5 was against Peter Svidler with the black pieces. They played a critical line of the Queen’s
Gambit Declined in which Veselin had some problems to solve but managed to hold the draw. In round
6 Veselin played aggressively with the white pieces against Rustam Kasimdzhanov, but the latter
outplayed him at some stage. That was a tough game, but Veselin again managed to hold the draw.
Probably Black didn’t even miss a win. Despite two games of not-too-high quality, everything was fine,
as Veselin was on +1 out of six games (3½/6).
The next game was against Hikaru Nakamura with the black pieces. Holding a ‘plus 1’ score was
important, but of course playing the American player with Black is a big test. The game didn’t go well.
Nevertheless, Vesko had a chance to hold it...
Nakamura, Hikaru (2775)
Topalov, Veselin (2793)
Thessaloniki 2013

263
Black to move

50...Nd6?

Spoiling the fortress.


50...Kg7! would (unexpectedly) have held a draw: 51.Kg5 [51.Kxh5 Kf6 transposes] 51...h4! 52.Kxh4
Kf6 53.Kh5 Nd6 54.Kh6 The only way to make progress. 54...Nc8 55.Kh7 Kf7! [55...Nd6 56.Kg8
wins for White] 56.Be2 Again, the only way to make progress, changing the diagonal of the bishop.
56...Nd6 57.Bh5+ Kf6 58.Kg8 b5!= [58...Nxf5? 59.Kf8 Nd4 60.Bd1+–]

50...Nd6? 51.f6!

Now it’s over.

51...Nf7 52.Be2 Nd6 53.Bxh5 b5 54.cxb5 Nxb5 55.Be2 Nd4 56.Bh5 Nxb3 57.d6 Nd4 58.d7 Ne6
59.Bg4 Nd8 60.Bf5 c4 61.Kg4 c3 62.Kf4 Nf7 63.Bd3 Nd8 64.Bc2 Kh5 65.Kf5 Kh6 66.Bb3 Kh7
67.Kg5 Nc6 68.Ba4 Nd8 69.Bc2+ Kg8 70.Kg6 Kf8 71.Bb3 Nc6 72.Kf5 Nd8 73.Ke5 Nb7 74.Kd4
1-0

A tough loss, but excellent play by the American player. It was important to react well against Etienne
Bacrot, but that was another terrible day...
Topalov, Veselin (2793)
Bacrot, Etienne (2725)
Thessaloniki 2013

1.e4 e5 2.Nf3 Nc6 3.Bb5 Nf6 4.d3 Bc5 5.Bxc6 dxc6

264
Position after: 5...dxc6

We had been trying all kinds of moves to avoid castling short and play a sharp position, but hadn’t
succeeded in finding anything matching our expectations. Veselin tried to be creative over the board,
but it all went wrong.

6.Qe2 Bd6 7.Nc3 c5 8.g3 Nd7! 9.Nd5 Nb8!

Bringing his knight to c6, Black reaches a very healthy position.

10.c3 Nc6 11.Bd2 Be7 12.b4?!

The only way to try to justify the previous moves, but the whole idea is dubious.

12...cxb4 13.cxb4 Bg4 14.Bc3

265
Position after: 14.Bc3

14...Bxb4!

Winning a pawn – bad news!

15.Bxb4

15.Nxb4 Nxb4 16.Bxb4 Qf6µ

15...Nxb4

15...Nd4 would have allowed Veselin to do what he loves most in chess: sacrifice his queen! 16.Nxd4
Bxe2 17.Nxe2 c6 18.Ndc3 Qxd3 19.0-0

266
Position after: 19.0-0

However, although at first it optically looks playable for White, after 19...h5 or 19...Qf3 with the idea
...0-0-0/...h5, the engine gives a nasty +3 for Black.

16.Nxb4 Qf6 17.Qb2

17.Nd5 Qxf3 18.Qxf3 Bxf3 just fails for White.

17...Qxf3 18.Qxe5+ Be6 19.0-0 0-0

267
Position after: 19...0-0

20.Nd5?!

20.Nc2! would have given White better drawing chances: 20...Qxd3 [20...Bh3?! 21.Ne1 Qg4
22.Ng2=] 21.Nd4 Rae8 22.Nxe6 Rxe6 23.Qxc7 Qxe4 albeit with a healthy extra pawn for Black.

20...Bh3 21.Nf4 Bxf1 22.Rxf1 c6 23.h3 Rfe8 24.Qc5 h6 25.Qc2 g5!

Position after: 25...g5!

Not giving White time for Re1-e3.

26.Ng2 Rad8 27.Ne3 Qf6 28.Kg2 Qd4 29.Rd1 Re5 30.Qe2

30.Nf5 Rxf5 31.exf5 Qd5+–+ is not something White wants.

30...Ra5 31.h4?

268
Position after: 31.h4?

Not a good move – but the position was very bad anyway.

31...gxh4 32.Qg4+ Qg7 33.Qxh4 Qg5 34.Qh3 Rxa2 35.Nf5 h5 36.d4 Kf8 37.Rh1 Qd2 38.Rf1 Qg5
39.Rh1 Ke8 40.Qxh5 Qxh5 41.Rxh5 a5 42.Rh8+ Kd7 43.Rh7 Rf8 44.g4 Re2 45.Kf3 Re1 46.g5 Rg1

Just a great game by Etienne. Despite being the ‘outsider’ of the event, he reacted with a lot of class to
Veselin’s aggressive play, and there was nothing to be done for White!

0-1

The tournament was turning into a big disappointment! But the roller coaster continued...
Morozevich, Alexander (2760)
Topalov, Veselin (2793)
Thessaloniki 2013

269
White to move

This was surely the critical moment of the game. Veselin was brave to bring his queen to b4 – and
White could have trapped it! Nevertheless, it is unclear whether Black would have been any worse.

23.Nd2?

23.Nc5! It’s hard to tell if Morozevich missed this idea or just wanted more. 23...Bxc5 24.Ra4

Position after: 24.Ra4

270
24...Ka8 [24...Qb5 25.Rd2 doesn’t help much.] 25.Rxb4 Bxb4 With the a5-pawn falling, it seems that
Black should make a draw comfortably.

23.Nd2? Nxa5

Black has taken a whole pawn, and White does not have compensation. Topalov went on to win fairly
comfortably.

24.Nf3 Qb6 25.Qa4 Bb4 26.h4 Rc7 27.Rh1 Rhc8 28.Bd1 Be7 29.b3 Rh8 30.Ng5 Bd8 31.Rf1 Nc6
32.Bg4 Nb4 33.Rd2 Ka7 34.Rff2 Nc6 35.Nxf7 Rxf7 36.Bxe6 Rc7 37.Bxd5 Ne7 38.Be4 Nf5 39.Bxf5
gxf5 40.d5 Rc3 41.d6 Rxb3 42.Rfe2

Position after: 42.Rfe2

42...Rxg3+!

A cute finish.

43.Kxg3 Qg1+ 44.Kf3

44.Rg2 Qe3+ 45.Kh2 Rxh4#

44...Bb6

And ...Qg4# can’t be reasonably avoided, as 45.Rg2 is answered by 45...Qe3#.

0-1

271
A Caro-Kann! Morozevich vs Topalov, Thessaloniki 2013 (photo Anastazia Karlovich)
Vesko was back on 50% – not enough, but getting close to our desired ‘plus 1’ score (6/11). In order
words: aggressive play was required!
The next game against Fabiano was a little sad for us, as Veselin spoiled a nice advantage. However,
the game was simply amazing. Only when writing this book did I realise what a pure joy it was!
Topalov, Veselin (2793)
Caruana, Fabiano (2774)
Thessaloniki 2013

1.e4 e5 2.Nf3 Nc6 3.Bb5 a6 4.Ba4 Nf6 5.0-0 b5 6.Bb3 Bc5 7.Nc3 d6 8.Nd5!

272
Position after: 8.Nd5!

Because of this annoying move, good players (like Carlsen!) later started to play ...0-0 on the previous
move, instead of ...d6. Back in 2013, it was still possible to outplay Fabiano Caruana in the opening! ☺

8...Na5

A) 8...Nxe4?! 9.Qe2 Nf6 10.Nxf6+ followed by Bd5 and d4 is problematic for Black.
B) 8...h6 9.c3 is also great for White, for example 9...0-0 10.d4 Ba7 11.Nxf6+ Qxf6 12.Bd5 Bb7
13.Be3 Qe7 14.a4 Bb6 15.dxe5 Bxe3 16.exd6 Bxf2+ 17.Rxf2 Qxd6 18.Nh4± 1-0 (67) Dominguez
Perez, L (2732) – Kryvoruchko, Y (2710) Berlin 2015.

9.d4!

Bad news for Black - White wants to play Bg5.

9...Nxb3 10.axb3 Nxd5™

The only way not to get a losing position right away, but the cost is high, since Black will no longer be
able to castle.

11.exd5 exd4 12.Re1+ Kf8 13.Nxd4 Bd7 14.Be3 Qf6 15.Qd2 h5!

273
Position after: 15...h5!

A crucial positional resource – expanding on the kingside and freeing the h8-rook. Without this idea,
Black’s position would just be lost.

16.b4!?

A brilliant idea, perhaps stronger than the more obvious 16.Nf3, and guaranteeing White a clear
avantage: 16...Bxe3 17.Rxe3 h4 [17...Qxb2 18.Rae1 Qf6 19.h4 is just awful] 18.Qa5± followed by
Rc3.

16...Bb6 17.Ra3!

17.Nf3?! is not quite as strong as on the previous move, as Black is not forced to take on e3 any more:
17...h4 18.Bxb6 [18.h3?! Bxh3∞] 18...cxb6 followed by ...h3, with some counterplay.

17...h4 18.Ne6+!

274
Position after: 18.Ne6+

This was Topalov’s idea – and it’s a good one!

18...Kg8™

A) 18...fxe6? 19.Bxb6 cxb6 20.Rf3+–


B) 18...Bxe6? 19.Bxb6 cxb6 20.dxe6+–

19.Bxb6 cxb6 20.Ng5

275
Position after: 20.Ng5

White could hardly hope for more: he is better on the queenside, controls the e-file, has more space, a
strong knight on g5, a strong threat in Rf3, and Black has weaknesses on a6 and d6.

20...Rc8?

A) 20...Qxb2 right away is a little too much: 21.Rc3! Rh5 [21...Rf8 22.Rc7 Bf5 23.h3+– followed by
Ree7; 21...Qxb4?? 22.Rc8++–] 22.Rc7 Qf6 23.f4 Qd8 24.Qc3 with a great advantage for White.
B) 20...Rh5! first was the best (not to say only) defence, in order to disturb White’s coordination a bit
before taking the b2-pawn. 21.f4 Already a small victory for Black: Ra3-f3 is no longer a threat.
[21.Ne4 Qh6 22.Qxh6 Rxh6 is better for White, but he’s clearly hoping for more.] 21...Qxb2 Black’s
position is a disaster, so at least he should take a pawn! [Inserting 21...h3 would allow 22.Ne4!
followed by g2-g3, since 22...Qxb2 now loses to 23.Rb3 Qa2 24.Nc3+–.]

Position after: 21...Qxb2

22.Rb3 [22.Rc3 is no longer that precise: 22...Rf8 23.Rc7 Bf5 24.h3 a5! and the position is very
different compared to the 20...Qxb2 line, since the defensive resource ...Rxg5 is coming at any
moment.] 22...Qf6 23.Rbe3 Rf8 24.Re7 Qf5 25.h3

276
Position after: 25.h3

White is clearly better, but Black has chances, as f2-f4 is a weakness White would have liked to avoid.

21.Rf3?

Tempting, but imprecise.


The straightforward 21.Rxa6 was winning:

Position after: 21.Rxa6

277
21...Qxb2 [21...h3 22.Ra7 Qf5 23.f4 hxg2 24.Re7+–; 21...Rh5 22.Ne4 Qxb2 23.Nxd6 is also losing for
Black, e.g. 23...Rxc2 24.Ra8+ Kh7 25.Qd3+ f5 26.Nf7! Rc1 27.Qe3 with the idea Rh8+ and Ne5+]
22.Ra7 with a devastating advantage, as the c2-pawn is untouchable.

21...Qg6!

Position after: 21...Qg6!

Suddenly the position is not that simple, and White needs to play a defensive move for the first time.
21...Qxb2? 22.Rxf7+–

22.Rd3!

White needs to be able to answer ...Rh5 with f4. His position remains great, but he definitely missed his
chance for a straightforward win.
22.Rxf7? Rh5! would be bad news: Black is better!

22...Rh5 23.f4 f6 24.Ne4 Rc7

278
Position after: 24...Rc7

25.Nxd6?

A huge risk in a great position. Instead White could have calmly played 25.c3 for instance.

25...f5

25...Rxd5 was the other critical option which Veselin had surely focused on. After 26.f5 [26.Rxd5?
Rxc2„] 26...Rxd3 27.Qxd3 it looks like White is doing great, but even here, after 27...Qg5 White’s
advantage is reduced compared to what it was three moves earlier.

26.Qe2

279
Position after: 26.Qe2

The obvious answer, which Topalov had in mind. White had other ways to ‘lose’ his d6-knight, and,
funnily enough, the engine evaluates almost all of them as 0.00!
Some sample lines, just for fun:
A) 26.Ne8!? Bxe8 27.d6 Rd7 28.Qc3

Position after: 28.Qc3

28...Rh8™ 29.Rde3 Kh7 30.Qc8 Qxd6 31.Rxe8 Rxe8 32.Qxe8 g6=

280
B) 26.Nf7 Qxf7 27.d6 Ra7 28.Re7 Qg6 29.Qf2

Position after: 29.Qf2

29...Rb7™ 30.Qf3 Ra7™ 31.Qf2= The cutest one!

26...Rh6™

Position after: 26...Rh6™

The move Veselin underestimated, trapping the white knight!

281
27.Ne8 Rc8 28.Re3 Kf8

Something has gone wrong: White definitely has compensation, but it’s clear that this position wasn’t
the desired one!

29.Nxg7?!

White had an astonishing way to force a draw: 29.Re7 Bxe8 30.d6 Rd8

Position after: 30...Rd8

31.Rxe8+!! Qxe8 [31...Rxe8 32.d7! Rxe2 33.Rxe2 Qd6 34.Re8+ Kf7 35.d8=Q=] 32.Qe7+! Qxe7
33.dxe7+ Ke8=.

29...Qxg7 30.c3 Qg4 31.Qd2 Rh7 32.Qd4 Rg7 33.R1e2 Qg6

282
Position after: 33...Qg6

Black plans ...Kg8-h7. Once his king is safe, finding counterplay will be harder for White.

34.b3?!

It looks logical for White to try to create an extra passed pawn with c3-c4, but in fact it is Black who is
going to create one with ...a5 and ...a4. Without this inaccurate pawn move, it would have been far
more difficult for Black to break through, as White definitely has some sort of domination as
compensation for the piece.
34.d6! was correct, with the simple idea to take another pawn on b6. At first it looks like 34...Kg8
35.Qxb6 Bc6 might be annoying, but White can go 36.Re6 Qg4 37.Qd4 (defending the f4-pawn and
threatening h2-h3) with the idea to meet 37...Bxg2?? with 38.Qxg7+!+–.

34...a5 35.bxa5 bxa5 36.h3 a4 37.Qb4+ Kg8 38.bxa4 bxa4 39.c4 Qf6 40.c5 Qf8!

283
Position after: 40...Qf8!

41.d6

Having to play this move, it becomes a little clearer that Black’s passed a-pawn is more useful than
White’s c-pawn.
41.Rc2 might have been better, but surrendering the e-file is not what White wants. After 41...Rg3
42.Qd4 Be8 he is worse anyway.

41...Rb8 42.Qd4 Bc6 43.Re7 a3 44.Qc4+ Kh8 45.Qc3

284
Position after: 45.Qc3

45...Ra8?

Tempting, but bad! The depth of the whole position is just amazing.
45...Kh7! was winning, as 46.Rxg7+ Qxg7 47.Re7?! is no longer possible: 47...Qxe7! 48.dxe7 Rb1+
49.Kh2 a2–+.

46.Qa1?

Again very natural, but wrong because of Black’s next move!


46.d7! was fine for White: 46...a2 47.Qa1 Kh7 48.Rxg7+! Qxg7 49.Qxg7+ Kxg7 50.Rxa2 Rd8

Position after: 50...Rd8

and now Veselin might have missed 51.Ra6! and White will even end up with an extra pawn. He wants
to push c6, while 51...Bxd7? loses to 52.Rd6+–.

46...Be4!

Bad news! Black wants ...Kh7 next, and the e2-rook is no longer able to get to e7.

47.Qe5!

285
Position after: 47.Qe5!

The only practical chance.

47...Bc6?

A) The simple 47...a2 48.Qa1 Bd5! was winning. Black just wants to play ...Rb8 or ...Kg8, while
49.d7 loses to 49...Kh7 50.Re8 Qxc5+ 51.Kh2 Qd6–+.
B) 47...Kh7?? is the trap to avoid: 48.Rxe4! fxe4 49.Rxg7+ Qxg7 50.Qxe4++–.

48.Qa1?

48.Kh2! was correct, as 48...a2 49.Qa1 Bd5 would no longer be so clear with the white king standing
on h2: 50.Rd2! and ...Rb8 is no longer possible.

48...a2?

Black could have repeated 48...Be4 and after 49.Qe5 would get a new chance to find 49...a2 50.Qa1
Bd5!–+.

49.Kh2?!

49.d7! would have transposed into 46.d7.

49...Be4

Clearly, Black is not finding his way.

286
50.Rc7

Not a bad move.


But 50.Rd2, supporting the d-pawn, was surely a better practical choice.

50...Qg8!

Position after: 50...Qg8!

A great move that might easily not enter one’s thoughts, defending the a2-pawn.
50...Kh7 51.Rxa2=

51.Re7??

51.Rd2! Kh7 52.Qf6! was (theoretically) the last chance to save the game: 52...Qf8 [Of course not
52...a1=Q 53.Qxh4+ Kg6 54.Qg5+ Kh7 55.Qh5#] 53.Qxh4+ [53.Qxf8 Rxf8 54.Rxg7+ Kxg7 55.Rxa2
should also be a draw] 53...Kg8 54.Rxg7+ Qxg7 [54...Kxg7 55.d7 Qd8 56.Qxd8 Rxd8 57.Rxa2 Rxd7
58.g4=] 55.d7 a1=Q 56.d8=Q+ Rxd8 57.Rxd8+ Kf7 58.Rd7+ Ke6 59.Rd6+ Kf7 60.Rd7+=.

51...Kh7 52.Qf6 Rxe7 53.Qxh4+ Kg7 54.Qxe7+

287
Position after: 54.Qxe7+

54...Kh8?

Wow! Fabiano gives Veselin one more chance. No need to say how difficult the position was to
understand, resulting in these two players making so many mistakes.
54...Qf7 55.Qg5+ Qg6 56.Qe7+ Kg8 was winning.

55.Rxe4?

55.Rxa2! was a draw by perpetual check! With no rook on a8 or queen on g8, the black king can’t
escape: 55...Rxa2 [55...Qxa2 56.Qf6+ Kg8 57.Qg5+ Kf7 58.Qe7+=] 56.Qh4+ Kg7 [56...Qh7
57.Qd8+=] 57.Qg5+=. An amazing pattern!

55...a1=Q

55...fxe4 was also winning, but to the human eye it doesn’t seem obvious that White has no perpetual
check. At least, if you were to show me that position and the one after 55.Rxa2 (which was indeed a
draw), and ask me which of the two is a draw, I’m not sure how long it would take me to answer
correctly.

288
Position after: 55...fxe4

56.Qe5+ Qg7 57.Qh5+ Kg8 58.Qd5+ Kh7 59.Qh5+ Qh6 60.Qf7+ [60.Qf5+ Qg6 61.Qd7+ Qg7
62.Qf5+ Kh6–+] 60...Kh8–+

56.Re5 Qb2

Position after: 56...Qb2

57.Re2?!

289
The last practical chance was 57.Qh4+ but it isn’t enough to save the game: 57...Qh7 58.Qg5 Qg7
[58...Rg8? 59.Qf6+=] 59.Qh5+ Kg8!–+ [59...Qh7? 60.Re8+! Rxe8 61.Qxe8+ Qg8 62.Qh5+ Kg7
63.Qg5+ Kf7 64.Qe7+ Kg6 65.Qg5+=].

57...Qbg7 58.Qh4+ Q8h7 59.Qe1 Qhg6 60.d7 Qxd7

That’s officially too many queens.

0-1

After this disappointing loss, we knew what was necessary: a ‘plus score’ in Beijing!
The last round of Thessaloniki proved that the Grand Prix system is not ideal. Indeed, Vesko’s
tournament had been a disaster, and his last round result did not matter in the race for the Candidates
Tournament.
The situation was as follows: he had to play against Leinier Dominguez, who was the tournament
leader, but had no chances of qualifying for the Candidates tournament. However, Kamsky, and
especially Caruana, who were second and third before the last round in Thessaloniki, did have chances.
Thus it was better for Veselin if Leinier won the event so the other two did not get more points. In other
words: it was better for him to lose! Obviously, people were wondering whether he would just give
away the point, but the game clearly shows that he didn’t, as despite losing, he fought like a lion for
seventy-one moves.
Dominguez, Leinier Perez (2723)
Topalov, Veselin (2793)
Thessaloniki 2013

White to move

290
This is already move 28 of the game. Black was fine in the middlegame, but after some slight
imprecisions, he had finished up in some trouble.

38.Re4!

White forces a two-results rook ending: definitely Black is only fighting for a draw, and achieving it
isn’t that easy. I decided to show this game, as there are some fascinating moments in the coming rook
endgame!

38...Nxd6 39.exd6 Rxd6 40.Rdxd4 Rxd4 41.Rxd4 Rb8 42.c3 bxc3+ 43.Kxc3 Kf8 44.Rd7 Rc8+
45.Kb2 Ke8 46.Ra7 h5 47.b4 h4 48.Kb3 h3 49.b5

Position after: 49.b5

49...Rc5

49...Rc1 50.Kb4 Rh1 51.b6 Kd8! was the toughest defence: 52.Rxf7 [52.Kc5 Kc8 53.Kc6 Rc1+
54.Kd6 Rd1+ 55.Ke5 Rd2 56.Rc7+ Kb8 57.Rxf7 Rxh2 58.Rh7 Rh1 59.Kxe6 h2 60.Kf6 Rg1 61.Rxh2
Rxg3 leads to a similar position to the game] 52...Kc8 53.Rc7+ Kb8 54.Rc2

291
Position after: 54.Rc2

A) 54...e5, liquidating some pawns, is the first engine suggestion, but it doesn’t see that the final 1 vs
2 position of its line is lost for Black: 55.fxe5 Re1 56.Rc7!? Re4+! (getting the white king further from
the b6-pawn) 57.Kc3 Rxe5 58.Rh7 Re6 59.Kd4 Rxb6 60.Ke5 Kc8 61.Kf4 Kd8 62.Rxh3 Ke7 63.Kg5
Kf7 64.Rh7+ Kg8 65.Rc7

Position after: 65.Rc7

and White wins after 65...Kf8 66.Kh6 Rb2 67.h4 Rb3 68.g4 Rb4 69.g5 Rxh4+ 70.Kxg6+–.

292
B) 54...Kb7 55.Re2 Kxb6 56.Rxe6+ Kc7 57.Re2 and after 57...Rc1 (preventing the white king from
invading the kingside) 58.Re7+ Kd6 59.Rh7 Rc2 60.Rxh3 Ke6 61.Rh7

Position after: 61.Rh7

White should definitely be winning, but engines do not call it a day: White will have to push the black
rook away and hold the kingside pawns at the same time. For example, after 61...Kf5 62.Kb3 Rc8
63.Re7 Kg4 64.Re2 g5 65.fxg5 Kxg5 66.Rc2 Rb8+

Position after: 66...Rb8+

293
followed by ...Kg4, tablebases say this is a win in seventy-one moves. It is not easy for White to get
his king next to his pawns, and after analysing the position with tablebases (which I advise you to
do!), I must say that winning it seems extremely difficult if Black defends sensibly.

50.Kb4 Rc2 51.b6 Kd8 52.Rxf7 Rxh2

Position after: 52...Rxh2

53.Rh7?

White played the move most people would have gone for, but the straightforward 53.b7! was winning.
53...Rb2+ 54.Kc3 Rb1 55.Rh7 h2 56.Rxh2

294
Position after: 56.Rxh2

56...Kc7™ [56...Rxb7 leads to a lost pawn ending: 57.Rh8+ Kc7 58.Rh7+ Kc6 59.Rxb7 Kxb7 60.Kd4
Kc6 61.Ke5 Kd7 62.Kf6+–] 57.Rh7+ Kb8 58.Rg7 Rg1 59.Rxg6 Kxb7 60.Kd4 and the black King is
too far away: 60...Re1 [60...Kc7 61.Ke5 Re1+ 62.Kf6+–] 61.Kc5 Kc7 62.Rg7+ Kd8 63.Kd6+–.

53...Kc8 54.Kc5 Rc2+ 55.Kd6 h2 56.Kxe6 Rg2 57.Ke5 Rxg3 58.Rxh2

Position after: 58.Rxh2

295
58...Rb3?!

A bad decision.
I don’t believe Veselin thought the alternative below was losing, but he probably thought both moves
were a draw – a logical assessment if you miss Leinier’s brilliant idea on the next move.
58...Kb7 was just a draw: 59.Rb2 [59.Rh6 Rg4=] 59...Rg1 60.Kf6 Rg4 61.Rb4 g5=.

59.Rh8+!

The only try.


59.Kf6 Rxb6+ is an easy draw for Black: 60.Kf7 Kd7 61.Rh7!? Ra6 62.Rg7 Ra4 63.Kxg6+ Ke8=.

59...Kb7 60.Rh7+

Position after: 60.Rh7+

60...Kxb6?

Difficult not to take this pawn in time trouble, but this is the losing move!
60...Kb8! and Black will put his rook on g4, when White has no way to break through!

61.Kf6

The most technical way to win. Crazily enough, Black loses because his king is cut off ‘the other way
round’ along the seventh rank – you usually say it’s cut off when it’s stuck on the eighth rank!
The more direct 61.Rh6 also worked: 61...Kc7 62.Rxg6 Kd7 63.Kf6!

296
Position after: 63.Kf6!

63...Rf3 [63...Rb6+ 64.Kf7 Rb5 65.Rg5!+–] 64.f5 Rf1 65.Rg8 followed by Rf8 is winning for White. If
the black king was on h7 rather than d7, it would be a draw. As I explain in my book Chess Calculation
Training – Volume 2, Endgames, the rule is that the defensive king should always be on the ‘short side’
in this configuration. Here, the g- and h- files are the short side, and the a- to e-files the long one. If you
do not know this endgame, I advise you to study it, as it is one of the must-know rook endings.

61...Rb4

Defending from the other side isn’t any better: 61...Rf3 62.Rh4

297
Position after: 62.Rh4

62...Rg3 [62...Kc7 63.Kxg6 Kd7 64.Kf6 Ke8 65.f5+–] 63.Rh6! Rg4 64.Rxg6 Rxf4+ 65.Ke5++–.

62.Kg5 Kc6 63.Rf7!

Position after: 63.Rf7!

The black king is unable to come back in time.

298
63...Kd5 64.Kxg6 Rb6+ 65.Kg5 Rb8 66.f5 Rg8+ 67.Kf6 Ke4 68.Ra7 Rf8+ 69.Kg6 Rg8+ 70.Kf7 Rh8
71.Kg7

A truly fascinating endgame! It is quite funny that after the story behind the Ivanchuk-Dominguez
game, the Cuban went on to win the event!

1-0

Despite the obvious fair play in this game, anti-Topalov people said publicly anyway that he had lost on
purpose, but the funny part was Veselin’s reaction: “When I win people say I cheat (referring to San
Luis 2005), when I draw they say I pre-arrange (referring to his game with Radjabov in Zug 2013), now
when I lose they say it’s on purpose!”. Of course he was saying that humorously, but it wasn’t entirely
wrong!

Things were hotting up. Vesko was leading the general standings with 355 points, closely followed by
Caruana on 305 points and Mamedyarov on 240. Quite a gap, you might think, but it could easily be
turned around in just one tournament, and Veselin couldn’t afford a second bad performance in Beijing
– that would have dropped him out of the top two for sure!

299
Chapter 7
Rollercoaster in Beijing!

Drawing of lots in order to determine pairings. 10, a lucky number or not? (photo Anastazia Karlovich)
Round one of the Beijing Grand Prix paired Veselin, as before, with the black pieces against Boris
Gelfand.
This time Veselin decided to switch from his usual Queen’s Gambit, and went into the game in highly
aggressive mode. He played extremely risky chess, which could have gone wrong, but eventually paid
off!
Actually it was Silvio Danailov who told us after Veselin’s previous game against Boris, «Come on!
Go for the Grünfeld next time!», and we decided to listen!
Gelfand, Boris (2773)
Topalov, Veselin (2767)
Beijing 2013

1.d4 Nf6 2.c4 g6 3.Nc3 d5

300
Position after: 3...d5

4.Nf3 Bg7 5.Qb3 dxc4 6.Qxc4 0-0 7.e4 Na6 8.Be2 c5 9.d5 e6 10.0-0 exd5 11.exd5 Bf5 12.Bf4 Re8!?
13.Rad1 Ne4 14.Nb5

Position after: 14.Nb5

In this still-theoretical position, Veselin went for an aggressive move, which is probably objectively
dubious.

301
14...g5?!

A bit risky, but knowing how much Vesko likes to advance his g-pawn, I can tell you from experience
that trying to convince him not to overdo it is very difficult. You might end up getting your knuckles
rapped!
A) 14...Bxb2? is far too risky. Two old games prove it: 15.Bd3!? [15.d6!? Bd7 16.Qb3 Qf6 17.Be3
and the black pieces already lack squares: Goloshchapov, A (2446) – Khaetsky, R (2294) Alushta
1999, 1-0 (28)] 15...Qa5 16.Rb1 Bf6 17.g4 Bd7 18.a4 Nc3 19.Bd2 and White won a piece in the
game Dlugy, M (2500) – Kouatly, B (2430) Paris 1986, ½-½ (49).
B) 14...Qf6 (first seen in Karpov v Kasparov 1986, a few months after the game I’ve just mentioned)
leads to complicated positions which have been explored in several high-level games.

15.Be3 h6

15...Bxb2 is again very dangerous for Black, for similar reasons as above.

16.d6

This pawn must be pushed!

Position after: 16.d6

To be honest, this isn’t a great version of the line for Black already.
The black pawn isn’t bad on g5, and he has gained some space and diminished the power of White’s
light-squared bishop, but objectively centralizing the pieces was more urgent.

16...Qd7 17.a3!?

302
A very interesting move, preventing ...Nb4 forever.
17.Qc1 was also good.

17...Rad8

White invited Black to take the b2-pawn in acceptable circumstances with 17...Bxb2, but of course
White would get huge compensation after several moves, like the two suggested by the engine:
18.Rd5!? (with the idea 19.Bd3) and 18.h4 (with the idea 18...g4 19.Nd2), or just 18.Rfe1.

18.Qc1

OK, time to protect the pawn.

18...Qe6 19.b4!

Position after: 19.b4!

An excellent move, and obviously another point in playing 17.a3.

19...cxb4 20.Nfd4 Qf6 21.Nxf5 Qxf5

After a perfect game so far, Gelfand makes a first bad decision.

22.Bxa7?

22.axb4! Nxb4 23.Bc4

303
Position after: 23.Bc4

would have given White tremendous compensation, probably with a great advantage, though after
23...a6 24.Nc7 Rf8 it would have been important to find 25.Qb1! with the idea 25...a5 26.Nd5 Nxd5
27.Rxd5 and Black’s position collapses: 27...Qg6 [27...Qe6 28.Rd4 Nc3 29.Qd3 wins a piece] 28.d7
with total domination for White.

22...Nc3 23.Nxc3 bxc3 24.Bd3 Qf6

Position after: 24...Qf6

304
25.Bxa6?!

Another dubious decision.


It was time to force a draw with 25.Bb5 Re6 26.Bd4 Qf5 [26...Qxd4 27.Rxd4 Bxd4 28.d7 is very
different compared to the 26.Bd4 line you will see in the game, as only White can be better with his
light-squared bishop still alive] 27.Bxg7 c2 28.Rd2 Kxg7 and the c2- and d6-pawns will be swapped.

25...bxa6 26.Bc5?!

Luck is definitely turning our way!


26.Bd4 allowing 26...Qxd4 27.Rxd4 Bxd4

Position after: 27...Bxd4

was necessary, but of course only Black can win here. He’s winning the d-pawn and his c3-pawn is a
monster. After the logical 28.h4 the engine finds a brilliant idea: 28...Re2! [stronger than 28...Re5]
29.hxg5 Rd2! followed by ...Rxd6, and White will have to be accurate if he wants to make a draw.

26...Qf5 27.Bb6 Rd7

305
Position after: 27...Rd7

28.a4?!

28.Bc7, to be able to answer 28...Re2 with 29.Rde1, was necessary, but after 29...Rd2 White’s position
remains very bad.

28...Re2

Now White’s position is just lost. No need to mention that Black’s c3-pawn is much stronger than the
one on d6.

29.Qa3 c2 30.Rc1 Bf8

A straightforward way to win the d6-pawn, although the engine likes 30...Be5 even more: 31.Bc7 Qf4
32.g3 Bxd6!–+.

31.Bc5™

31.Bc7? Rxc7–+

31...Re5! 32.Bb4 a5 33.Bc3 Bxd6 34.Qb3 Re2

306
Position after: 34...Re2

Now it’s easy.

35.Qc4 Rde7 36.Qc6 R7e6 37.Qc8+ Bf8 38.Bb2 Qd3 39.g3 Rd2 40.Qc3 Qxc3 41.Bxc3 Rde2

0-1

Next day Veselin played Peter Leko with White. It’s worth a look as an example of how top preparation
from both sides can lead to a draw. Veselin came to the game with a clear idea that we had prepared,
but Leko had anticipated it, just as we had managed to do against Morozevich in Zug.
Topalov, Veselin (2767)
Leko, Peter (2737)
Beijing 2013

1.d4 Nf6 2.c4 e6 3.Nf3 b6 4.g3 Ba6 5.b3 Bb4+ 6.Bd2 Be7 7.Nc3 c6 8.e4 d5 9.Bd3 dxe4 10.Nxe4
Bb7 11.Qe2 Nbd7

307
Position after: 11...Nbd7

12.Nxf6+!?N

An interesting try, involving a pawn sacrifice on move 14.


One month prior to this game, the players met with the same colours at the Kyiv Sberbank rapid in
Ukraine. Vesko had decided not to show the above idea for a rapid game, and the game ended in a
boring draw: 12.Rd1 c5 13.dxc5 Nxc5 14.Nxc5 Bxc5 15.0-0 0-0 16.Bg5 Bxf3 17.Qxf3 Qe7 18.Bc2
Rad8 19.Kg2 h6 20.Bxf6 Qxf6 21.Qxf6 gxf6 22.Rxd8 Rxd8 23.Rd1 Rxd1 24.Bxd1 ½-½ Topalov, V
(2784) – Leko, P (2737) Ukraine 2013.

12...Nxf6 13.0-0

13.b4?! would be too early: 13...c5 14.dxc5 bxc5 15.b5 a6! and Black is better because White doesn’t
have the move a2-a4 here.

13...c5!

Necessary.
13...0-0?! would now allow 14.b4! with a nice space advantage for White: 14...c5 15.dxc5 bxc5 16.b5!
a6 17.a4±.

14.d5!

308
Position after: 14.d5!

Critical.

14...exd5 15.Rfe1 dxc4 16.Bxc4 0-0! 17.Rad1

Position after: 17.Rad1

All the previous moves were forced. Now we were hoping that Peter would be out of his preparation
and, hopefully, make a bad decision. But unfortunately, he knew it all.

309
17...Qd7!

A) 17...Nd5 leads to a crazy line by force: 18.Qe4 Bf6 19.Bf4!? Nc3™ 20.Qxb7 Nxd1 21.Ne5

Position after: 21.Ne5

and now after 21...Bxe5! [21...Qd4 22.Qf3 Qxf2+ 23.Qxf2 Nxf2 24.Kxf2²] 22.Bxe5 b5! 23.Qxb5
Qd2 24.Re2 Qc1 25.Kg2 Nc3 26.Bxc3 Qxc3 27.Qxc5 we had considered that White was pushing,
but that Black should hold.
B) But another knight move would not be a good idea: 17...Ne4?! 18.Bf4 Qe8 19.Qc2

310
Position after: 19.Qc2

19...Nf6 [19...Rd8 20.Rxd8 Bxd8 21.Qd3 and again Black is paralyzed, since 21...Qc6 loses to
22.Ne5! and if 22...Nxf2 then 23.Kxf2 Qg2+ 24.Ke3+–] 20.Ne5 Rd8 21.Qe2‚ and White has got
what he wanted: Black is dominated with no counterplay.
C) The same applies to another queen move: 17...Qc7? 18.Ng5!

Position after: 18.Ng5!

18...Rae8 [18...Nd5? 19.Bxd5 and Black doesn’t have ...Qxd5 as he does with his queen on d7]
19.Bc3 and if 19...Bd8 then 20.Qd3 with the terrible threat of Bxf6. Black has huge problems:
20...Rxe1+™ 21.Rxe1 g6™

311
Position after: 21...g6™

22.Nxf7! Rxf7 23.Rd1 Nd5™ [23...Be7 24.Bxf6 Bxf6 25.Qd7!+–] 24.Bxd5 Bxd5 25.Qxd5±.
D) However, the counter-sacrifice 17...b5!? made sense, with the idea ...Qb6.

18.Ng5

Here we had also tried 18.Ne5, but without success. However, maybe Veselin should have gone for
this. 18...Qh3 19.f3

312
Position after: 19.f3

Here Black has two options, both leading to fun lines:


A) 19...Bd5 20.Bc3 [20.Bg5 Bxc4 followed by ...Bd8 appeared rock solid] 20...Rad8! 21.Nxf7
[21.Rxd5 Nxd5 22.Nc6 Nxc3 23.Nxe7+ Kh8 24.Qc2 Nxa2 25.Qxa2 Rd6!∞] 21...Bxc4 22.Qxc4
Rxf7 23.Rxd8+ Bxd8 24.Bxf6

Position after: 24.Bxf6

24...Qd7!= [24...gxf6 25.Re8+ Kg7 26.Rxd8 would be highly unpleasant for Black].
B) 19...Nd5 20.Ng4 h5 21.Nh6+! gxh6 22.Bxh6 Bf6

313
Position after: 22...Bf6

23.Bxf8 [23.Bxd5 Bxd5 24.Bxf8 Qf5 25.Bd6 Qxf3 26.Qxf3 Bxf3©] 23...Nf4™ 24.Qf1 [24.gxf4?
Bxf3–+] 24...Kxf8 25.gxf4 Bd4+ 26.Rxd4 Qxf1+ 27.Kxf1 cxd4 with approximate equality.

18...Nd5!

The tempting 18...Qc6?! doesn’t equalize: 19.f3

Position after: 19.f3

314
19...h6™ [19...Nd5 isn’t possible now, as after 20.Bxd5 Qxd5 21.Qxe7 there is no longer a mate on g2]
20.Nxf7 Rxf7 21.Bxf7+ Kxf7 22.Qxe7+ Kg6 23.Qe3 Qxf3 24.Qxf3 Bxf3 25.Rc1 with definitely some
advantage for White.

Position after: 18...Nd5!

19.Qd3

We had no greater success with other moves. Some sample lines:


A) 19.Nxh7 Kxh7 20.Bc3 Rad8 21.Bxd5 Bxd5 22.Qh5+ Kg8 23.Re5 f5=
B) 19.Qh5 Nf6! [19...Bxg5 20.Bxg5©] 20.Bxf7+ Kh8 21.Qh4 h6 22.f3!?

315
Position after: 22.f3!?

Covering the important a8-h1 diagonal. 22...Nd5 23.Be6 Qd8 24.Nf7+!? Rxf7 25.Qh5 Rf6 26.Bxd5
Qxd5 [26...Bxd5? 27.Bf4±] 27.Qxd5 Bxd5 28.Rxe7 Re6=

19...Bxg5 20.Bxg5 f6 21.Bxf6 Rxf6

Position after: 21...Rxf6

22.Bxd5+

316
22.Re5 was also possible, but remained very drawish.

22...Qxd5 23.Qxd5+ Bxd5 24.Rxd5 Kf8

And the game is just a draw.

25.Kf1 Re8 26.Red1 g6 27.h4 h5 28.Rd7 Re7 29.Rd8+ Kf7 30.Rc8 Rfe6 31.Rd3 Re8 32.Rc7+ R8e7
33.Rc8 Re8 34.Rc7+ R8e7 35.Rc8 Re8

½-½
An interesting game, but which actually wasn’t much of a fight!
The next two games were also draws, against Kamsky and Ivanchuk.
Then round 5 could have quickly gone wrong when the Chinese star Wang Hao missed a beautiful
opportunity to get a significant advantage with the white pieces.
Wang, Hao (2752)
Topalov, Veselin (2767)
Beijing 2013

1.d4 Nf6 2.c4 g6 3.g3 Bg7 4.Bg2 d5 5.cxd5 Nxd5 6.Nf3 0-0 7.0-0 Nb6 8.Nc3 Nc6 9.e3

Position after: 9.e3

9...e5?!

A risky line which seems dubious.

317
Nowadays it is well-known that 9...Re8! is a better move. Actually, Veselin had already played it twice
before, and I assume he wanted to surprise his opponent in this game!

10.d5 Na5 11.Nd2 f5 12.e4 Nac4 13.Na4 Nd6 14.Nc5 a5 15.b3 Qe7 16.a4 Rd8 17.Ba3 Qf7 18.Rc1
fxe4 19.Ndxe4 Nxe4 20.Nxe4 Nxd5?!

Position after: 20...Nxd5?!

I have also used this example in one of my books of exercises.

21.Nd6?

21.Bd6!! would have been an amazing blow: 21...Nb4 [21...cxd6? 22.Ng5+–; 21...c6? 22.Ng5+–;
21...h6?! 22.Bxc7!+–] 22.Rxc7 Qxc7 23.Bxc7 Rxd1 24.Rxd1 and White is much better.

21...cxd6 22.Rxc8 Nf6!∞

Now the position is just unclear, and the game ended in a draw in 49 moves.
After this game there followed a draw with Alexander Grischuk. Veselin was still standing on ‘+1’ (one
win and the rest draws), which was enough to qualify if he could keep it until the end, but the next day,
a complicated tactical battle against Mamedyarov didn’t go our way...
Mamedyarov, Shakhriyar (2761)
Topalov, Veselin (2767)
Beijing 2013

1.d4 Nf6 2.c4 e6 3.Nc3 Bb4 4.Qc2 0-0 5.Nf3 c5 6.dxc5 Na6 7.g3 Nxc5 8.Bg2 Nce4 9.0-0 Bxc3

318
10.bxc3 d5 11.Ng5 Nd6 12.cxd5 exd5 13.Rd1 Bf5 14.Qb3 h6 15.Nh3 Bxh3 16.Bxh3 Re8 17.Bg2

Again, this line is one of those Veselin had in store before starting working with me – it is difficult for
me to comment on it. But, the result of the opening was definitely completely acceptable for Black!

Position after: 17.Bg2

17...Rxe2!?

A risky choice, but objectively correct.


17...Rc8 was however the easy way: 18.Bxd5 [18.Be3 followed by Bd4 could have been the other way
for White to get some advantage, but there’s no time for it, since Black can happily play 18...Rxe3!
19.fxe3 Qe7 with huge compensation] 18...Nxd5 19.Rxd5 [19.Qxd5 Ne4©] 19...Qf6 with typical
compensation, as the c3-pawn is very weak and the c1-bishop passive.

18.Bxd5 Nxd5 19.Qxd5 Qf6 20.Bf4

319
Position after: 20.Bf4

20...Ne4?

Entering a forced variation that doesn’t work for Black.


After 20...Nf5! 21.Qxb7 Rae8 22.Qf3 R2e4! with the idea ...Nh4, Black would have enough
compensation for equality!

21.Qd3! Nxc3

320
Position after: 21...Nxc3

22.Rac1!

The move Veselin missed, I assume. Otherwise, Black would be better.


22.Rdc1?? Rc8 [or even 22...Rd8!? with the idea 23.Qxc3? Rd1+!–+] and 23.Rxc3? no longer works:
23...Qxc3 24.Qxe2 Qxa1+–+.

22...Nxd1?

22...g5! was the only way to stay in the game; after 23.Rxc3 [23.Be3? Rxe3 24.Qxe3 Nxd1µ] 23...gxf4
24.Qxe2 Qxc3 25.Qg4+ Qg7 26.Qxf4

Position after: 26.Qxf4

White is clearly better due to Black’s weakened king, but Black is still fighting.

23.Qxe2 Nc3 24.Qc4

321
Position after: 24.Qc4

And the c3-knight is trapped. Playing 24...g5 no longer helps, as now White can just move his bishop.

1-0

There were now just four games left, and two and a half more points were needed.

322
The next game saw Topalov, with the black pieces again, take on Sergey Karjakin. The latter definitely
didn’t play a good game. Veselin stayed focused and scored an important point.
Karjakin, Sergey (2776)
Topalov, Veselin (2767)
Beijing 2013

1.e4 c5 2.Nf3 e6 3.d4 cxd4 4.Nxd4 Nc6 5.Nc3 Qc7 6.Be3 a6 7.Qd2 Nf6 8.0-0-0 Be7 9.f3 b5 10.g4
Nxd4 11.Bxd4 Bb7 12.Qf2?!

Position after: 12.Qf2?!

This doesn’t look like a dangerous move.

12...0-0 13.Bd3 Rac8!?

13...b4 14.Na4 d5 15.Nb6 dxe4 16.Nxa8 Bxa8 17.fxe4 Nxe4 was also great for Black in Wegener, O
(2444) – Nisipeanu, L (2678) Germany 2016, 0-1 (40).

14.Ne2 d6 15.Ng3 Nd7 16.Rhe1 Nc5

White’s whole plan is quite weird. Black is already slightly better.

17.Kb1 Rfd8 18.b3 e5 19.Bb2

323
Position after: 19.Bb2

19...Bh4!

Stopping any h4-move. Of course, the bishop only stays there temporarily... but is temporarily very
useful!

20.Rg1

The aggressive 20.f4? isn’t good: 20...exf4 21.Qxf4 Nxd3! 22.cxd3 Qc2+ 23.Ka1 Qxh2! and White’s
best is now 24.Rf1 f6 25.Nf5 Qxf4 26.Rxf4 Bg5, but with a big extra pawn for Black.

20...g6 21.Qe3

21.f4 loses a pawn again: 21...Nxe4µ.

21...f6 22.f4

White should seek counterplay, and this is a better moment to play this move than it was on moves 20
and 21.

324
Position after: 22.f4

22...exf4

A natural move, as even if Black wants to go for the usual ...Nxd3+, ...Qc2, ...Qxh2 idea, an early take
on f4 is necessary: 22...Nxd3 23.cxd3 Qc2+ 24.Ka1 Qxh2?! [24...exf4! was still in time, see 23...Nxd3
in the game] 25.Rd2! Qh3 26.Rdg2! and White gets nice counterplay, as the queen on h3 is a little
suspect.

23.Qxf4 d5!?

Going for a pawn was also good, although I prefer Topalov’s move: 23...Nxd3 24.cxd3 Qc2+ 25.Ka1
Qxh2 26.Rh1! Qxg3 27.Qh6 with some drawing chances for White.

24.Qxc7 Rxc7

325
Position after: 24...Rxc7

25.Rgf1?!

25.e5 probably caused the least damage, although after 25...Nxd3 26.Rxd3 fxe5 27.Bxe5 Rf7 Black
remains better.

25...d4 26.Rf3 Re7 27.Ba3 Re5 28.Bxc5

Not really a move you like to play, but otherwise Black will just win a pawn.

28...Rxc5 29.c4

326
Position after: 29.c4

29...bxc4?!

Imprecise, it seems.
29...Re5 or 29...Kg7 was probably stronger.

30.Bxc4+ Kg7 31.Kb2

Position after: 31.Kb2

327
31...Rg5

The best practical choice.


Black could also go after a pawn with 31...Bxg3 32.Rxg3 Re5 but White will definitely have great
drawing chances.

32.h3?

32.Rfd3! was necessary, though after 32...Rxg4 33.Rxd4 Rxd4 34.Rxd4 Kh6! White still has to suffer.

32...h5 33.Ne2 hxg4 34.hxg4 Rxg4 35.Nxd4

35.Rf4 Rxf4 36.Nxf4 Rd6 37.Nd5 g5µ

35...Kh7!

Position after: 35...Kh7!

Black’s position is technically winning: the f- and g- pawns will prove too strong.

36.Rh3 g5!

36...Rxe4 37.Rhd3 f5 38.Ne6 Rxd3 39.Rxd3 Kh6 was also good, but Topalov’s move is even stronger.

37.Kc3 Bxe4 38.Bxa6 f5 39.Bd3 Kg6

328
Position after: 39...Kg6

40.Nf3?

The final mistake.


40.a4 was better, though after 40...Bf2 [40...Kf6!?] 41.Nc6!? Bxc6 42.Bxf5+ Kxf5 43.Rxd8 Be1+
Black should be winning.

40...Bg3!

Now White is fully dominated, and loses the house.

41.Be2 Rc8+ 42.Kd2 Rc2+ 43.Ke3 Rc3+ 44.Bd3 Bxd3 45.Rxd3 Bf4+ 46.Ke2 Rg2+ 47.Kf1

329
Position after: 47.Kf1

47...Rxd3

47...Rcc2!?–+

48.Kxg2 Rd8!

Of course not 48...g4?? 49.Nh4++–.

49.Ne1

49.Rh1 g4–+

49...Rd2+ 50.Kf1 g4 51.Rc3 Rxa2 52.Nd3 Kg5 0-1

In the next game, we had a nice (and at the time, not so well-known) idea to try to beat the ‘Chinese
Wall’, Wang Yue. He surprised us on move 16, playing a move we hadn’t considered critical, and it
turned out to be difficult to find a real advantage after it.
Topalov, Veselin (2767)
Wang, Yue (2705)
Beijing 2013

1.d4 d5 2.c4 c6 3.Nf3 Nf6 4.Nc3 dxc4 5.a4 Bf5 6.Ne5 Nbd7 7.Nxc4 Nb6 8.Ne5 a5 9.f3 Nfd7 10.e4
Nxe5

330
Position after: 10...Nxe5

11.exf5!?

At the time, entering an endgame with 11.dxe5 was critical, and had even been played several times by
Topalov himself.

11...Ned7

The best move. Instead, 11...Nec4?! is inaccurate. Later, I won a nice game in this line: 12.Qb3! Nd6
[12...Qxd4? 13.Bxc4+–] 13.g4!

331
Position after: 13.g4!

13...g6 [Topalov and I considered 13...Nd7!? as a slightly better move.] 14.Bf4 Bg7 [We considered
14...gxf5 extremely unpleasant for Black: 15.0-0-0 e6 16.Kb1!?N; 14...Nd7?! 15.Be5 Rg8 16.Bxd6
exd6 17.Qxb7 Qc8 18.Ba6! also turned into a nightmare for Black in Li Shilong, – (2519) – Jia, H
(2369) China 2011, 1-0 (46).] 15.0-0-0 gxf5 16.Rg1!?

Position after: 16.Rg1!?

16...Qc7 [16...fxg4 17.Rxg4 Kf8 18.Kb1‚] 17.gxf5 Bf6 18.Kb1 0-0-0 19.Rc1 [19.Ne4!?] 19...Bxd4?
[19...Kb8 was required] 20.Nb5!+– 1-0 (26) Edouard, R (2670) – Oleksienko, O (2554) Dubai 2013.

12.d5

332
Position after: 12.d5

Critical, otherwise Black solves his problems with 12...g6.

12...g6!

Anyway! If 12...cxd5?!, then 13.Bb5! e6 14.Qe2 is great for White.

13.dxc6 bxc6 14.Qd4 Rg8!

After 14...Nf6 15.Qc5 Black is likely to lose his c6-pawn.

15.fxg6

We didn’t like White’s position too much after 15.Qe4 gxf5 16.Qxf5 e6!? 17.Qxh7 Rg6©.

15...Rxg6!

333
Position after: 15...Rxg6!

An important move.
We considered 15...hxg6N 16.Qe4! Qc7 17.Be3 followed by Rc1 highly unpleasant for Black.

16.Qf2!

A good move which was almost a novelty at the time. White just wants to free his f1-bishop. If he can
complete his development, his position will be great, as he enjoys the bishop pair, better pawn structure
and safer king.
16.Be3? had been played, for example, but after 16...Bg7 17.Qd2 Rb8!N White has serious problems
with his development.

16...Nd5

The move we underestimated.


A) We considered 16...Rb8 critical. We wanted to go for 17.Bd3!?N [A few months later, 17.Be3?!
proved not to be great for White: 17...Re6 18.Be2 Nd5 19.Nxd5 cxd5 20.Bd4 Bh6³ ½-½ (31) Li Chao
– Vachier-Lagrave, M Gibraltar 2014.] 17...Re6+ 18.Be2

334
Position after: 18.Be2

(the idea of provoking ...Re6 was that Black can’t develop easily with ...e6) and here we considered
that one of the best lines for Black was 18...Nf6 19.0-0 Nbd5 20.Bc4 Qb6 21.Qxb6 Rxb6 with
nevertheless a more pleasant position for White after 22.Nd1 with the idea Rb1, Nf2 etc.
B) One of the traps behind 16.Qf2 is that the tempting 16...Ne5?! 17.f4!N 17...Nd3+? is bad: 18.Bxd3
Qxd3

Position after: 18...Qxd3

335
19.f5! [19.Qxb6?? Bg7 20.Ne2 Re6 21.Qf2 Rd8 and Black wins] 19...Rd6 20.Qxb6 Bg7 21.Be3!
Rad8 [21...Bxc3+ 22.bxc3 Qxc3+ 23.Kf2 Qc2+ 24.Kg1+–] 22.Ne2! Qxf5 23.Rf1 with insufficient
compensation for the piece.

17.Nxd5 cxd5 18.Bb5

Position after: 18.Bb5

White’s position looks very pleasant, but thanks to his next few moves, Black also manages to get easy
play.

18...Re6+ 19.Kf1 Bg7 20.g3 Kf8 21.Kg2 d4 22.Rd1 Nb6 23.Ra3 Rc8

336
Position after: 23...Rc8

24.Rad3?!

After this move, Black solves all of his problems.


24.Rc3! was the only try.

24...Nd5 25.Rxd4

Logical, otherwise Black is just slightly better.

25...Rxc1

25...Bxd4! 26.Qxd4 Kg8! was better for Black, according to the engine.

26.Rxd5 Rxd1 27.Rxd8+ Rxd8

337
Position after: 27...Rxd8

White has an extra pawn, but no winning chances; Black’s rooks are too strong. Later, White even got
into slight trouble, but the position was too drawish.

28.Bc4 Rc6 29.Qc2 Rd4 30.b3 Rcd6 31.Qf5 Rd2+ 32.Kf1 Bf6 33.Qxh7 e6 34.Qh6+ Ke7 35.Qe3
Rxh2 36.Qc5 Bc3 37.Bd3 Bb4 38.Qg5+ Kf8 39.Kg1 Ra2 40.Qh6+ Ke7 41.Qg5+ Kd7 42.Qh5 Kd8
43.Qh7 Rd5 44.Kf1 Bc3 45.g4 Ke7 46.Qe4 Kf8 47.f4 Rd2 48.Bc4 R5d4 49.Qe3 Bb4 50.f5 Rxg4
51.fxe6 Rdd4 52.Qh6+ Ke7 53.exf7 Rdf4+ 54.Ke2 Rg2+ 55.Kd3 Rg3+ 56.Ke2 Rg2+ 57.Kd3 Rg3+
58.Ke2 Rg2+ 59.Kd3 ½-½

Only one more point from two games was necessary to qualify, and even probably win the Grand Prix
series.
Veselin had to play Anish Giri with the black pieces, and then against his ‘lucky cat’ of the series,
Alexander Morozevich, this time with White. The plan was to play solidly against Anish, but somehow
‘Topi’ soon forgot about the instructions.
Giri, Anish (2734)
Topalov, Veselin (2767)
Beijing 2013

1.e4 c6 2.d4 d5 3.e5 Bf5 4.Nf3 e6 5.Be2 Bg6 6.0-0 Nh6!?

338
Position after: 6...Nh6!?

An interesting but risky line that will lead to unbalanced positions once White has given up the bishop
pair to spoil Black’s pawn structure on the kingside.

7.c3 Be7 8.Bxh6 gxh6 9.g3 0-0 10.Nbd2 a5 11.Ne1!?

An interesting and well-known plan.

11...c5!

The right reaction.


Often, Black reacts with 11...f6 in that kind of position, but here White is just too organized in the
centre: 12.Nd3±.

12.h4

339
Position after: 12.h4

12...cxd4!

12...f6 still seems a bit early: 13.h5 Bf7 14.exf6 Bxf6 15.dxc5 should be good for White.

13.h5!?

13.cxd4 f6! would now be fine for Black, as the b8-knight is coming to c6.

13...Be4

I don’t know why Vesko didn’t play the concrete 13...dxc3, but it was quite good: 14.bxc3 [I assume he
was afraid of 14.hxg6, as the rest is clearly an improvement compared to the game, but it is absolutely
fine for Black: 14...cxd2 15.gxf7+ Rxf7 16.Qxd2 Qb6 17.Qxh6 Rg7∞.] 14...Be4

340
Position after: 14...Be4

15.Qa4! [15.f3?! is too risky: 15...Bf5 16.g4 Bc5+ 17.Kg2 Nc6 and due to ...Qg5+, it is difficult to see
how White is ever going to capture the f5-bishop.] 15...Qc7! 16.Nxe4 Qxe5! with an easier position for
Black. [16...dxe4? 17.Qxe4 followed by Bd3 would be terrible for Black.]

14.cxd4 Nc6

Position after: 14...Nc6

341
15.Bf3

Also here 15.f3? is too risky: 15...Bf5 16.g4 Nxd4! 17.gxf5 Nxf5 followed by ...Bc5+, and Black is
close to winning.

15...Bf5?

Veselin tries to go for the same idea as above, but with Bf3 played and not f2-f3, it doesn’t work at all!
15...Bxf3 is not an exchange Black generally wants, but actually after 16.Ndxf3 Qb6 followed by ...f6,
he would be fine. In fact, since White hasn’t trapped Black’s light-squared bishop, you might wonder
what the h-pawn is doing on h5!

16.g4

Position after: 16.g4

16...Nxd4?

A) 16...Bb4? doesn’t work either: 17.a3!? [17.gxf5 Qg5+ 18.Bg2 Qxd2 isn’t so clear] 17...Bxd2
18.Qxd2 Be4 19.Be2 Qg5 20.Rd1 and f2-f3 is unavoidable.
B) Going back with 16...Be4 was the lesser evil: 17.Nxe4 dxe4 18.Bxe4 Qxd4 19.Qxd4 Nxd4
20.Kg2².

17.gxf5

That’s just a piece!

342
17...Kh8

17...Nxf5 18.Bg4+–

18.f6 Bb4 19.Bg2 Rg8 20.Ndf3 Nc6 21.Nd3 Qb6 22.Kh1 Bc5 23.Qc1

Position after: 23.Qc1

What a disillusionment! 1-0

From a spectator’s point of view, it is difficult to explain how Topalov could make so many mistakes in
a row, but what I believe is that over the board he didn’t like his position and thought he must sacrifice
something – just underestimating his chances after some normal moves. Sometimes your feeling during
a game when you are playing move by move is totally different from the one you have when you look
with an engine!

343
Just after playing the risky move 6...Nh6!?. (photo Anastazia Karlovich)
After this disaster we were thankfully spared sniping from the Topalov accusers, but only for a day! By
then they had managed to concoct a new theory, that Veselin had bet against himself in round 10, as he
knew he would win the day after and qualify anyway! A really sad day for researchers trying to find a
link between playing chess and intelligence.
Anyway, for people living in the normal world, the worry was to calculate what the situation was. With
a win in the last round, Vesko would win the GP series, but how much risk should he take? What would
his chances be if he drew the game? Would the war be over if he lost? Silvio Danailov asked me to do
some calculations and I sent him the following email:
Situation with the current (live) ranking:

Mamedyarov 390
Topalov 375
Grischuk 315 (one GP left to play, maximum 400)
Morozevich 315
Caruana 305 (one GP left to play, maximum 395)
Karjakin 255 (one GP left to play, maximum 375)
Ponomariov 345 (one GP left to play, maximum 355)

344
Dominguez 225 (one GP left to play, maximum 375)
Nakamura 215 (one GP left to play, maximum 370)

Situation if Vesko loses and other games are drawn:

Mamedyarov 390
Topalov 350 (places 8-10)
Morozevich 340
Caruana 305 (one GP left to play, maximum 395)
Grischuk 300 (one GP left to play, maximum 385)
Karjakin 260 (one GP left to play, maximum 380)
Ponomariov 235 (one GP left to play, maximum 355)
Dominguez 225 (one GP left to play, maximum 375)
Nakamura 215 (one GP left to play, maximum 370)

But if Mamedyarov loses, suddenly Morozevich would be ahead of Vesko too!


And Mamedyarov still ahead too. Losing would most probably eliminate Vesko.
The ‘situation with the current standings’ was a good approximation of what the standings would be in
the event of Vesko drawing. As you can see, in that case he could still hope for second place (remember
the top two qualify), but with three to six people having the possibility of overtaking him! Highly
unpleasant.
However, should he lose, although it would not be over theoretically, his chances of qualifying would
be close to zero. In other words, Vesko had to play for a win, but not do anything crazy if things went
wrong.
Topalov, Veselin (2767)
Morozevich, Alexander (2736)
Beijing 2013

1.e4 d6 2.d4 Nf6 3.Nc3 e5 4.Nf3 Nbd7 5.Bc4 Be7 6.0-0 0-0 7.a4 c6 8.Re1 a5 9.h3 Nb6 10.Bb3
Nfd7 11.Be3 exd4 12.Nxd4 Nc5 13.Qf3 Nxb3 14.cxb3

345
Position after: 14.cxb3

The opening has definitely gone well, and White has a very pleasant position with easy moves to play.

14...Be6?!

14...Nd7 was a better move.

15.Rad1 Nd7 16.Qg3 Ne5 17.Re2!?

A very clever and simple idea: White wants to push f4 and avoid losing an exchange after ...Bh4.
However, the engine actually thinks 17.Nf5 Bxf5 18.exf5 is even stronger: White wants to play Ne4,
and possibly f6, and Black never manages to liberate himself with ...d5.

17...Ng6 18.f4

346
Position after: 18.f4

18...c5?!

Too weakening.
18...Bd7 was better.

19.Nf5!?

An interesting pawn sacrifice, although after 19.Nxe6 fxe6 20.Qg4 Qc8 21.Nb5 the engine thinks
White is close to winning.

19...Bxb3 20.Rdd2 Rc8

Defending the c5-pawn.

21.Rf2!!

347
Position after: 21.Rf2!!

Probably the most amazing move of the game. The obvious idea is to threaten Nxe7 and f5, followed
by f6. But there is another hidden idea: to push h3-h4 and make ...Bxh4 fail, since White will trap the
knight with f5 and g3 at the end (see the 22...Bxh4 alternative in the game). Thanks to this subtle move,
White is just winning!

21...Kh8

21...b6? 22.Nxe7+ Qxe7 [22...Nxe7 23.f5+–] 23.f5 Ne5 24.f6+–

22.h4!

22.Nxd6 Bxd6 23.Nb5 was also crushing: 23...Rc6 [23...Bxf4 24.Bxf4 and the b3-bishop is hanging]
24.e5+–.

22...Rg8

22...Bxh4 23.Nxh4 Qxh4 24.Qxh4 Nxh4 25.f5

348
Position after: 25.f5

and the h4-knight is trapped. Black should react with 25...d5, but after 26.Nxd5 Rfe8 27.Nc3 the
problem of the black knight remains.

23.h5 Nf8 24.Nxd6 Bxd6

Position after: 24...Bxd6

25.e5

349
Logical, although it seems 25.Nb5 first was even stronger. After 25...Rc6 26.e5 White is losing his h4-
pawn, but will get a powerful knight on d6.

25...f5?!

This move doesn’t help Black’s case, but his position was quite lost anyway.

26.Rxd6 Qe8 27.Qh3 Be6 28.Rfd2 h6 29.Nb5 Nd7 30.Qf3 b6 31.Qd1 Nf8 32.Rxb6 Qe7 33.Qf3 Nd7
34.Rb7 Rb8 35.Ra7 Rgd8 36.Rd6 Qf7 37.Nc7 Bc4 38.e6 Qe7 39.Rxd7 Rxd7 40.exd7 Qxd7 41.Nb5
Qe6 42.Qg3

Position after: 42.Qg3

Black resigned. 1-0

A really good game by Veselin, and what a relief!


Phew! Topalov had just beaten Morozevich 3-0 in the Grand Prix series. According to Veselin, the
Russian player didn’t shake his hand at the end of this game, a sad situation with such a fantastic player
as Morozevich.
But anyway, the good work had paid off: Topi had won the Grand Prix series, on a knife-edge! When I
got home, I saw that a significant, non-negotiated bonus had reached my bank account. When I asked
Vesko about it, he smiled and said, “You told me you just bought a powerful new computer, so
consider it my present!”.
Next step: the Candidates tournament.

350
351
Final ranking of the Grand Prix: Topalov took first place and qualified for the Candidates, where he
was joined by Mamedyarov, who won the last leg in Paris. Remember, for players who competed in
four Grand Prix, the worst result didn’t count, which is why you’ll find it in parentheses.

When writing this book, I realized how impressive Alexander Grischuk had actually been, with a worst
result of 75 points!

352
Chapter 8
Preparing for the Candidates

An interesting training match


Shortly after the end of the Grand Prix series we started making plans to prepare for the Candidates
tournament. The first thing we did was to think of a top GM who would provide additional help. We
were very pleased that our first choice accepted. Alas, I can’t reveal his name in this book; for very
strong GMs, discretion is one of the keys to success...
Veselin and I started the preparation ourselves, as our top GM was hired to help from about two months
prior the event, which was taking place in March. Vesko asked me to join him for his friendly match
with Viktor Laznicka, taking place in Novy Bor in September.
It was an excellent opportunity to meet and discuss plans, in a city other than the beautiful Salamanca
for a change! We had a wonderful time, and the way the organizers welcomed us was just awesome.
We not only received wonderful treatment, but also had the opportunity to visit the Ajero glass factory,
one of the most beautiful in Europe, and to try to make something out of glass ourselves. To be honest,
we were not too talented at it. Actually, whenever I have a glass of wine at home, I use a glass from
Novy Bor, but not one I made myself!

353
Laznicka vs Topalov and Romanishin vs Kriebel (photo Vladimír Jagr)
Of course Veselin was taking the match with Viktor very seriously. In the morning we would discuss
the coming game, then initial plans for the Candidates so I could start investigating lines in the
afternoon when he was playing. The match was a tough fight which Vesko ended up winning 4-2. As
you will see from the two games presented in this chapter, things could easily have gone the other way,
although at critical moments Veselin had been more precise than his opponent.
Rounds 1 and 2 were two decisive results (both Vesko and Viktor won with White), but the big
theoretical debate of the match happened in game 3.
Topalov, Veselin (2769)
Laznicka, Viktor (2677)
Novy Bor 2013

1.d4 Nf6 2.c4 e6 3.Nf3 d5 4.Nc3 dxc4 5.e4 Bb4 6.Bxc4 Nxe4 7.0-0 Nxc3 8.bxc3

Position after: 8.bxc3

A typical position where White has sacrificed a pawn but gets a huge space advantage. Black’s position
is not too pleasant.

8...Be7

Taking another pawn is too much: 8...Bxc3?! 9.Rb1 and for example if 9...0-0 then 10.Qd3 followed by
Ng5 is terrible for Black. That’s why he needs his bishop on e7!

9.Ne5

354
Since we felt the idea Veselin used in Game 1 (9.Qc2 0-0 10.Bf4) was not good enough, even though
he won, I decided to investigate the main line with 9.Ne5 in depth to try to find new ideas there for
White.

9...0-0 10.Qg4 Nc6 11.Re1

Position after: 11.Re1

11...f5!

Nowadays, it is well-known that this and the coming moves are critical.
We considered 11...Nxe5 inferior: 12.dxe5 Kh8 13.Qe4! followed by Bd3, and Black’s position looks
very dangerous. [13.Re3!? is also interesting.]

12.Qf3 Nxe5 13.Rxe5 Kh8

355
Position after: 13...Kh8

14.Bf4!

Also critical.
Practice has shown that 14.Re1 doesn’t pose serious problems: 14...c5! 15.Bf4 cxd4 16.Rad1 Bf6
17.cxd4 Re8

Position after: 17...Re8

356
18.Re3 [Later 18.h3 was tried by a promising young German GM, but also led to nothing: 18...a6
19.Be5 Bxe5 20.Rxe5 Qc7 21.Qe2 b5 22.Bxe6 Bxe6 23.Rxe6 Rxe6 24.Qxe6 Qc2 25.Re1 h6= ½-½
(32) Svane, R (2587) – Praggnanandhaa, R (2509) Tarvisio 2017] 18...Bd7 19.Qh5 g6 20.Qe2 Ba4
21.Rd2 Rc8 22.h3. All this was played in Ding, L (2702) – Meier, G (2646) Saint Louis 2012, 1-0 (46),
but now for example after 22...Kg7!?∞ Black should be fine.

14...g5!?

Also critical according to the engine.


A) However, given how difficult the resulting position is for Black to handle, recently 14...Bd6 has
been preferred: 15.Re2 Bxf4 16.Qxf4 Qd6

Position after: 16...Qd6

17.Re5!? [17.Qxd6 cxd6 18.Bxe6 Bxe6 19.Rxe6 is slightly better for White, but should be a draw, for
example after 19...Rfc8 as in Salomon, J (2441) – Getz, N (2447) Oslo 2016, ½-½ (80)] 17...Bd7
18.Rae1 with compensation for the pawn, although it is not easy to get more than a draw. Still,
White’s position remains easier, for example: 18...Rf6 19.h4

357
Position after: 19.h4

19...h6?! [Playing 19...b5 20.Bb3 a5 right away looked more to the point.] 20.Bb3 a5 21.Qe3 b5
22.d5! a4 23.dxe6 axb3 24.exd7 Qxd7 25.axb3 f4 26.Qf3² and Black was slightly worse in Karjakin,
S (2763) – Mamedyarov, S (2809) Moscow 2018, ½-½ (37) .
B) 14...Bf6 15.Re2 followed by Rae1 would be a success for White: there is no ...c5 here any more!

15.Bd2 Bd6

Position after: 15...Bd6

358
16.Rae1!

An excellent sacrifice once Black has weakened his kingside with ...g5. Of course, the engine doesn’t
think White is better but, even so, in practice Black’s position remains ugly.
16.Re2 g4 17.Qd3 Qh4 18.g3 Qh3 with ...f4 ideas, is quite unclear.

16...Bxe5 17.Rxe5 Qf6

The logical follow-up.

18.h4

The text move ended up being my favourite choice, but 18.Bb3!?, with the idea of c4 and Bc3, was
definitely a decent alternative.

18...gxh4 19.Bf4

Position after: 19.Bf4

The bishop is back to its ideal square.

19...Qg6!

The best move, as White was about to play Rc5, threatening both Rxc7 and Be5.

20.Qh3

The tempting and direct 20.d5 doesn’t bring White any advantage: 20...Bd7! [20...exd5? 21.Qxd5 just

359
wins for White, as the f4-bishop is soon coming to e5, and the black king is trapped] 21.dxe6 Bc6
22.Qh3 Qg4™

Position after: 22...Qg4™

23.Qxg4 fxg4 24.Be3 Kg7! and Black holds. Of course, it seems like he has had to find many only
moves, but, believe it or not, it is quite likely that Viktor also had this line in his computer, as he
entered all this mess willingly!

20...Rg8 21.Re3

360
Position after: 21.Re3

Making room for the bishop on e5. Another idea is to play Qh2/Rh3/Rxh4.

21...h6 22.Qh2!?

That was our try for this game.


If this position arose again over the board, we would consider the prophylactic 22.Bf1!? (threatening to
take on h4) 22...Qh5 and now two ideas are possible, both of them posing serious practical problems:

Position after: 22...Qh5

23.Be2!? [23.Qh2 Bd7 24.Rh3 Rg4™ 25.f3 Rxf4 26.Qxf4 and Black should save with 26...Qg5!
27.Qxc7 Qe3+ 28.Kh2 Bc6 29.Rxh4 Rg8 30.Qe5+ Qxe5+ 31.dxe5 Kg7 32.Bc4 Kg6=] 23...Rg4!
[23...Qg6 24.Bf3 looks great for White] 24.Bxg4 fxg4 25.Qh1!

361
Position after: 25.Qh1!

and with this amazing move, White wants to play g2-g3 and target the b7-pawn should the c8-bishop
move. Actually, I’m not sure Black can equalize here!

22...b5?

Just helping White, who is quite happy to retreat his bishop to f1.
22...Kh7! 23.Rh3 Qf6! 24.Rxh4 Rg6

362
Position after: 24...Rg6

was the only way for Black to hold, according to my analysis!

23.Bf1 Kh7 24.Qxh4

Position after: 24.Qxh4

There is no longer any need to play Rh3, as g2 is protected! White also has the deadly threat of Rg3.

24...e5™

The only move not to lose at once.

25.dxe5?

A bad decision by Veselin, probably thinking he was winning by force, but now Black had to find two
only moves in a row, after which his problems were nearly solved.
25.Rg3! would have given White a decisive advantage. 25...Qe6 [25...Qd6 26.dxe5 Qe6 27.Rxg8 Kxg8
28.Qd8+] And here, while a simple exchange on g8 followed by Bxe5 would be good, the engine
comes up with a +5 line for White: 26.d5! Qd6 27.Rxg8 Kxg8 28.Bxh6

363
Position after: 28.Bxh6

28...Qg6 [28...Kf7 29.Be2! Qxd5 30.Qg5! somehow wins] 29.Bxb5 Bb7 30.Bc4 Qg4

Position after: 30...Qg4

31.Qf6 [31.Qe7 would be the human way] 31...Kh7 32.Bg5 Qd1+ 33.Bf1 Qh5 34.d6 cxd6 35.Qe7+
Kg6 36.Bf6+–. Of course, such a line is impossible to calculate. I just give it for pleasure – but of
course, 26.Bxe5 (instead of 26.d5) would have been a huge improvement compared to the game, with a
nearly winning position anyway.

364
25...Qe6™ 26.Bd3 Rg6™

Position after: 26...Rg6™

The only defensive set-up - but difficult to crack.

27.Rg3?

After this move, White ends up worse.


The only way to fight for the initiative was however pretty sophisticated: 27.Rh3! Kg7 [27...Bb7?
28.Bxh6! Rxh6 29.Bxf5++–] 28.Rf3

365
Position after: 28.Rf3

and here Black’s position still isn’t easy, as both natural moves lose by force: 28...Bb7? [28...Kh7?
29.Bxh6! Rxh6 30.Bxf5+ Kh8 31.Qg5+–] 29.Bxf5! Qxf5 30.Bxh6+ Rxh6 31.Qe7++–.

27...Rxg3 28.fxg3 Bb7!

Position after: 28...Bb7!

29.Bc2?!

366
Veselin wants to be able to answer Bb3 to ...Be4, but his position is getting worse and worse.
29.g4 Be4 30.gxf5 Bxf5 31.Bxf5+ [31.Bxb5 should be OK as well.] 31...Qxf5 32.Qxh6+ Kg8 33.Bg5!
and White has compensation.

29...Rf8?!

29...Re8! was a huge improvement on the game. The only minus point is that it doesn’t add extra
protection to the f5-pawn, but we already understood that there were no tactics to be afraid of: 30.g4
Be4 31.gxf5 Bxf5 32.Bxf5+ Qxf5 33.Qxh6+ Kg8 and especially with the black rook on e8, White isn’t
in time to play Bg5-f6, and thus should be lost.

30.g4?!

After 30.Bb3! White would have had sufficient compensation, but it was necessary to realize that after
30...Qc6 [30...Bd5? 31.Bxd5 Qxd5 32.Qxh6+ Kg8 33.Qg6+ Kh8 34.e6+–; 30...Qb6+ 31.Kh2©]
31.Qe7+ Kg6

Position after: 31...Kg6

White could just escape the mating threat with 32.Kf2!, and he’s winning: 32...Qxg2+ 33.Ke3 Qe4+
34.Kd2 Qg2+ 35.Kc1 Qf1+ 36.Kb2 Qe2+ 37.Ka3 b4+ 38.Kxb4!+–.

30...Qg6!?

30...Be4 31.Bb3 Qg6 was good as well.

31.e6

367
Position after: 31.e6

31...Qxe6?

Letting it slip – phew!


31...Be4! was just bad for White.

32.Bxf5+ Rxf5 33.gxf5 Qxf5 34.Qxh6+ Kg8 35.Qg5+ Qxg5

½-½
Given the depth of each player’s preparation in Game 3, it was clear that this line had become the
debate of the match. Viktor was being prepared by Jiri Dufek, who is ‘only’ an FM, but is a computer
and chess engine specialist.

368
Game 3 (photo Vladimír Jagr)
My computer was running on this line for several hours each day, as, I suspect, was Jiri’s! Jiri is also
famous for his collection of correspondence games, and since the match he’s been generously sending
me an updated database of them every year!
In the next game, Viktor played excellently, and it even looked like he might take the lead in the match.
However, he ran into terrible time trouble and Vesko converted a ‘suspicious mess’ into a full point.
The game is definitely worth a look: you won’t believe your eyes when you see some of the possible
lines, especially if White had played 32.h4.
Laznicka, Viktor (2677)
Topalov, Veselin (2769)
Novy Bor 2013

369
Black to move

White is obviously better as he has a clear extra pawn. To get chances, Black should open the position.
The same rule applies here as in endgames: in opposite-colored bishop positions, activity is more
important than material.

26...e4 27.Rd6 Re6™

Not the greatest move to have to play by a long shot, but Black can’t allow c5-c6.

28.Rxe6 fxe6 29.Qc2 Bc6

370
Position after: 29...Bc6

30.Rd1?!

The start of a bad plan. White wants to play Rd6 and force events, but it is too risky, especially with
little time on clock, as was the case.
A) 30.fxe4 Bxe4 31.Qd2 was enough for a nearly decisive advantage. White will play h2-h3 next and
it’s difficult to see Black’s counterplay.
B) The concrete 30.Re1 was the other option, just going for the e4-pawn: 30...Rf8 31.fxe4 Rf4 (the
only way to fight on) 32.a6

371
Position after: 32.a6

32...Rxe4™ [32...Bxe4? loses nicely to 33.Rxe4! Rxe4 34.Qb1! Qf5™ 35.a7 Qd5 36.Qa1! Qa8 37.h3
and Black is paralyzed, if 37...Re2 then 38.c6 Rc2 39.Qe5!+– …Qb8] 33.Rxe4 Qxe4 34.Qxe4 Bxe4
35.Kf2

Position after: 35.Kf2

Now the only question is whether this endgame is winning or not. It actually wins by force, meaning
30.Re1 was surely the best move: 35...Kf7 36.g4 e5 [Passive defence is no use, as after 36...Ke7

372
37.Ke3 Bc6 38.Kd4 Kd7 39.Ke5 the b6-bishop will soon manoeuvre to attack to g7-pawn; 36...g6
37.Ke3 Bc6 38.Kd4 and Ke5 next is also too easy.] 37.Ke3 Bc6 38.h4! (avoiding any kind of
counterplay on the kingside, and preparing to answer ...g7-g6 with h4-h5, fixing the black pawn on
h6)

Position after: 38.h4!

38...g6 [38...Ke6 is even worse: 39.Bc7 Kd5 40.Bd6 and the bishop reaches the f8-square] 39.h5!
gxh5 40.gxh5 Kf6.

Position after: 40...Kf6

373
And now White can win an important tempo in the race: 41.Bd8+! [41.Kd2 Kg5 is a draw] 41...Kf5
42.Kd2 e4 [42...Kg4 43.Kc3+–] 43.Ke3 and it’s a piece of cake for White, who will attack the h6-
pawn with his bishop. If 43...Kg4 then 44.Kd4 Kf3 45.Ke5 e3 46.Kd6 e2 47.Ba5+–.
C) 30.f4!? was a third good possibility. If White manages h3 and Kh2, Black can resign. However,
after 30...Qf5 (…...e5) 31.Qc3 Rf8 Black will get serious practical chances, as ...g5 is coming soon.

30...Rf8!

Position after: 30...Rf8!

After this move, the position is already unclear, as it matters a lot who owns the f-file.

31.Rd6?

Entering complications that do not work for White.


31.fxe4 was better, and the arising position after 31...Bxe4 32.Qb2 Bd5 is just unclear: the d5-bishop is
a monster, and Black has several ideas to create counterplay, like ...Qe4 (threatening ...Qe3+ and
...Rf2), ...Rf4/f5 to move to the g-file, etc.

31...Qg5!

Of course, Black doesn’t move his bishop.


31...Qf6!? was the other good move, though not as strong: 32.Qe2™ [32.Rxc6 exf3 33.gxf3 Qxf3 just
wins for Black] 32...exf3 33.gxf3 Bd5

374
Position after: 33...Bd5

and White’s position is extremely unpleasant. For example: 34.c6 Qg5+ 35.Qg2 Qf4! 36.Rxd5™
36...exd5 37.c7 Qc1+ 38.Qf1 Qxf1+ 39.Kxf1 Kf7 and the black king is in time. It is very unclear
whether White can save himself here or not.

32.Rxc6?

Just losing.
32.h4™ was the only move, but it probably loses.

375
Position after: 32.h4™

32...Qf6! On the one hand, with the extra move h2-h4 (compared to 31...Qf6), White has made room
for his king. On the other, the pawn is just hanging. [32...Qe3+ 33.Qf2 Qc1+ 34.Qf1=; 32...Qxh4
33.Qf2 Qg5 34.Rxc6 Qc1+ and White can repeat moves with 35.Qf1, while 35.Kh2 Rf5! seems very
dangerous for him.] 33.fxe4 [33.Qe2 exf3 34.gxf3 Bd5 is just a bad version of the 31...Qf6 line for
White, as he’s going to lose the h4-pawn.] 33...Qf1+ 34.Kh2 Rf4!!

Position after: 34...Rf4!!

and Black is winning! White has two ways to defend himself, and the wins are superb: 35.Qd2 [35.Bd8
trying to defend the h4-pawn doesn’t help because of 35...g5!!–+] 35...Bb5!! Black has the devastating
threat of ...Be2. 36.Qe3 [36.c6 Be2! Threatening ...Rxh4+, followed by ...Rg4+ and a mate on g2. 37.g3
Rf2+! 38.Bxf2 Qxf2+ 39.Kh3 Bf1+ 40.Kg4

376
Position after: 40.Kg4

40...h5+! 41.Kxh5 Qf3+ 42.Kg5 Qxg3+ 43.Kh5 Kh7! and White needs to play 44.Qh6+ to prevent
...g6#/...Qg6#.] 36...Rxh4+ 37.Kg3

Position after: 37.Kg3

37...Be2! The prettiest, threatening ...Rg4+. [37...g5 with the idea ...Qh1 also wins.] 38.Kxh4 Qxg2–+
and Black threatens several mates (...Qg4/ ...g5/ ...Qh2). White needs to sac his queen on e2 and he’s
lost.

377
Back to the game (32.Rxc6).

Position after: 32.Rxc6?

32...exf3 33.g3 Qe5!

A) 33...f2+?! was the obvious way, and after 34.Qxf2 Rxf2 35.Kxf2 Qd2+ 36.Kf1™ 36...Qxh2
37.Rd6 Qxg3 Black is winning. However, the game move is stronger!
B) 33...Qf6! was as good, according to the engine; but better to have an extra threat in ...Qe1+!

34.Qf2

A) 34.Kf1 doesn’t help: 34...Qa1+ 35.Kf2 Qh1–+.


B) Nor does 34.Kf2 Qd4+–+.

34...Qa1+ 35.Qf1 Qd4+ 36.Qf2 Qd1+ 37.Qf1 f2+

378
Position after: 37...f2+

White resigned in view of 37...f2+ 38.Kg2 Qd5+–+.

0-1

As mentioned in the analysis of game 3: we had a deep improvement prepared for the following white
game (at move 22!). Unfortunately, Viktor didn’t repeat the opening, and opted for the Slav Defense.
Veselin pushed in a slightly better endgame, but was unable to convert.
In the last game, Viktor absolutely needed to win with the white pieces. As often happens in that kind
of situation, the aggressive play required led to a loss, so in the end 4-2 for Veselin!
We were obviously happy with the result, and had already made some initial plans for the Candidates,
not just about the openings themselves – we had also decided to organize a 10-day training camp in
Lanzarote in December.

379
At the closing ceremony. (photo Vladimír Jagr)
Training plans
We had a very productive training session in Lanzarote, and such a wonderful time, that we decided to
make it our training base. Veselin booked a house there for the two months prior to the Candidates, and
came with his wife and daughter. Besides fruitful chess work, Veselin’s idea was to get as much sun as
possible before flying to Khanty-Mansiysk, where he definitely wouldn’t get any! We had great
training sessions, found many new ideas, played interesting training games, and garnered some energy
thanks to long walks and the beach – things couldn’t have gone better! I would happily reveal some of
our opening ideas here, but actually our best ones were used in the tournament, so you’ll soon discover
them in the book!

380
381
382
LANZAROTE
Visa denied!
After the first training session in Lanzarote, we booked flights to Khanty for the whole team (minus our
extra GM helper, who was going to help from home). We decided to fly directly from Lanzarote in
March, first to Madrid, then Paris, Moscow and Khanty. Everything was going smoothly and after
receiving the letter of invitation from the organizers, I went to Paris to get my visa. It was denied! “The
organizers made an obvious mistake in the invitation letter”, the Russian Embassy in Paris said.
They refused to explain any more, and, to cut a long story short, it was getting too tricky for me to wait
for a new letter and go to Paris again, so we decided I would also help Veselin from home. It was a
disappointment, not only for Veselin, who always preferred to discuss concrete plans at breakfast,
rather than have to do everything through computers, but also for me, not being able to watch a
Candidates tournament live for the first time. It was good that Silvio Danailov was able to join Veselin
a few days later, as it would have been no fun for him to be on his own in deepest Russia for three
weeks! There was one upside: the time difference between Khanty-Mansiysk and France allowed me to
work at more normal times than if I had been on site!

383
And so when our training sessions in Lanzarote came to an end, I finished up using the Madrid-Paris-
Moscow and Khanty-Moscow-Paris tickets, but only to travel from Madrid to Paris – probably the most
expensive two-hour flight ever! And it was economy class... After the tournament, the organizers told
Silvio that I had been silly, as I hadn’t applied for the correct visa, which wasn’t true, and Vesko had no
doubt about it. However it gave birth to a nice joke, Veselin then telling me several times, “You’re so
smart! You found a way to escape Khanty!”.
Joking aside, Veselin flew to Khanty-Mansiysk on his own a few days prior to the event, and said that
the organization was perfect. He arrived there full of chess ideas and ready to win. The only things
missing were sun and... his luggage, which the airline had lost somewhere in Moscow. Anyway, it was
now time to focus on the games.

384
Chapter 9
The Candidates Tournament

As the Candidates Tournament was the completion of all our work, we’re going to do a round by round
analysis. The most interesting games are deeply analyzed and explained, while others are shown in
brief.
Game 1: Mamedyarov (Black)
Mamedyarov, Shakhriyar (2757)
Topalov, Veselin (2785)
Khanty-Mansiysk 2014

White to move

This was a rather boring game.


After the above position was reached on move 14, Veselin made a few imprecise moves, but the
position was too drawish and the point was halved on move 36.
½-½
Game 2: Anand (White)
Topalov, Veselin (2785)
Anand, Viswanathan (2770)
Khanty-Mansiysk 2014

1.Nf3 d5 2.c4 c6 3.e3 Nf6 4.Nc3

385
Position after: 4.Nc3

4...Nbd7

This rarely-played move came as an annoying surprise (Black usually plays ...e6 first). Black’s idea is
to play ...e5, when 5.d4 transposes into a Semi-Meran which White basically wanted to avoid by
starting with 1.Nf3 and 2.c4.
We had focused on 4...e6, where we had found some incredible ideas, so I was very sad when this line
didn’t appear on the board. See why... 5.b3 Nbd7 [We considered 5...Bd6 a safer move.] 6.Qc2 Bd6
7.Bb2 0-0 8.Be2!? b6

386
Position after: 8...b6

9.Rg1! An almost new idea then. [We couldn’t make an immediate g2-g4 work, but another decent try
we had - and which hasn’t been played by anyone yet – was 9.h4!? Bb7 10.g4!? Nxg4 11.Ng5 Ngf6
12.0-0-0 with compensation.] 9...Bb7 10.g4
A) A typical defensive idea is 10...Qe7!? 11.h4N 11...Ba3, but we quite liked White: 12.g5 Ne8
13.Bxa3 Qxa3

Position after: 13...Qxa3

14.h5!? [14.cxd5!? cxd5 15.Bd3 g6 16.Bb5 Qe7 17.Bxd7 Qxd7 18.Qd3 is the simple solution.
White’s position looks more pleasant, as his f3-knight is coming to g4.] 14...Qe7 15.0-0-0!? e5 16.d4
e4 17.Nd2 f6 18.gxf6 Ndxf6 19.Rde1 b5 20.f3 with a huge mess.
B) 10...e5 11.g5 Ne8 12.h4

387
Position after: 12.h4

I don’t have to feel guilty for showing you what our deep idea was, since it was all played a year later
by the Dutch prodigy Anish Giri! 12...Nc7 13.Bd3! A crucial move, based on a piece sacrifice.
Black’s position would be fine otherwise. When Anish played it (right in front of my eyes, as he was
playing for my team in the French league!) I thought “Damn, this game could have been Veselin’s!”.
13...g6™ [13...e4? 14.Nxe4! dxe4 15.Bxe4 with a huge attack;

Position after: 15.Bxe4

388
...g6 is always answered by Qc3.] 14.h5 All this was our main line, and only here did Giri’s game
deviate! He won an extremely impressive game, which we’ll have a look at.

Position after: 14.h5

B1) 14...Qe7 was the move Veselin and I had considered critical, and here we wanted to try
15.Nh2!?, with the idea to always answer ...e4 with Ng4. A sample line from our analysis went
15...Na6!? 16.hxg6 fxg6 17.Bxg6! Nb4 18.Qb1

Position after: 18.Qb1

389
18...e4 [18...hxg6 19.Qxg6+ Qg7 20.Qxd6 Nd3+ 21.Kd1 is also a big mess] 19.Nxe4 dxe4 20.Ng4
hxg6 21.Nh6+ Kh7 22.Rh1 Nd3+ 23.Qxd3 exd3 24.Ng4+ Kg8 25.Nh6+ with a draw! Obviously,
making a draw wasn’t the goal from the start, but Black had many opportunities to blunder!
B2) 14...Nc5 15.Be2 White can’t allow Black to exchange on d3. 15...e4

Position after: 15...e4

16.Nxe4!! That’s the spirit! 16...Nxe4 [16...dxe4 17.Qc3 f6 18.hxg6 hxg6 19.gxf6 is crushing] 17.d3
Bb4+ 18.Kf1 Bc8? [18...Nxf2! was required, and if Black is able to find 19.Kxf2 Re8! 20.Ne5 d4!
21.Bxd4 Ne6

390
Position after: 21...Ne6

it seems the position is a forced draw: 22.Bc3 Nxg5 23.Ng4 Nh3+ 24.Kf1 Nxg1 25.Nf6+ Kf8
26.Nxh7+ Kg8 27.Nf6+ Kf8 28.Nh7+=. A fun line, isn’t it?] 19.dxe4!

Position after: 19.dxe4!

19...Bh3+ 20.Rg2 with a great advantage for White: Giri, A (2776) – Michiels, B (2536) France
2015, 1-0 (39).
As you can understand, by playing 4...Nbd7, Vishy Anand spoiled the whole plan!

5.Qc2 e5 6.cxd5 Nxd5

391
Position after: 6...Nxd5

7.d4

Critical, but Anand had simply outprepared us.


7.Bc4 is the other option, with a complicated game.

7...Bd6! 8.dxe5 Nxe5 9.Nxe5 Bxe5 10.Qe4

White wins a pawn, but isn’t better.

10...Qe7! 11.Nxd5 cxd5 12.Bb5+ Kf8 13.Qxd5 g6

392
Position after: 13...g6

A nice idea. It’s amazing that, a pawn down and with his king on f8, Black isn’t worse!

14.Bd2

A) 14.0-0 Kg7 followed by ...Be6 or ...Rd8 is good for Black.


B) 14.f4 Bf6! doesn’t help. [And not 14...Bxf4? 15.0-0! when Black can even be a pawn up by taking
on e3, but the initiative is clearly on White’s side!]

14...Kg7 15.Qxe5+ Qxe5 16.Bc3 Qxc3+ 17.bxc3 Be6

Black has typical compensation: he has great coordination and the c3-pawn is an annoying weakness.

393
Position after: 17...Be6

18.Ke2 Rac8 19.Rhc1 Rc5 20.a4 Rhc8 21.Ra3 a6 22.Bd3 b5

Anand decided to enter a 3 v 4 ending, which is a draw without too much suffering. However,
according to the engine, Black could just have waited and White had zero advantage.

23.axb5 axb5 24.Rb1 Rxc3 25.Rxc3 Rxc3 26.Rxb5 Bc4 27.Bxc4 Rxc4

Position after: 27...Rxc4

394
Veselin played on for another twenty-seven moves, but there’s not much to be done if Black defends
accurately. Sometimes your opponent springs a surprise on you, and you can’t do anything with White
– that’s chess!

28.Kf3 h5 29.h3 Rc2 30.Rb1 Kf6 31.Re1 g5 32.Ra1 Kg6 33.Ra6+ f6 34.Ra4 h4 35.g3 hxg3 36.Kxg3
Rb2 37.e4 Rb1 38.f3 Rg1+ 39.Kf2 Rh1 40.Kg2 Rb1 41.Ra6 Kf7 42.Ra5 Kg6 43.Ra6 Kf7 44.Ra2 Ke6
45.Kg3 Rg1+ 46.Rg2 Rxg2+ 47.Kxg2 Ke5 48.Kf2 Kf4 49.Kg2 Ke5 50.Kg3 f5 51.exf5 Kxf5 52.h4
gxh4+ 53.Kxh4 Kf4 54.Kh3 Kxf3 ½-½

Game 3: Aronian (White)


Topalov, Veselin (2785)
Aronian, Levon (2830)
Khanty-Mansiysk 2014

1.e4 e5 2.Nf3 Nc6 3.Bb5 a6 4.Ba4 Nf6 5.0-0 Be7 6.Re1 b5 7.Bb3 0-0 8.a4

Position after: 8.a4

The Anti-Marshall, which has gained massive popularity in recent years.


8.h3 d5 would be Levon’s beloved Marshall Gambit.

8...b4 9.d4 d6 10.dxe5!? dxe5

Critical, but 10...Nxe5!? is a very reliable alternative.

11.Nbd2 Bc5

395
The e7-bishop takes advantage of the exchange on e5 to go to a more active square.

12.h3

Position after: 12.h3

Almost a new idea at the time of the game, as it had been played only twice, by GM Jansa in 1978 and
1996! Since this game by Topalov, it’s been repeated many times. The idea is to play Qe2 and Nc4,
and prevent any ...Ng4 or ...Bg4.
In case of 12.Qe2, top level games have gone 12...Qe7 13.h3 and Black managed to get his queen on
e7, so he can play ...Rd8 (preventing Bd5) or ...Na5 (since the e5-pawn is protected). With our move
order, 12...Qe7 isn’t so good, as you will see.

12...Bb7

We considered many alternatives to this move.


A) 12...Na5 13.Qe2! [Recently 13.Ba2 has been played a few times, but I prefer the text move which
we had planned with Veselin.] 13...Qe7 [13...Nxb3?! 14.cxb3² and White will enjoy himself after
Nc4 and Be3.] 14.Nc4!

396
Position after: 14.Nc4!

14...Nxc4 [14...Nxb3?! 15.cxb3 Nd7 16.Be3± ½-½ (39) Heimann, A (2546) – Roiz, M (2600) Skopje
2015] 15.Bxc4 Here we considered White’s position to be more pleasant, for example 15...Bb7
[15...h6 16.Nh4! Be6 17.Nf5 Bxf5 18.exf5²] 16.Bd3 Rfd8 17.b3 a5 18.Be3!?ƒ.
B) 12...Qd6!? The young Indian prodigy Aravindh won a great game with White after this move.
13.Qe2

Position after: 13.Qe2

397
13...h6 [We had focused on 13...Na5 and quite liked White after 14.Nc4 Nxc4 15.Bxc4 Bb7 16.Bd3!?
followed by 17.Be3. The advantage looks minimal, but the f3-knight will end up on c4, and Black has
long-term problems.] 14.Nc4 Qe7 15.Ne3!? [15.Be3 should also lead to a slight advantage for
White.] 15...Na5 16.Ba2 Be6 17.Bxe6 Qxe6 18.Nf5 Rfd8

Position after: 18...Rfd8

19.Bxh6! gxh6 20.Nxh6+ Kg7 21.Ng5!


B1) 21...Qc4?! Tempting, but wrong. 22.Qf3! Rd2 23.Rf1!

398
Position after: 23.Rf1!

23...Qe2 24.Nf5+ Kf8 25.Qg3 White has a deadly threat of Qg3-h4. 25...Nxe4 26.Nh7+ Ke8
27.Qxe5+ Kd7 28.Qxe4! Qxe4 29.Nf6++– 1-0 (56) Aravindh, C (2573) – Narayanan, S (2568)
Patna 2017.
B2) 21...Qb6! was right, keeping the queen on the sixth rank with an eye on the f2-pawn:

Position after: 21...Qb6!

22.Nf5+ [22.Nhxf7?! Rf8 23.Nxe5 Bxf2+! 24.Qxf2 Qxf2+ 25.Kxf2 Nxe4+ 26.Kg1 Nxg5³]
22...Kg8 [22...Kf8!?] 23.Qf3 [23.Nh6+ Kg7=] 23...Rd2 24.Rf1 Rxc2 25.Qg3?! Nh5! 26.Qg4 Qg6!
(the nuance!).
C) 12...Qe7?! 13.Bd5! One of the ideas of playing h2-h3 before Qd1-e2. 13...Bb7 [13...Nxd5 14.exd5
wins the e5-pawn] 14.Nb3!² …Bg5 and White is better.
We analyzed other moves, but the analysis above gives you an idea of the kind of positions we were
seeking. Back to the game!

399
Position after: 12...Bb7

13.Qe2!? Nd4!?

Before White plays Nc4 or Nf1.

14.Nxd4 Bxd4

Now White can’t move the d2-knight that easily, as e4 will be hanging.

15.Bc4!?

15.Qf3 was the other option, and here Black should be alert: 15...Rb8! [15...Kh8 16.Nc4 Nxe4? fails to
17.Rxe4 f5?! 18.Rxd4+–] 16.Nc4 h6 and Black should be fine – the little move ...Rb8 enables Black to
answer Na5 with ...Ba8.

15...a5 16.Bd3

400
Position after: 16.Bd3

16...Nd7!?

16...Qe7 was also possible. After 17.Nf3 the engine comes up with a very funny idea: 17...c5!? (Self-
trapping the d4-bishop. Of course retreating it was more human). 18.Rb1 And while it looks like Black
should be concerned about Bd2 followed by c3, the engine just laughs: 18...h6 19.Bd2 Rfd8

Position after: 19...Rfd8

20.c3 [20.Red1 Qd6! 21.Be1 Bxb2 22.Rxb2 c4³] 20...Bxf2+! 21.Kxf2 Qd6 22.Bc2 b3 23.Bxb3 Nxe4+

401
24.Kg1 [24.Ke3? Qf6–+] 24...Nxd2³.

17.Nf3 Nc5 18.Nxd4 exd4!

18...Nxd3 19.cxd3 Qxd4 20.Be3 Qd6 21.Rac1 Rfd8 22.Red1 is no fun for Black.

19.Bf4 Re8

Position after: 19...Re8

20.Qh5!

Critical.
20.f3 Nxd3 21.cxd3 c5=

20...Qe7!

20...Nxe4 21.f3 is good for White: 21...Qf6 [21...Nf6 22.Qc5±; 21...g6 22.Qh6‚] 22.Bxc7 g6 23.Qg4
Rec8 [23...h5 24.Qf4±] 24.Rxe4 Rxc7 25.Rf4²

21.Bb5!

21.Bc4 Nxe4! 22.f3 Qf6 23.Bxc7 is not problematic for Black as in the line above:

402
Position after: 23.Bxc7

23...Re7! [23...Rec8!? 24.Be5 Qe7= …...g6] 24.Bxa5 Nd2 25.Bxb4 Rxe1+ 26.Rxe1 Nxc4 27.Qb5
Qc6∞.

21...c6 22.Bc4

Position after: 22.Bc4

22...Ba6

403
Objectively good, although I get the impression Levon underestimated White’s next move.
Starting with 22...g6?! would be quite weakening. 23.Qf3 Ba6 24.Ba2! d3 25.Be3! and the dark squares
will be a significant problem for Black: 25...Nxa4? 26.Bb3 Nxb2 27.Bc5!+–.

23.Bd6

23.Ba2 d3 24.Be3?! Nxa4 25.Bb3?! Nxb2 26.Bc5 Qd7 would lead nowhere. In the line above, White
would have Qf3-f6 followed by Bd4 after this sequence.

23...Qxd6 24.Bxf7+ Kf8 25.e5 Qd7!

Position after: 25...Qd7!

26.e6

26.Bxe8 Rxe8 27.Qxh7 Qd5© looks easier for Black.

26...Nxe6 27.Bxe8 Rxe8 28.Qxh7

28.Qxa5?! Bb7 29.Qxb4+ c5 and it’s likely that White will suffer damage on the light squares.

28...Qd5 29.f4!

White needs to react.

404
Position after: 29.f4!

29...d3!

Forcing a draw.

30.f5 d2 31.Qh8+ Kf7

31...Ke7!? was worth a try, but it seems White holds:

Position after: 31...Ke7!?

405
32.Qh4+! [32.Qxg7+?! Kd8 33.Rxe6 d1=Q+ 34.Rxd1 Qxd1+ 35.Kh2 Rxe6 36.fxe6 Qd6+³;
32.Rxe6+? Qxe6 33.Qxg7+ Qf7µ] 32...g5!? [32...Kd7 33.fxe6+ Kc8 34.Red1 Rxe6 is fine for Black,
but he isn’t better] 33.Qxg5+ Kd7

Position after: 33...Kd7

34.Rxe6 [34.fxe6+ Kc8 and both the queen on g5 and the rook on e1 are hanging] 34...d1=Q+ 35.Rxd1
Qxd1+ 36.Kh2 Rxe6 37.fxe6+ Kxe6 38.Qg6+ Kd7 [38...Ke5 39.Qe8+ Kf4 40.Qxc6=] 39.Qf5+ Kd6
40.Qxa5=

32.Qh5+

406
Position after: 32.Qh5+

32...Kf8 33.Qh8+ Kf7 34.Qh5+ Kf8 35.Qh8+

A great game by both players!


½-½
Game 4: Karjakin (Black)
Karjakin, Sergey (2766)
Topalov, Veselin (2785)
Khanty-Mansiysk 2014

407
White to move

Like game 1, there isn’t much to show here. As you can see, the position is just dry equality.

24.R1c3 Re6 25.b4 g6 26.a4 Rd4!

Position after: 26...Rd4!

A nice move, but it doesn’t bring more than a draw:

27.exd4 exd4 28.Qxe6 Qxe6 29.R3c4 Qd6 30.a5 h5 31.h4 Kg7 32.Kg1 Kf8 33.Rc1 Qe6 34.R5c4

408
Qe5 35.Kf1 Qd5 36.Re1 Qh1+ 37.Ke2 Qd5 38.Kf1 Qh1+ 39.Ke2 Qd5 40.Kf1 ½-½

Game 5: Svidler (Black)


Svidler, Peter (2758)
Topalov, Veselin (2785)
Khanty-Mansiysk 2014

1.e4 e5 2.Nf3 Nc6 3.Bb5 a6 4.Ba4 Nf6 5.0-0 Bc5 6.c3 b5 7.Bc2 d5

Position after: 7...d5

8.d4

This move was the trend at the time. We also found some amazing ideas against the other two critical
moves.
Some sample lines:
A) 8.exd5 Qxd5 9.d4 exd4 10.Bb3 Qd6 11.Re1+

409
Position after: 11.Re1+

11...Be6!?N A novelty – and still is today! [11...Kf8 is also a decent move, but can’t be compared
with the ideas we found after 11...Be6!] 12.Ng5 Nd8! So far, the moves seem quite straightforward...
13.Qf3 0-0!!

Position after: 13...0-0!!

14.Qxa8 [14.Bf4 doesn’t help: 14...Qb6 15.Qxa8? dxc3 16.Nxc3 Bxf2+ 17.Kh1 Bxe1 18.Rxe1 Nb7!
19.Rxe6 Qf2!–+] 14...Ng4 15.Nf3 [15.g3 dxc3 16.Nxc3 Bxf2+ 17.Kg2 Bxb3 18.axb3 Bxe1 19.Qe4

410
f5 20.Qxe1 Qc6+ 21.Nf3 Ne6©] 15...dxc3 16.Rd1 Qb6 17.Rxd8 c6!? 18.Rxf8+ [18.Re8!? Bd6
19.Be3 Qxe3 20.fxe3 cxb2 will transpose] 18...Bxf8 19.Be3

Position after: 19.Be3

19...Qxe3!! 20.fxe3 cxb2 21.Bxe6 fxe6 22.Qxc6 bxa1=Q 23.Qxe6+ Kh8 24.Nfd2=
B) 8.a4 dxe4 9.axb5

Position after: 9.axb5

9...exf3! [9...Bg4 doesn’t quite equalize, for example 10.bxc6!N 10...exf3 11.gxf3 Be6 12.Ra5! Qd6

411
13.f4! e4 14.d4 exd3 15.Qxd3² 1-0 (28) Caruana, F (2795) – Onischuk, A (2664) Saint Louis 2016]
10.Qxf3 e4!N This move was new at the time. Unlike the line after 8. exd5, all our new ideas have
been played in recent years. 11.Bxe4 Ne5 12.Qe2 0-0

Position after: 12...0-0

B1) 13.d4 Bg4! 14.Qe1 [14.f3?! Bxd4+ makes Black’s life easy] 14...Nxe4 15.Qxe4 Nf3+! 16.gxf3
Bh3 17.dxc5 [17.Re1?! Bd6 18.Rxa6 Rb8 19.c4? Qf6–+ …...Rbe8: 0-1 (59) Zilka, S (2533) –
Lampert, J (2488) Prague 2016.] 17...Bxf1 18.Kxf1 Qd1+ 19.Qe1 Qxf3© ½-½ (51) Pichot, A (2552)
– Yu Yangyi (2760) chess.com 2018.
B2) 13.Bxa8 Nd3 14.h3 Almost necessary. [14.bxa6? Re8 15.Qf3 Bg4 16.Qg3 Bd6 17.Qh4 Qxa8–
+] 14...Re8

412
Position after: 14...Re8

15.Qd1 [The main line of our analysis went 15.Qf3 Ne5!? 16.Qd1 Qd3 17.b4 Bxh3! 18.bxc5 Rxa8
19.gxh3 Nf3+ 20.Kg2 Nh4+ 21.Kh2 Nf3+ 22.Kh1? Qf5 23.Kg2 Nh4+ 24.Kh2 Qf4+ 25.Kh1
Nf3–+] 15...Bxh3! 16.Bf3 Qd6 17.b4 Bxf2+ 18.Rxf2 Nxf2 19.Kxf2

Position after: 19.Kxf2

19...Qh2! 20.Qf1? [20.Qg1™ was required: 20...Qf4 21.gxh3 Ne4+ 22.Ke2™ 22...Nxd2+ 23.Kd3
Qxf3+ 24.Kc2 Qe4+ 25.Kb2 Nc4+ 26.Kb3 Qe6 27.Kc2 axb5 and although Black’s position seems

413
easier to play, the engine assesses it as equal.] 20...Ne4+ [20...Bg4! was even stronger] 21.Bxe4
Rxe4 22.d3 Bxg2 23.Qxg2 Re2+ 24.Kxe2 Qxg2+ 25.Ke3 axb5–+

Position after: 25...axb5–+

The kingside pawns will promote soon. A win in great style by the German talent Dmitrij Kollars:
Feuerstack, A (2461) – Kollars, D (2410) Germany 2015, 0-1 (43).

8...dxe4

Position after: 8...dxe4

414
9.dxe5

This was known to be critical, with White getting some advantage according to theory.

9...Qxd1!

Also critical.
9...exf3?! Five years earlier, Peter Svidler had shown why this move isn’t good. 10.exf6 Qxf6?!
[10...Qxd1N might be the least damage, still, after 11.Bxd1! fxg2 12.Re1+ Kf8 13.a4, Black’s position
is unpleasant: 13...b4 14.Bf3 Bb7 15.Nd2²] 11.Nd2!

Position after: 11.Nd2!

11...0-0 [11...fxg2? 12.Re1+ Be7 13.Ne4 is crushing for White, for example 13...Qg6 14.Ng3 Qd6
15.Qxd6 cxd6 16.Be4 Bd7 17.Nf5±] 12.Ne4 Qg6 13.Ng3 Qf6™ [13...Qg4 14.Qd5 fxg2 15.Re1±]
14.Qd3!

415
Position after: 14.Qd3!

14...g6 [14...Qg6 15.Qxf3 Bg4 16.Qd5 Qxc2 17.Qxc5±] 15.Ne4 Qf5 16.Nxc5 fxg2 17.Re1 Qxc5
18.Qf3!

Position after: 18.Qf3!

The threat of Bh6 is destructive. 18...Kg7 [18...Bb7 19.Bh6+–; 18...Ne5 19.Qxa8 Bf5 was the best
chance, still, after 20.Qxg2 Bxc2 21.Be3, Black needs to play 21...Qc6 (otherwise f2-f4 will attack
both the e5-knight and the c2-bishop) and after 22.Qxc6 Nxc6 23.Bf4 he is much worse.] 19.Be3 Qc4

416
20.Bb3 Ne5 21.Qg3 Qh4 22.Qxe5+ 1-0 Svidler, P (2723) – Stefanova, A (2557) Gibraltar 2009.

10.Rxd1

In case of 10.Bxd1?, of course Black won’t transpose to the line above (9...exf3), but just move his f6-
knight, e.g. 10...Ng4µ. This is the point behind Black’s move order.

10...exf3 11.exf6 gxf6! 12.Be4 Bd7

Position after: 12...Bd7

13.a4!

This was considered the refutation of the line, until this game!
13.Bxf3 makes it easy for Black: 13...0-0-0ƒ.

13...0-0-0!N

Anyway!

14.axb5 Ne5!

An amazing idea.

417
Position after: 14...Ne5!

Black’s threat is ...Bc6! At the press conference, Peter Svidler said this move came as a shock, and that
he quickly understood that he should just try not to lose by force.

15.Bf4

And indeed he found the best move. After any other move, White is much worse:
A) 15.Rxa6 Bc6! 16.Bf5+ [16.Bc2 Rxd1+ 17.Bxd1 Rd8 18.Nd2 Bb7 19.Rxf6 Ng4 20.Rxf3 Re8µ]
16...Kb8 17.Rxd8+ [17.Re1 Bb7µ] 17...Rxd8 18.Nd2 Bb7!

418
Position after: 18...Bb7!

19.Bc2 [19.Ra1 Nc4–+; 19.Ra4 fxg2–+] 19...Bxa6 20.bxa6 Nd3µ


B) 15.Rxd7 Kxd7 16.bxa6 Kc8 17.Nd2 Kb8µ
C) 15.Na3 f5 16.Bd5 [16.Bc2 Rhg8 17.g3 Ng4µ] 16...Be6 17.Bxe6+ fxe6 18.Be3 Rxd1+ 19.Rxd1
Bxe3 20.fxe3 fxg2µ
D) 15.bxa6? Bc6 16.Bf5+ Kb8 17.Rxd8+ Rxd8 18.Nd2 fxg2–+

15...Bxb5

Our preparation stopped here.


15...Bc6? is no longer good, because of 16.Nd2 and if 16...Rxd2 then 17.bxc6 Bxf2+ [17...Rxf2
18.Kh1!±] 18.Kh1 fxg2+ 19.Kxg2! Rg8+ 20.Kh3 Rxd1 21.Rxd1 and White is better, although Black’s
position is playable.

16.Na3

Position after: 16.Na3

16...Rxd1+?!

The start of a bad idea, focusing too much on keeping the extra pawn.
16...Be2! was appropriate: 17.Rxd8+ Rxd8 with a slight but comfortable advantage for Black. If
18.Bxe5 fxe5 19.Bxf3 Bd3 Black keeps a strong bishop pair.

419
17.Rxd1 Be2 18.Rd5

Position after: 18.Rd5

18...Rg8?!

At first it might seem clever to force g2-g3 and keep the extra pawn, but the game proves that this
move just helps White since his coordination becomes too good. Furthermore, the now-hanging h7-
pawn is a concrete problem.
18...Bd6 and Black has no problems. I guess White should play 19.Bxe5 fxe5 20.Bxf3 Bxf3 21.gxf3
with a drawish position.

19.g3

19.Rxc5 is inadvisable, although after 19...Rxg2+ 20.Kh1 Rxf2 21.h3! Black doesn’t have more than a
draw: 21...Rf1+ 22.Kh2 Rf2+=.

19...Bxa3

19...Bd6?! is now out of the question due to 20.Bxh7±.

20.bxa3 Ng6 21.Be3 Re8! 22.Bf5+ Kb7 23.Rd4 Re5

420
Position after: 23...Re5

Black’s position is still acceptable.

24.g4 a5 25.h3 h5 26.Be4+ Ka6?

A huge blunder, walking into a mating net.


I do not think White has anything better than repeating moves after 26...Kc8, as for example after
27.Kh2 hxg4 28.hxg4 c5 29.Ra4 Kd7 Black is more than fine.

27.gxh5!

I believe Veselin underestimated this move.

27...Rxh5

421
Position after: 27...Rxh5

28.Rd8!

28.Rd7 was better for White as well, but the text move is stronger.

28...Ne5

28...Rxh3? 29.Rb8! and 30.Bb7+ will be curtains.

29.a4!

Threatening mate in one. Instead, 29.Rb8 would not to the job: 29...c5 30.Bxc5 Rg5+! [30...Nd7?
31.Bb7+ Kb5 32.Bxf3+ and we see why the black rook should be on g5] 31.Kh1 Nd7 32.Ra8+ Kb5
33.Be3 Rh5„.

29...c6™ 30.Rb8!

422
Position after: 30.Rb8!

30...Nc4

30...Nd7 31.Rc8! isn’t any better.

31.Bd4 c5™ 32.Bxf6 Nb6™ 33.Bd8 Nd5™

Good defence by Black. Still, the resulting position is close to lost, or perhaps just lost anyway.

34.Ra8+ Kb7 35.Rxa5 Re5

35...Kc6 36.c4! The most precise. 36...Bxc4 37.Be7+–

36.Rxc5 Rxe4 37.Rxd5 Rxa4

423
Position after: 37...Rxa4

We generally say that opposite-coloured bishop endings give drawing chances to the weaker side, but
here White has two passed pawns that are far apart and Black has doubled pawns. Even if he could
keep them it wouldn’t be such a great help. White’s h-pawn will decide the game.

38.Rf5 Kc8 39.Bg5 Rc4 40.Bd2 Rc7 41.h4 Kd8 42.Bg5+ Ke8 43.Rd5 f5 44.h5 Rc4 45.Rd4

Position after: 45.Rd4

424
45...Rxc3

45...Rxd4 46.cxd4 Bc4 47.Kh2 is an easy win for White: even if the c-pawn became a d-pawn, it is far
enough from the h-pawn so the black king can’t stop both of them.

46.h6 Rc8 47.h7 Kf7 48.Bd8!

Black resigned in view of 48.Bd8 Kg7 49.Rd7++–. What a pity!

1-0

425
Game 6: Kramnik (White)
Topalov, Veselin (2785)
Kramnik, Vladimir (2787)
Khanty-Mansiysk 2014

1.d4 Nf6 2.c4 e6 3.Nf3 d5 4.Nc3 Be7 5.Bf4 0-0 6.e3 Nbd7 7.c5 Nh5!?

Position after: 7...Nh5!?

This is Kramnik’s favourite move, while 7...c6 is Aronian’s.

8.Be5!?

This move itself isn’t new, but our whole idea behind it is. I’m going to show you some extracts from
our analysis, which I believe is the best and most amazing preparation I was lucky enough to work on.
Why some extracts and not everything? Because all of it would take, no exaggeration, one hundred
pages!

426
These two aren’t sitting opposite each other for fun... (photo Anastazia Karlovich)
8.Bd3 Nxf4 9.exf4 c6 10.0-0 b6 11.b4 a5 12.a3 is the much quieter line players usually choose.

427
Position after: 8.Be5!?

8...c6!?

As far as I remember this was played almost a tempo by Kramnik.


A) 8...Nxe5 We checked numerous moves for Black, some of which even objectively solved his
problems, but the most amazing part of our analyis is after this one. 9.dxe5! g6 [9...f5 leads to unclear
positions: 10.Qd4 c6 11.Be2 b6 12.Na4 f4 13.0-0 g6 14.b4 fxe3 15.fxe3 Ng7 16.g4∞] The problem
for White, according to the engine, is that Black vacates the g7-square for his knight, and it’s difficult
to defend the c5-pawn in meaningful fashion. However, White will just sacrifice it...
10.g4! Ng7 11.h4

Position after: 11.h4

Here I analyzed deeply no fewer than eight possibilities for Black! I will just show you some sample
lines, as not everything can fit into the book! Generally speaking, White’s idea is to completely ignore
his c-pawn and focus on the kingside. He has two different plans: to develop his queen on the fourth
rank (d4, or possibly a4) to prepare it to swing to the kingside, or just to develop it to c2. In both
cases, White will castle queenside as soon as possible.
A1) 11...f5 12.Qd4! Qe8 13.Bd3

428
Position after: 13.Bd3

13...fxg4 [13...b6?! 14.c6! In giving this pawn away White should disorganize Black’s position as
much as possible, in other words, not just allow ...bxc5 or ...c5. 14...Qxc6 15.h5 Qc5 16.hxg6 hxg6
17.gxf5 Qxd4 18.Nxd4 gxf5 19.Kd2‚ Even without queens, Black will have a hard time.] 14.Qxg4
Qf7 15.Rh3 [15.Ke2!?] 15...Nh5 Black needs his knight here, otherwise White will soon play h5
himself. [15...Bxc5 16.0-0-0±] 16.0-0-0 Qg7 17.Rg1 c6 [17...Bxc5 18.Ng5±] 18.Ne2!± White will
play Ng3, when Black won’t be able to hold the position for long.
A2) 11...f6 12.h5 gxh5 [12...g5 13.h6 Ne8 14.Bd3 Bxc5 15.Rh5©] 13.Qd4! Again!

429
Position after: 13.Qd4!

13...fxe5 [13...b6!? 14.gxh5‚ followed by Qg4] 14.Nxe5 b6 15.gxh5 bxc5 [Another of our lines
went 15...Bxc5 16.Qg4 Qf6 17.h6 Qxe5 18.Bd3! Rf7 19.Bxh7+! Kf8 20.Bg6 Nf5 21.Bxf7 Kxf7
22.h7 Qh8 23.0-0-0 Bd6 24.Nxd5 exd5 25.Rxd5±] 16.Qg4 Bf6 17.f4 and the position is just a mess.
A3) 11...Bxc5 12.h5 Bb4 [12...f5 13.hxg6 hxg6 14.gxf5 Nxf5 15.Rg1‚; 12...g5 13.h6 Ne8 14.Bd3
f6 15.Rh5©] 13.Qc2 c5 14.0-0-0

Position after: 14.0-0-0

This is another big mess, for example... 14...Bd7 15.Kb1!? [15.Bd3!? c4 16.Bxg6 fxg6 17.hxg6 h5™
18.Nd4©; 15.Be2!? Bxc3 16.Qxc3 Ne8 17.hxg6 fxg6 18.Rh3 Qe7 19.Qd2!‚] 15...Rc8 16.Be2 Bc6
17.hxg6 fxg6

430
Position after: 17...fxg6

18.Rxh7! Kxh7 19.Rh1+ Kg8 20.Qxg6 Rxf3 21.Bd3!!+– (…Rh8+)


B) 8...f6 9.Bg3 Nxg3 10.hxg3 b6 11.Bd3

Position after: 11.Bd3

This is already more than dubious for Black. One of Veselin’s Bulgarian compatriots won an excellent
game with White, actually finding by himself one of the ideas we had in store. 11...f5 12.g4! bxc5
13.gxf5 c4 [13...exf5 14.Nxd5±] 14.fxe6 Nf6

431
Position after: 14...Nf6

15.Bxh7+!! Nxh7 16.Qc2 Nf6 17.Ng5 Bb4 18.Qg6 Bxe6 [18...Qe7 19.Nf7! Rxf7 20.exf7+ Qxf7
21.Rh8+! Kxh8 22.Qxf7+–] 19.Nxe6 Qd7 20.Nxf8 Rxf8 21.Rh4 Qb5 22.Rb1 Qa5 23.Ke2 1-0 Iotov,
V (2553) – Jianu, V (2579) Tromso 2014. A model game for the line!

9.Bd3

Position after: 9.Bd3

432
In a rapid game a year later, Vishy Anand pushed things a little too far: 9.g4? Nhf6 10.Rg1 Nxe5
11.Nxe5 Nd7µ. White is just much worse. Of course, pushing g2-g4 when the h5-knight can just go
back to f6 wasn’t our idea. Anand, V (2797) – Nakamura, H (2776) Zürich 2015, 0-1 (29).

9...g6!?

Again, we checked many moves here in detail. A few lines:


A) 9...b6 10.b4 a5 11.a3 isn’t necessarily in Black’s favour, as now the c6-pawn will be hanging in
many lines. In case of 11...Ba6 White has the typical 12.b5! Bb7 [12...cxb5 13.c6 Nxe5 14.Nxe5±]
13.cxb6 Nxb6 14.0-0²
B) 9...f6!?
B1) 10.Ng5!? was a good try, but is only a draw: 10...fxe5 11.Qxh5 Bxg5 12.Qxh7+ Kf7 13.Qg6+
Kg8 14.Qh7+ Kf7. And now interesting is...

Position after: 14...Kf7

15.Bg6+ Kf6 16.Bh5, but Black holds if he’s accurate: 16...Bh6™ 17.g4! Ke7™ 18.g5 Rh8! 19.Qg6
Qf8 and the position becomes tricky.
B2) 10.Bg3 g6! [10...f5 11.b4 Nxg3 12.hxg3 is easier for White. Black should probably play 12...g6
followed by 13...Nf6 if he wants to prevent any annoying g4 move, but White has two nice squares
for his knights (e5 and f4) and will push his pawns on the queenside.] This is probably a good line
against the Be5 idea. 11.b4 e5!

433
Position after: 11...e5!

12.Be2 [12.dxe5?! Nxg3 13.hxg3 Nxe5³] 12...e4 13.Nd2 Nxg3 14.hxg3 f5 followed by ...Nf6, and
the position is just unclear, probably around equal.
C) 9...Nxe5 10.dxe5! would be pretty similar to the 8...Nxe5 notes, e.g. 10...g6 11.g4 Ng7 12.h4
Bxc5 13.h5©

Position after: 13.h5©

…Qa4, 0-0-0, Qf4, h5, etc..

434
10.h4!

Position after: 10.h4!

A brilliant move, making room for the bishop on h2. White stays ready for a dxe5 and g4 attack if
Black takes on e5.

10...f5

10...Nxe5 11.dxe5 f5!? Otherwise White is likely to get an improved version of the 8...Nxe5 notes, as
Black has played the rather slow ...c6 while White has played the mandatory attacking move h4. Now
White can’t push g2-g4 that easily. All the following lines were in our notes! 12.Qc2

435
Position after: 12.Qc2

A) 12...b6!? 13.g4 fxg4 [13...Ng7 14.h5! fxg4 15.Nh2 and Black’s position will soon collapse. One of
our lines went 15...Nf5!? (…...g5) 16.hxg6 h5 17.0-0-0 Bh4 18.Kb1 bxc5 19.Ne2 Bxf2 20.g7! Kxg7
21.Nf4+–] 14.Bxg6!

Position after: 14.Bxg6!

14...Rxf3 [14...gxf3 15.Bxh7+ Kh8 16.Rg1! Bxh4 17.Rg4‚ …0-0-0] 15.Bxh5 Bxh4 16.0-0-0 Rxf2
17.Qa4‚

436
B) 12...Bxc5 13.0-0-0 Qe7 [13...Be7 14.g4 fxg4 15.Bxg6 is also a huge mess, but definitely easier to
play as White!] 14.Rdg1

Position after: 14.Rdg1

Here White has several ideas: g4 (followed by Bxg6 if ...fxg4), Ng5, etc. An intermediate Na4 might
be useful sometimes, as in the following line: 14...Bd7 15.Na4! [15.Ng5!?] 15...Bb4 16.g4 fxg4
17.Bxg6 gxf3 [17...hxg6? 18.Qxg6+ Ng7? 19.Ng5+–] 18.Rg5! hxg6 19.Qxg6+ Ng7 20.Rhg1

Position after: 20.Rhg1

437
Without the inclusion of Na4 and ...Bc5, Black would win here by playing ...Bxe3+ and ...f2. Now
Black has to find 20...Rf5™ and he’s fine: 21.Qxg7+ Qxg7 22.Rxg7+ Kh8 23.a3!? Be1! [23...Ba5
24.Rxd7 Rg8 25.Rxg8+ Kxg8 26.b4²] 24.R7g6 Rh5! 25.Rxe1 c5! and Black is even a little better.
Well, we knew from the start that playing such a line was a risk!

11.Bh2!

Position after: 11.Bh2!

11...b6

11...Bxh4? would have been too greedy: 12.Qc2 Bf6 13.0-0-0‚ and g2-g4 will soon be destructive.

12.b4 f4!?

Seeking counterplay on the queenside wouldn’t be any good either: 12...a5 13.b5! Bb7 [13...cxb5?!
14.c6±] 14.cxb6 Nxb6 15.0-0!

438
Position after: 15.0-0!

Just ignoring the h4-pawn, as in the game. 15...Bxh4 [15...cxb5 16.Nxb5± …Bc7] 16.Rb1 White has
tremendous compensation, e.g. 16...Be7 17.bxc6 Bxc6 18.Ne5 Be8 19.Qb3 Nc8 [19...Nd7? 20.Nc6+–]
20.Rfc1‚ and despite his extra pawn, Black is paralyzed.

13.0-0!

Position after: 13.0-0!

439
Atypical. Wouldn’t this look more natural if the h4-pawn was on h3? Topalov got what he wanted out
of playing h2-h4, and now he has to focus on the queenside – at the cost of the h4-pawn!
13.exf4?! Nxf4 14.Bxf4 Rxf4 15.h5 g5 would be unclear.

13...a5?

This move is almost losing.


A) The brave 13...Bxh4 was probably the most reliable option: 14.b5 [14.Ne2!?] 14...Bb7 [14...cxb5?
15.c6 Nb8 16.c7 Qxc7 17.Nxb5 Qe7 18.Nxh4 Qxh4 19.Nc7+–] 15.Qa4!

Position after: 15.Qa4!

15...Nb8™ [15...cxb5? 16.Bxb5+–] And now, in order to be better, White simply needs to ‘unlock’
the h2-b8 diagonal. 16.Ne2! fxe3 [In case of 16...Bf6 White may just continue 17.Rac1©.] 17.fxe3
cxb5 18.Bxb5 a6 19.Bd3 b5 20.Qc2 Nc6 21.Bd6

440
Position after: 21.Bd6

with huge compensation: 21...Be7 22.Bxe7 Nxe7 [22...Qxe7? 23.Bxg6! hxg6? 24.Qxg6+ Ng7
25.Ng5+–] 23.Ne5‚ [23.g4!? followed by Nf4 is strong as well.]
B) 13...Bb7?! is too slow: 14.exf4 Nxf4 15.Bxf4 Rxf4 16.h5 g5 17.Qc2

Position after: 17.Qc2

and contrary to the 13.exf4 note, after 17...Rf7 White can just play 18.Rae1 when not only does
White’s extra tempo matter, but the e6-pawn is also weaker since Black has already developed his c8-

441
bishop.

14.b5!

Position after: 14.b5!

14...bxc5

The logical continuation.


14...Bb7 15.bxc6 Bxc6 16.cxb6 Qxb6 [16...Nxb6 17.Ne5± …Qf4] 17.Qe2± Without an extra pawn,
Black’s concept makes no sense at all.

15.bxc6 Nb8 16.Bb5! Ba6 17.a4

Black doesn’t manage to win the c6-pawn, or at least not for free!

17...Qc8 18.dxc5

18.Ne5!? was good as well.

18...Nxc6

442
Position after: 18...Nxc6

19.Nxd5!

Forcing Black into a lost endgame.

19...exd5 20.Qxd5+ Kh8 21.Qxc6 Qxc6 22.Bxc6 Rac8 23.Bb5 Bxb5 24.axb5 Bxc5 25.Rxa5 fxe3
26.fxe3 Bxe3+ 27.Kh1

Position after: 27.Kh1

443
27...Rc2?

Trying to seek counterplay, but now White just wins by force.


27...Nf6 was more resistant.

28.Rb1 Rfc8 29.Raa1!

Preventing a back rank check, and the b-pawn becomes unstoppable.

29...Bb6 30.Be5+ Kg8 31.Ra6 Be3 32.b6 Rc1+ 33.Rxc1 Rxc1+ 34.Kh2 Rb1

Position after: 34...Rb1

35.g4! Bf4+

35...Nf4 36.Ng5 Kf8 37.b7! Rxb7 38.Rf6++–

36.Kg2! Bxe5 37.Nxe5 Nf4+ 38.Kf3 Ne6

38...Nd5 39.Ra8+! Kg7 40.Ra7+ Kf6 41.Nd7+! Ke6 42.b7+–

39.b7

444
Position after: 39.b7

Black will either lose his e6-knight or his rook.

39...Rb3+ 40.Kf2!

Avoiding any knight check.

40...Rb2+ 41.Ke3

What a game!

1-0

Game 7: Andreikin (Black)


Andreikin, Dmitry (2709)
Topalov, Veselin (2785)
Khanty-Mansiysk 2014

1.Nf3 d5 2.d4 Nf6 3.c4 e6 4.Bg5 h6

445
Position after: 4...h6

We had assessed this move as a clean solution to this system.

5.Bxf6 Qxf6 6.Nbd2

In case of 6.Nc3 Black has a choice between transposing into a Ragozin (6...Bb4) or a Semi-Slav
(6...c6).

6...Be7 7.Qc2 0-0 8.e3 c5

Black has solved all of his problems already.

9.dxc5 Nd7 10.cxd5

446
Position after: 10.cxd5

10...exd5!?

10...Nxc5 was a good solution too. If 11.dxe6 Bxe6 Black has tremendous compensation for the pawn,
with ...Rac8 or ...Bf5 coming.

11.Nb3 a5!

Stronger than playing ... b6 right away.

12.a4 b6 13.c6 Bb4+ 14.Kd1

447
Position after: 14.Kd1

White’s set-up is very risky: he has an extra pawn, but Black has the bishop pair and the king on d1 is
very suspicious. If Black manages to recover his pawn, he will win the game.

14...Nc5

A) After the more primitive 14...Nb8 Black would regain his pawn with the better position: 15.Nbd4
[If White starts with 15.Bb5 Black can transpose, but he also has extra moves like 15...Bh3!?.]
15...Bc5 16.Bb5 Bg4 and the white set-up will collapse if the coordination between his pieces is
broken. Black recovers his pawn and White has quite a few things to worry about.
B) 14...Ne5!? was good enough as well: 15.Nxe5 Qxe5 16.Bd3 [16.Nd4 Bc5„] 16...Ra7!

448
Position after: 16...Ra7!

17.Kc1 [17.Ke2 Re7 18.Kf1 f5 is probably just winning for Black: 19.g3 f4! 20.gxf4 Bh3+ 21.Kg1
Rxf4! 22.exf4 Qxf4 – careful, a check on e1 wouldn’t be mate due to Bf1! – 23.Bh7+™ 23...Kh8
24.Qd3 Rf7 25.Qg3 Kxh7–+] 17...Rc7 18.Kb1 Bc5! 19.Nxc5 [19.Nxa5 Be6‚] 19...Rxc6³

Position after: 14...Nc5

But all the same Veselin’s natural move was good as well.

15.Nbd4

449
White has a clear plan though: if he manages to bring his king to a2 and solidify his position, he’s just
winning. Now Veselin played several moves in the wrong direction.

15...Ne4?!

15...Bg4 with the idea of ...Ne6 was still excellent for Black, for example 16.Bb5 [If 16.Kc1 Black can
be smart and play 16...Bh5! to prevent Kb1 due to ...Bg6.] 16...Ne6 17.Kc1 Bc5

Position after: 17...Bc5

18.h3 [18.Nxe6 fxe6 19.Kb1 Bf5 20.Bd3 Bxd3 21.Qxd3 Rac8³] 18...Bxf3 19.Nxf3 Nd8³

16.Bb5 g5?

450
Position after: 16...g5?

It was still the time to unbalance the position by playing 16...Bg4 or 16...Ng5. After the text move
Black deprives himself of these options and also weakens his king.

17.h3 h5?!

The least damage for Black was probably to change his plans and reorganize his pieces: 17...Ra7
18.Kc1 Bc5 19.Kb1 and now after 19...Bxd4 20.Nxd4 Qxf2 Black is clearly worse, but not lost.

18.Kc1 Bc5

I assume 18...g4 was Veselin’s idea, but after 19.hxg4 hxg4 the white king just continues his journey:

451
Position after: 19...hxg4

20.Kb1! and 20...gxf3?! 21.gxf3 followed by Ka2 and Rag1 wins for White.

19.Kb1

Position after: 19.Kb1

The white king has escaped the danger zone and Black’s attempts to create counterplay have failed.
White is close to winning.

452
19...Re8

19...g4 20.hxg4 hxg4 21.Nd2 Bxd4 22.Nxe4 dxe4 23.exd4 Qxd4 24.Ra3+–

20.Ka2 Ra7 21.Rad1 Kf8 22.Rhf1 Kg7 23.Ka1 Bf8 24.Ne2 Rd8 25.h4! g4 26.Nf4 Kg8 27.Nxd5

Position after: 27.Nxd5

Black loses the house.

1-0

A bad game and a sad loss, after which the journey to a World Championship match was almost over.
Veselin was clearly not playing at 100% of his strength, and now that the goal seemed almost beyond
reach, this was unlikely to change, although during the event we were really hoping it would!

453
Game 8: Mamedyarov (White)
Topalov, Veselin (2785)
Mamedyarov, Shakhriyar (2757)
Khanty-Mansiysk 2014

1.e4 c5 2.Nf3 d6 3.d4 cxd4 4.Nxd4 Nf6 5.Nc3 a6 6.h3!?

Position after: 6.h3!?

A line that was increasing in popularity at the time of the Candidates. Right now, it is the third most
popular move after 6.Be3 and 6.Bg5, I believe. Mamedyarov surprised us with his next move, not that
we had ignored it completely, but we had focused on the main lines, such as 6...e5.

6...g6 7.g4 Bg7 8.Bg2 0-0 9.Be3 Nc6 10.Qd2 Bd7 11.0-0-0 Rc8 12.f4 Na5 13.b3 Qc7

454
Position after: 13...Qc7

All the previous moves are quite common. Now Veselin’s choice is probably not the critical one.

14.Nde2

14.Nd5! should lead to a small advantage: 14...Nxd5 15.exd5 e5 16.dxe6 fxe6

Position after: 16...fxe6

and now the engine improves on a game by Gawain Jones with 17.Kb1!? and likes White. [The direct

455
17.f5 led to a good position for Black after 17...exf5 18.gxf5 Rxf5! 19.Nxf5 Bxf5‚ ½-½ (34) Jones, G
(2623) – Hillarp-Persson, T (2546) Douglas 2015.]

14...b5 15.Nd5 Nxd5 16.exd5 Rfe8 17.Rhf1 e6 18.Rf2

Position after: 18.Rf2

Here Mamedyarov found an impressive way to reach a drawish endgame, although it might not be the
best move objectively.

18...Nc4!? 19.bxc4 bxc4

White has no way to prevent ...c4-c3.

20.Bd4

A) 20.c3? Rb8 21.Qc2 Qa5 22.Kd2 Ba4–+


B) 20.Nc3? Qa5 21.Bd4 Qa3+ 22.Kb1 Rb8+–+

20...c3 21.Qxc3

White has to agree to many exchanges.


A) 21.Nxc3?! e5 can’t be good for White.
B) And not capturing the c3-pawn isn’t an option: 21.Qd3? Qa5–+.
C) 21.Qe1? Qa5 22.Rd3 Bb5–+

456
21...Qxc3 22.Nxc3 Bxd4 23.Rxd4 Rxc3 24.dxe6 Rxe6

Position after: 24...Rxe6

25.Rfd2?!

Just accepting that the game is a draw, but after 25.Kd2 Black would have had to suffer a bit: 25...Rg3
[25...Ra3?! 26.Bd5±] 26.Rb4!? [Even 26.Bd5 Re7 27.f5 Rxh3 28.Bb3 applies some pressure.] and now
26...Ree3 is best, according to the engine, but after 27.Rb6 Ra3 28.Rxd6 Be6 29.Bf1!

Position after: 29.Bf1!

457
followed by f5, when White can push with no risk.

25...Rg3 26.Rxd6 Rxd6 27.Rxd6 Bxg4 28.hxg4 Rxg2 29.g5 Rf2 30.Rd4 f6 31.a4 fxg5 32.fxg5 Rf5

½-½
Game 9: Anand (Black)
Anand, Viswanathan (2770)
Topalov, Veselin (2785)
Khanty-Mansiysk 2014

1.e4 c5 2.Nf3 d6 3.d4 cxd4 4.Nxd4 Nf6 5.Nc3 a6 6.h3 e6

The same line – this time with Black!

7.g4 Nfd7!?

Position after: 7...Nfd7!?

One of the sharpest lines, later played many times by Grischuk. This move is prophylaxis against g5,
and also quite flexible, keeping the option of several plans, e.g. ...b5 (if White doesn’t play 8.Bg2) or
...Nc6 plans, as in the game.

8.Bg2 Be7 9.Be3 Nc6 10.h4 Nde5 11.g5 Bd7 12.Nxc6 Bxc6 13.b3

458
Position after: 13.b3

Black has definitely got a comfortable position.

13...f5!?

A critical decision.
13...Qa5 was the natural follow-up. 14.Bd2 [14.Qd4 0-0 15.Bd2 Qc5 16.Qxc5 dxc5 is completely fine
for Black; 14.Qd2 d5! 15.exd5?! Bb4 16.Bd4 Bxd5! 17.Bxd5 0-0-0!–+] 14...Qc5! 15.f4 Nd7 and I like
Black. He has several ideas: ...Rc8, ...b5 or even ...Bd8-a5.

14.f4 Ng4 15.Qe2 Nxe3 16.Qxe3

459
Position after: 16.Qxe3

16...fxe4?!

The beginning of a bad concept.


Again 16...Qa5 was a natural continuation: 17.0-0-0 and now castling either side leads to a very
acceptable position for Black.

17.0-0-0 d5 18.Nxe4 Ba3+?!

This move helps White a lot.


18...0-0 was correct:

460
Position after: 18...0-0

19.Nf2 [19.Kb1 Bd7 20.Nf2 Qc7 21.Nd3 Bd6∞] 19...Qc7! 20.Qxe6+ Kh8 21.Nd3 Rae8 22.Qg4 Bd7
23.Qf3 Rc8 24.Qf2 Bd6©.

19.Kb1 Qe7 20.Nf2 Bc5 21.Qg3

Position after: 21.Qg3

21...Bxf2

461
This decision is difficult to assess, but the position arising after Nd3 and a future exchange on e5 would
be quite similar. In any case, Black stands slightly worse.

22.Qxf2 0-0 23.Qd4 Rf5 24.Rde1 Raf8 25.Rhf1 Qd6 26.Re5 Rxe5 27.fxe5 Rxf1+ 28.Bxf1 Qe7 29.a4
Be8 30.Kb2 Bg6 31.Bh3

Position after: 31.Bh3

And in this slightly worse (but highly unpleasant) position, where Black should have waited passively,
Veselin made a wrong decision.

31...h6? 32.gxh6 gxh6 33.Qg4!

White wins by force.

33...Kf7 34.h5 Be4

34...Bf5 35.Qf4+–

35.a5! Bh7 36.c3

462
Position after: 36.c3

Zugzwang! Black doesn’t have a move.

36...Be4

36...Qd7 37.Qf4+ Kg7 38.Qf6++–

37.c4 Bf5

Not a fun decision, but the position is dead lost after 37...Bh7 as well: 38.cxd5 exd5 39.Qf3++–.

38.Qf4 dxc4 39.Bxf5 exf5 40.Qxf5+ Ke8 41.Qc8+ Kf7 42.Qxc4+ Kg7 43.Qd5+–

463
Position after: 43.Qd5+–

Anand’s technique has been faultless.

43...Kf8 44.Kc3 Ke8 45.b4 Qc7+ 46.Kd4 Qe7 47.Qg8+ Kd7 48.Kd5 Kc7 49.Qg6 Qh4 50.Qd6+
Kc8 51.Kc5 Qf2+ 52.Qd4 Qf7 53.Qc4 Qg7 54.Kb6+ Kb8 55.Qc5 Qf7 56.Qd6+ Kc8 57.e6

1-0

Game 10: Aronian (Black)


Aronian, Levon (2830)
Topalov, Veselin (2785)
Khanty-Mansiysk 2014

1.d4 d5 2.c4 c6 3.Nf3 Nf6 4.Nc3 a6 5.e3 Bf5 6.Bd3 Bxd3 7.Qxd3 e6 8.0-0 Bb4 9.Bd2 0-0 10.Rfd1
a5 11.a3 Be7 12.e4 dxe4 13.Nxe4 Nxe4 14.Qxe4 Nd7 15.Bf4 Re8

464
Position after: 15...Re8

After many natural moves by both sides, Veselin has reached a very solid position.

16.Qc2 Qb6 17.Rd3 c5 18.d5 Bf6 19.Rad1 exd5 20.Rxd5 Nf8 21.R1d2 Ne6 22.Rd6 Qc7 23.Bg3

Position after: 23.Bg3

23...Nd4!

465
A very cute equalizer!

24.Qd1™

24.R6xd4?? Qe7!–+ threatening ...Qe1+ is the trick!

24...Qe7

24...Nxf3+ 25.gxf3 would be about equal. The activity on the d-file compensates for the doubled f-
pawns.

25.h3 Rad8 26.Rxd8 Rxd8 27.Nxd4 cxd4 28.a4 h5 29.b3 h4

Position after: 29...h4

Black is almost a little better, but after accurate defence by White, there wasn’t much to be done.

30.Bf4! g5

30...Re8 31.Kf1=

31.Re2! Qc5 32.Bd2 Qf5 33.Re1 b6 34.Qf3

34.Qh5!? Kg7 35.f4 Qg6 36.Qg4 gxf4 37.Qxf4 d3∞

34...Qxf3 35.gxf3 Kg7 36.Kf1 Kg6 37.Re4! Kf5 38.Ke2 d3+ 39.Kd1 Bd4 40.Rg4 f6

466
Position after: 40...f6

41.f4 gxf4 42.Rxf4+ Ke5 43.f3 Bf2 44.Rg4 Kf5 45.Rf4+ Ke5 ½-½

Game 11: Karjakin (White)


Topalov, Veselin (2785)
Karjakin, Sergey (2766)
Khanty-Mansiysk 2014

1.Nf3 Nf6 2.c4 b6 3.g3 c5 4.Bg2 Bb7 5.0-0 g6 6.d4 cxd4 7.Qxd4 Bg7 8.Nc3 d6 9.Rd1 Nbd7 10.Be3
Rc8 11.Rac1 a6 12.b3 0-0 13.Qh4

467
Position after: 13.Qh4

13...Rc7

This has become the main move, played twice more than 13...Re8.
Also after 13...Re8 we were ready with a novelty. 14.Bh3 Ba8 15.g4 b5 This is a tabiya. 16.g5!? Our
new move, which, since the Candidates, has already been played six times. [16.cxb5 Qa5! is known to
be fine for Black] 16...Nh5 17.c5 [17.Qg4!?] 17...Rc7 18.b4 dxc5 19.bxc5 e6 20.Ne4 with
complications that, according to analysis, were in White’s favour.

14.g4

Until then, almost everyone played 14.Bh3 with, besides g3-g4, the additional idea of recycling the f3-
knight to d5 via e1 without exchanging the light-squared bishops. The main line goes 14...Qb8 (…...b5)
15.g4 e6! 16.g5 Ne8 17.Nd4 Nc5,

468
Position after: 17...Nc5

but according to theory (which we couldn’t improve on much!) Black is doing fine.

14...Rc8!

This amazing move, just moving the rook back, is the best solution for Black. It had already been
played by Karjakin himself against Jakovenko in 2012. Basically, Black avoids the threat of g5 and
Nd5, and changes his plan, considering that the g4-push has pluses and minuses for White as well.
14...h6?!

469
Position after: 14...h6?!

This was the move chosen by several top GMs, like Aronian and Karpov. However, we considered it
dubious, see for example our improvement on a game Kramnik – Karpov: 15.h3 g5 16.Qg3 b5 17.h4
bxc4 18.hxg5 hxg5 19.b4 Qa8 20.Bxg5 Re8

Position after: 20...Re8

21.Bxf6!N [21.Be3 Nf8 22.a4 Ne4 23.Nxe4 Bxe4∞ ½-½ (33) Kramnik, V (2751) – Karpov, A (2710)
Dos Hermanas 1999] 21...Nxf6 22.g5 Nh7 23.g6 fxg6 24.Qxg6 Nf8 25.Qg3±

470
Position after: 14...Rc8!

15.g5N

This logical move was actually our novelty. Instead, 15.Bh3 b5 16.cxb5 was played in the above-
quoted game Jakovenko – Karjakin, but we believed Black would have been doing fine after
16...Bxf3!N [The game continued 16...Qa5 17.Bd2 Bxf3 18.exf3 axb5 19.g5 b4, but here after
20.Na4N 20...Ne5 21.Bg2 I quite liked White.] 17.exf3 Ne5 18.Kg2 axb5!

Position after: 18...axb5!

(The reason why I thought 16...Bxf3! is more precise than playing ...Qa5 first, as Karjakin did.) 19.g5
[19.Nxb5? Rxc1 20.Bxc1 Qa8µ] 19...Rxc3! 20.Rxc3 Qa8 21.Bd4 Nd5©.

15...Nh5

15...Ne8 is a little too passive. For example, one of our lines continued 16.Nd4!? [16.Qh3!?] 16...Bxg2
17.Kxg2 Qc7 18.Qg4 Ne5 19.Qe4 e6 20.f4 Nd7 21.Qf3². White wants h2-h4 and seems to be doing
great.

16.Ne4!?N

Heading to g3.
16.Bh3!? was our other try. Our main line continued 16...b5 17.Nd2 Re8 18.Nde4!? bxc4 19.bxc4

471
Position after: 19.bxc4

19...Rxc4! 20.Nf6+ exf6 21.Qxc4 Rxe3! 22.fxe3 f5 which is very unclear.

16...Rc7!

Position after: 16...Rc7!

Again the best move. I had difficulties in assessing how great the chances were that it would be played,
given how strange it is to play ...Rc7, ...Rc8, and then again ...Rc7, but the Russian was just too well

472
prepared! Apart from there being no Nd5 move for White any more, there is another explanation to this
move which you will see on move 18.
16...b5?! 17.c5 d5 18.Ng3 Nxg3 19.hxg3‚

17.Ng3 Nxg3 18.hxg3 Qa8!

Position after: 18...Qa8!

This is the reason. Without this, White would play Nd4 and try to swing a rook to h1. Now it is much
more difficult to achieve that. Here we mainly analyzed two moves, both leading to a slightly better,
although drawish, position for White.

19.Ne1

19.Bh3!? was our other try, for example 19...Bxf3 20.exf3 Ne5 21.Bg2 Nxf3+ 22.Bxf3 Qxf3 23.Bxb6
Rc6 24.Be3 f6 25.gxf6 Bxf6 26.Qh3².

19...Nc5

A) 19...b5 didn’t fully equalize either: 20.Bxb7 Qxb7 21.cxb5 axb5 22.Rxc7 Qxc7 23.Qb4 Rb8
24.Rc1 Qd8 25.Nd3 Ne5 26.Nxe5 Bxe5 27.Kg2².
B) 19...Rfc8?! would allow 20.Bh3!².

20.Qh1!?

If I remember correctly, Veselin found this very interesting move over the board.

473
We had tried 20.b4 Na4 21.Bd4, but if Black finds 21...e5! 22.Ba1 d5! then it is difficult for White to
achieve anything serious:

Position after: 22...d5!

23.cxd5 Rxc1 24.Rxc1 Bxd5 25.Bxd5 Qxd5 26.Qc4 Qxc4 27.Rxc4 Rd8 and Black should manage to
draw.

20...Rfc8

Position after: 20...Rfc8

474
21.Bxb7?!

A dubious decision, showing that Veselin was simply not in his best shape.
Playing 21.Nd3 first was definitely an improved version of the game: 21...Bxg2 22.Qxg2 Qxg2+
23.Kxg2 Nxd3 24.exd3 b5 [24...Rc6 25.a4 f6 26.b4²] 25.a4².

21...Qxb7 22.Qxb7 Rxb7 23.Nd3 Nxd3 24.exd3 f6

Now Black’s position is just fine.

25.gxf6 Bxf6 26.a4 h5 27.b4 Kf7 28.Kg2 Ke6 29.Kf3 Rf8 30.Ke2 Kf5 31.f3 g5 32.Rh1 Kg6 33.Rc2
e5 34.b5

Position after: 34.b5

And now the position which has arisen is equal, but it is Black who is on the better side.

34...Ra8!?

34...a5=

35.a5 bxa5 36.b6 Bd8 37.Rb1 Rab8 38.Ra2

38.c5 dxc5 39.Bxc5 Kf7 40.Ra2 h4³

38...Bxb6!

A good exchange sacrifice. Fortunately, the position is a forced draw.

475
38...h4 39.gxh4 gxh4 40.Rh1 Bxb6 41.Rxh4=

39.Rab2 Bxe3 40.Rxb7 Rxb7 41.Rxb7 Bc5 42.Rb8 a4 43.Kd1 h4 44.Rg8+

Position after: 44.Rg8+

44...Kf6

44...Kf5!? would have been an excellent try: 45.Rf8+ Ke6! [45...Kg6 46.Rg8+ Kf6 would transpose to
the game] 46.gxh4! [suddenly 46.g4? loses: 46...Be3 47.Rh8

476
Position after: 47.Rh8

47...Kd7! 48.Rb8 Kc7 49.Ra8 Kb7 50.Rh8 Bf4 51.Kc2 Kc6 52.Rc8+ Kb6 53.Rh8 a3 54.Kb3 Kc5
55.Rh6!? e4! 56.dxe4 Kd4–+] 46...gxh4 47.Kc2 Be3 [47...Bf2 48.f4=] 48.Kb2 Bf2

Position after: 48...Bf2

49.Ka3 [49.f4!? Ke7 50.Rf5 h3 51.fxe5 Bd4+ 52.Ka3 Bxe5 53.Kxa4 h2 54.Rh5=] 49...Bg3 50.Rh8
Kf5 51.Kxa4 Kf4 52.Rh6 Ke3 [52...Kxf3 53.Rxd6 e4 54.Rf6+=] 53.Rxd6 e4 54.Re6 Kxd3 55.fxe4=.

45.g4

477
Position after: 45.g4

45...Bf2

45...Be3, with the same plan as in the above line (44...Kf5!? 45.Rf8+ Ke6 46.g4?) is too slow: 46.Kc2
Bf4 [46...e4 47.fxe4 Ke5 48.Rd8 also holds] 47.Rh8 Ke6 48.Rh6+ Kd7 49.c5!? dxc5 50.Kb2=.

46.Rh8 Kg7 47.Rh5 Kg6 48.Kc2 Bd4 49.Kb1 a3 50.Ka2 Bb2 51.Kb3

Position after: 51.Kb3

Black cannnot make progress.

51...Bc1 52.Ka2 Bb2 53.Kb3 a5 54.Ka2 a4 55.Kb1 Bd4 56.Ka2 Bb2 57.Kb1 Bd4

½-½

478
Game 12: Svidler (White)
Topalov, Veselin (2785)
Svidler, Peter (2758)
Khanty-Mansiysk 2014

1.e4 c5 2.Nf3 e6 3.d4 cxd4 4.Nxd4 Nc6 5.Nc3 Qc7 6.Be3 a6 7.Be2

Position after: 7.Be2

7...b5!?

An interesting concept when White has already played Be3. With the bishop on c1 and the white king
already castled, ...b5 is also playable, but much riskier.
Of course 7...Nf6 is the standard continuation.

8.Nxc6 dxc6 9.a4?!

Not such a great move, but fortunately Svidler didn’t seem to know about it!
Something like 9.f4 Bb7 10.0-0 c5 11.Bf3 should be critical.

9...b4?!

A positional mistake, but I assume Svidler just couldn’t find a solution over the board.
A) 9...Rb8? is even worse: 10.axb5 axb5

479
Position after: 10...axb5

11.e5! (probably best) 11...Qxe5 12.0-0 Nf6 [12...Bd6 13.g3 …Bf4 is already game over] 13.Bf3 Qc7
14.Ra7 Bb7 15.Qd4! Bd6 [15...e5 16.Qb6 Qxb6 17.Bxb6±] 16.Rd1! Bxh2+ [16...Nd5 17.Qxg7±]
17.Kf1 0-0 18.g3+–.
B) 9...Bb7 10.axb5 Rd8!= is the right solution [and not 10...axb5? 11.Rxa8+ Bxa8 12.Qa1±].

10.Nb1

Position after: 10.Nb1

480
The knight heads to c4. Black is already clearly worse.

10...Nf6 11.Nd2 c5 12.f3 Bb7 13.Nc4 Nd7 14.Qc1!

14.Bf2!? …Bg3 would be similar.

14...Be7 15.0-0 0-0 16.Bf4 e5 17.Bg3±

Position after: 17.Bg3±

Veselin converted this into a win relatively easily.

17...h6 18.c3 a5 19.Qc2 Ba6 20.Rfd1 Rfd8 21.b3 Bg5 22.Bf2 Nf8 23.cxb4 cxb4 24.Bb6 Rxd1+
25.Rxd1 Qb8 26.Bf2 Qc7 27.Rd5 Rc8 28.h4 Be7 29.Qd2

481
Position after: 29.Qd2

29...Rd8?

A blunder in a very bad position.


29...Bxc4 30.Bxc4 Rd8 was required, but of course White is close to winning.

30.Bb6 Rxd5 31.Qxd5 Qb8 32.Qxa5 Bxc4 33.Bxc4 Qd6 34.Bf2 Qd1+ 35.Kh2 1-0

Even when a tournament goes so badly, winning games is good for morale!
Game 13: Kramnik (Black)
Kramnik, Vladimir (2787)
Topalov, Veselin (2785)
Khanty-Mansiysk 2014

1.d4 Nf6 2.c4 e6 3.Nf3 d5 4.Nc3 c6 5.Bg5 h6 6.Bxf6 Qxf6 7.e3 g6 8.Bd3 Bg7 9.0-0

482
Position after: 9.0-0

9...0-0

9...Nd7 is the other popular option, avoiding White’s next move. I very much assume Veselin can play
both moves, but this isn’t a position we have checked together.

10.Ne5 c5?!

This move looks natural, but the way Kramnik played in the game seems rather convincing for White.
A) In recent games, GMs have preferred 10...dxc4 11.Nxc4 Qe7 with the idea ...b6 and ...Bb7. White
has more space, but Black is very solid and will equalize easily if he manages to play ...c5 under
favourable conditions. For example, a game by Anish Giri continued 12.Qb3 b6 13.Be4 Bb7 14.Rfd1
Na6 15.a3 Rfd8 16.Rd2 c5= ½-½ (31) Andreikin, D (2722) – Giri, A (2768) Tashkent 2014.
B) 10...Nd7 11.f4 is unpleasant for Black.

11.cxd5

483
Position after: 11.cxd5

11...cxd4

11...exd5? 12.Nxd5 Qd8 13.Nf4! cxd4 14.Nfxg6! fxg6 15.Qb3+ and although Black survives with
15...Kh7 16.Nxg6 Rf6, his position remains bad: 17.Ne5+ Bf5 18.Bxf5+ Rxf5 19.Qxb7±.

12.exd4 exd5 13.Re1!

Attacking the d5-pawn for real, as there will be an extra square on e7 for the white knight.
13.Nxd5?! Qd6 14.Bc4 Nc6©

13...Be6

13...Nc6?! 14.Nxd5 Qd6 15.Nxc6 bxc6 16.Ne7+±

14.Bb5!

484
Position after: 14.Bb5!

A very nasty move. White prevents ...Nc6 and makes Black’s development more difficult.

14...Rd8

14...a6 15.Ba4 b5 16.Bb3 and ...Nc6 remains impossible, while the d5-pawn is problematic.

15.Rc1 a6 16.Ba4 Rd6 17.Qb3 b6 18.Ne2!

485
Position after: 18.Ne2!

Another excellent move, with the idea of sliding the queen to the kingside.

18...b5 19.Qa3

The point! Strategy often works better when supported by tactics.

19...Bf8 20.Bc2 Nc6 21.Qg3?!

An aggressive idea, but which objectively seems fine for Black.


21.Qe3, with the idea Nf4, is probably stronger. White is also threatening Bxg6. 21...Ne7!?

Position after: 21...Ne7!?

22.Nf4 [22.g4!? Gaining space. 22...Rc8 23.Nf4 Qg5 24.h3²] 22...h5!? [22...Nf5 23.Bxf5 Bxf5
24.Rc7±] 23.Nxe6 Rxe6 24.Bb3 Nf5 25.Qd2 Rd8 26.f4 Bh6 27.g3²

21...Rc8 22.h4

486
Position after: 22.h4

22...Ne7

It seems the concrete response 22...Be7 worked for Black: 23.h5 [23.Bxg6 fxg6 24.Nxg6 Kh7 25.Nef4
gives White compensation, but not enough for an advantage, e.g. 25...Rg8 26.h5 Bd8 27.Rxe6 Rxe6
28.Nxe6 Qxe6 29.Qd3 Kg7 30.Qg3 Kh7=] 23...Qg5! [If 23...g5? 24.Bb1 Black just stands much
worse.] 24.hxg6 Nxe5 25.dxe5 Qxg3 26.fxg3 Rdc6 27.gxf7+ Kxf7 with sufficient compensation for
the pawn.

23.h5 g5 24.f4 Qg7 25.Rf1 f6

487
Position after: 25...f6

26.Ng4?!

An imprecise move. The engine suggests two possibilities for White, both leading to an unpleasant
position for Black:
A) 26.Ng6 Nxg6 27.Bxg6 Rxc1 28.Nxc1 Bf7 29.Bxf7+ Qxf7 30.Qf3 Qe8 31.Ne2 Qe4 32.Kf2²
B) 26.fxg5 fxe5 27.dxe5 Rdc6 [27...Rb6 28.Nd4 Qxg5 29.Qxg5+ hxg5 30.Nxe6 Rxe6 31.Bf5 Rcc6
32.Bxe6+ Rxe6 33.Rf6 transposes] 28.Nd4 Qxg5 29.Qxg5+ hxg5 30.Nxc6 Rxc6 31.Bf5 Rb6
32.Bxe6+ Rxe6 33.Rf6 Rxf6 34.exf6 Nf5 35.Kf2²

26...Rdc6 27.Ne3

488
Position after: 27.Ne3

27...f5

A good concrete move.


The engine also liked 27...Bf7!?, for example after 28.Qh3 Kh8 it considered the position unclear.

28.fxg5

White couldn’t just allow 28...g4.

28...Qxg5 29.Qxg5+ hxg5 30.Bxf5 Nxf5 31.Rxc6 Rxc6 32.Nxf5 Rc2 33.Neg3 Rxb2

489
Position after: 33...Rxb2

All this sequence was more or less forced.

34.Re1 Bd7?!

Tempting, but risky.


34...Bxf5 35.Nxf5 Rxa2 was much safer. 36.Re8! White wants h6 followed by Rxf8+ and h7. 36...Rc2
[36...Kf7!? 37.Rb8 g4 is also completely fine for Black] 37.Nh6+ [37.h6 Rc7³] 37...Kh7!? 38.Nf5™
and White should make a draw, but definitely no more!

35.Ne7+ Kf7?

Black starts to blunder it all away.


35...Kg7 would still be unclear, e.g. 36.Nxd5 Bd6 37.Ne4 Be6∞.

36.Nxd5 Bc6 37.Rf1+ Ke6

490
Position after: 37...Ke6

38.Ne3?!

This move allows Black to breathe a little. Stronger was 38.Nc7+! Kd6 39.d5! Kxc7 and now Black is
in trouble after two possibilities: 40.Rc1 [Or 40.Rf7+!? Kb6 41.Rf6±] 40...Kd6 41.dxc6 Kc7 42.Nf5±.

38...Bg7 39.Ngf5 Bh8 40.h6

Position after: 40.h6

491
40...Be4!

40...Rd2 41.Ng7+! Ke7 42.Nef5+ Kd7 43.Re1! is even problematic for Black.

41.d5+! Kd7 42.Ng3

42.Ng7, with the idea Rf8, does not seem to be winning:

Position after: 42.Ng7

42...Rb1™ [42...Re2? 43.Rf7+! Kd6 44.Ngf5+ Bxf5 45.Nxf5+ Kxd5 46.Ng7!+– followed by Rf8,
again!] 43.Ne6 Rxf1+ 44.Kxf1 Bxd5! 45.Nc5+ Kd6 46.Nxd5 Kxc5 47.Nc7 a5 and Black should
manage to draw.

42...Bd3

Not a bad move, although 42...Bg6 (preventing 43.Rf7+) or 42...Bb1 (preventing the Rf7+ and Rf3
idea) was probably safer.

492
Position after: 42...Bd3

43.Rf7+

43.Rf8!? Bd4 44.Kh2™ 44...Rxa2 [44...Bxe3 45.Rf3 is unpleasant for Black, although after 45...Rb4
46.Rxe3 Rh4+ 47.Kg1 Bb1 48.Re6 a5 he keeps very good drawing chances] 45.Ngf5 Bxe3
[45...Be5+!? 46.Kh3 b4 47.Rf7+ Kd8∞] 46.Nxe3 Re2 47.Nf5 Bxf5 48.Rxf5 Re4 49.g3 g4 and Black
should manage to draw.

43...Ke8™

43...Kd6? 44.Nef5+ Kxd5 45.Rd7+ Kc4 46.Ne3+ Kc3 47.Nd1++–

44.Rf3 Rxa2 45.Nef5

493
Position after: 45.Nef5

45...Rd2?

A serious blunder in a very tricky position.


The best defence was probably 45...Ra1+ 46.Kh2 Bxf5 47.Nxf5 Kf7! and Black should hold, for
example 48.d6 Kg6

Position after: 48...Kg6

494
49.Ne7+!? [49.Rd3 Kxf5! 50.d7 Be5+ 51.g3 Ra2+ 52.Kg1 Ra1+ with a draw, as after 53.Kf2 Bc7 the
black rook is going to h1] 49...Kxh6 50.Rh3+ Kg7 51.Rd3 Kf7 52.Nc6 Ke6 53.d7 Bf6 54.d8=Q Bxd8
55.Nxd8+ Kf5 with a very likely draw, as the a- and b-pawns are quite powerful.

46.Ng7+! Ke7 47.d6+! Kxd6 48.Rf8

Position after: 48.Rf8

Black loses his h8-bishop, and it looks like it’s all over, but the game still had some surprises in store
for us.

48...b4 49.Rxh8 b3

495
Position after: 49...b3

50.N7f5+??

Giving Black a golden chance...


A) 50.h7 Bxh7 [50...b2 51.Rd8++–] 51.Rd8+ Kc6 52.Rxd2+–
B) 50.Ne4+ Bxe4 51.Rd8+ Kc5 52.Rxd2 Kc4 53.Ne6 Kc3 54.Nc5+–

50...Bxf5??

But Topalov already felt that the game was lost, and missed an amazing opportunity.
50...Kc7! 51.h7 b2 52.Rc8+ [52.Rf8 b1=Q+ 53.Kh2 Qb2] 52...Kxc8! 53.h8=Q+ Kb7

496
Position after: 53...Kb7

An unexpected draw: White cannot manage to capture the d2-rook!


A) 54.Nd6+ Kc6 55.Qc3+ Kd7= [55...Kxd6?? 56.Ne4++–]
B) 54.Qg7+ Kb8 [54...Kb6!? 55.Qd4+ Kb5 56.Nd6+ Kc6 57.Qc3+ Kd7=] 55.Qf8+ Ka7™
[55...Kb7? 56.Qb4+ Ka7 57.Qxd2 b1=Q+ 58.Kh2 Bxf5 59.Qf2++–] 56.Qe7+ [56.Qc5+ Ka8™=]
56...Ka8™ 57.Qe8+

Position after: 57.Qe8+

497
57...Kb7™ [57...Ka7? 58.Qe3+ Kb8 59.Qxd2 b1=Q+ 60.Kh2 Bxf5 61.Qd6++–] 58.Nd6+ Kc7=

51.Nxf5+ Kc7 52.h7 b2 53.Rc8+ Kxc8 54.h8=Q+ Kd7 55.Qh7+ 1-0

Position after: 55.Qh7+

Without a bishop on d3, there is no reason for Black not to resign, as Ne3 is coming next.
Despite the unpleasant result, a truly fascinating game in which both players lost control several times
due to the complexity of the position!
Game 14: Andreikin (White)
Topalov, Veselin (2785)
Andreikin, Dmitry (2709)
Khanty-Mansiysk 2014

1.e4 e5 2.Nf3 Nc6 3.Bb5 Nf6 4.d3 Bc5 5.0-0 Nd4 6.Nxd4 Bxd4 7.c3 Bb6 8.Na3!?N

498
Position after: 8.Na3!?N

This was another of the big ideas we had prepared for the tournament. Since then, it has been used by a
huge number of GMs, including several top players like Carlsen, Anand and Karjakin.

8...c6

One of the few resources which we had missed for Black was shown by Kramnik: 8...0-0!? 9.Bg5
[9.Nc4 c6 10.Nxb6 axb6 11.Ba4 d5= ½-½ (38) Hracek, Z (2631) – Kovalev, V (2531) Jerusalem 2015]
9...d5!? 10.exd5 Qxd5

499
Position after: 10...Qxd5

11.Bxf6 [11.Bc4 Qd6 12.Bb3 Bf5 13.Nc4 Qxd3 14.Nxe5 Qxd1 15.Raxd1 Rae8 16.Nc4 Bg4 17.Rc1
Be2 was a forced draw in Efimenko, Z (2647) – Hovhannisyan, R (2599) Yerevan 2015] 11...gxf6
12.Bc4 Qd7 13.Nc2 [Perhaps 13.Kh1 with the idea of f2-f4 would be a better try.] 13...Qg4 14.d4
Qxd1 15.Raxd1 Bg4 16.Rd2 exd4 17.Nxd4 Rfd8= ½-½ (32) Anand, V (2784) – Kramnik, V (2801)
Zürich 2016.

9.Ba4 0-0

We quite liked White’s position after 9...d6. Here’s an overview of what became theory:

Position after: 9...d6

10.Bc2! Our pick. [10.Bb3 was Carlsen’s and Karjakin’s choice, but Aronian proved it not to be great:
10...a5 11.Nc4 Ba7! 12.a4 0-0 13.Bg5 h6 14.Bxf6 Qxf6 15.Nxa5 d5³ 0-1 (44) Carlsen, M (2850) –
Aronian, L (2781) Reykjavik 2015.]
A) We considered 10...Bg4 11.Qe1 Be6 12.Nc4 Bc7 13.f4 to be great for White, and practice seems
to have been on our side:

500
Position after: 13.f4

13...exf4 14.Bxf4 Nh5 15.Bd2 Bxc4 16.dxc4 h6 17.c5! 0-0 18.Rf5 g6 19.Rxh5 gxh5 20.Bxh6± 1-0
(62) Van Foreest, J (2629) – Van Foreest, L (2481) Wijk aan Zee 2018.
B) 10...h6 11.Nc4 Bc7 12.f4 brought me an excellent position in Edouard, R (2613) – Gascon Del
Nogal, J (2455) Roquetas 2017, ½-½ (38).
C) 10...0-0 11.Bg5!? [11.Nc4 Bc7 12.Bg5 d5 was a little dry in Van Foreest, J (2609) – Leko, P
(2679) Berlin 2018, ½-½ (33)] 11...h6 12.Bh4 Be6 [12...d5 13.exd5 cxd5 14.Bb3 transposes into the
game; 12...Bc7 13.d4 and Black should always watch out for f2-f4] 13.Bb3ƒ

10.Bg5 h6 11.Bh4

501
Position after: 11.Bh4

11...d5?!

We considered 11...d6 to be a better move, although practice has also given preference to White:
12.Nc4
A) 12...g5 13.Bg3 Bg4 14.Qd2

Position after: 14.Qd2

502
14...Bc7 [14...Nh5 15.d4 Be6 16.Ne3 Nf4 17.Bb3 Qf6 18.d5² 1-0 (45) Balogh, C (2632) – Buhmann,
R (2582) Germany 2014] 15.Ne3 Be6 16.Qe2 Qe7 17.Rad1 Rad8 18.Bb3 Rfe8 19.Qf3 d5 20.Nf5
Bxf5 21.Qxf5 Nh5 22.Qh3 Nxg3 23.fxg3!² 1-0 (56) Fedorchuk, S (2657) – Pelletier, Y (2568)
France 2015.
B) 12...Bc7 13.Ne3 Be6 [13...Bb6 14.Qf3 Kh7 15.Bb3 g5 16.Bg3² ½–½ (70) Anand, V (2786) –
Karjakin, S (2781) Stavanger 2017] 14.Bc2 d5 15.exd5 cxd5 16.d4² 1-0 (59) Van Kampen, R (2615)
– Sevian, S (2511) Wijk aan Zee 2015.

12.exd5 cxd5 13.Bb3 g5 14.Bg3 Re8 15.Re1 Bc7

Position after: 15...Bc7

16.Nc2

The prophylactic 16.h3!? made sense as well.

16...Bg4 17.Qd2 d4?!

17...Nh5 looks more natural, although the position arising after 18.d4 Nxg3 19.fxg3!, with the idea
Ne3 and Rf1, seems easier to play as White.

18.h3?!

The engine came up with an excellent move: 18.Bc4!

503
Position after: 18.Bc4!

with the idea to threaten cxd4 (as ...Ba5 will be met with b2-b4) and expand on the queenside with a4,
for example 18...a6 19.a4 dxc3 20.Qxc3 Nd7 21.Ne3².

18...Bf5 19.Ba4 Bd7 20.Bxd7 dxc3 21.Qxc3

Position after: 21.Qxc3

21...Nxd7?!

504
21...Qxd7 was a clear equalizer: 22.Bxe5 Nd5! 23.Qd4 Rxe5 [23...Bb6!? 24.Qg4 Qxg4 25.hxg4
Rab8©] 24.Rxe5 Bxe5 25.Qxe5 Nf4 26.Ne3 [26.d4?? Re8–+] 26...Re8=.

22.d4 Rc8 23.Qb3 Ba5 24.Re2?!

24.Red1! was the real test for Black.

Position after: 24.Red1!

After 24...exd4 White’s two options to recapture the pawn are challenging for Black, who must be
extremely precise if he wants to escape the danger zone: 25.Rxd4 [25.Nxd4 Nc5! 26.Qf3 Ne4! 27.Nb3
Qb6 28.Nxa5 Nxg3 29.Nxb7 Ne2+ 30.Kh1 Rc4! 31.Nd6 Rf4 32.Qb3 Rb8©] 25...Bb6 26.Rd6 Qe7!
27.Rxh6 Kg7 28.Rd6

505
Position after: 28.Rd6

28...Bxf2+! 29.Kxf2 Qe2+ 30.Kg1 Rxc2 31.Qd5 Re5! [31...Ne5 32.Rf1 is highly unpleasant for Black]
32.Bxe5+ [32.Re1 Rxd5 33.Rxe2 Rc1+ 34.Kh2 Rxd6 35.Bxd6 Rd1=] 32...Nxe5 followed by ...g4,
which White will have to take (to prevent ...Nf3+), and Black will have at least a perpetual check with
...Qf2/h4.

24...exd4

Position after: 24...exd4

506
Now it’s too many exchanges.

25.Rxe8+ Qxe8 26.Nxd4 Nc5 27.Qf3 Qe4?!

Not such a bad move, but Black loses a pawn he had no reason to give away.
27...Rd8 28.Nf5 Qe6 was just fine for him.

28.Nf5 Qxf3 29.gxf3 Rd8 30.Nxh6+ Kg7 31.Nf5+ Kg6 32.Nd6 Bb6

White has won a pawn, but Black’s position remains easy to play.
The outcome should definitely be a draw, and it was. Veselin was unable to create any real problems
and the Russian defended accurately.

Position after: 32...Bb6

33.Nc4 Ne6 34.Re1 Bd4 35.b3 Rd5 36.Kf1 Bc5 37.Re2 a6 38.Rd2 Rxd2 39.Nxd2 f5 40.Bb8 f4
41.Ne4 Be7 42.Ba7 Kh5 43.Bb6 Kh4 44.Kg2 Bd8 45.Bc5 Kh5 46.Bd6 Kh4 47.Bb4 Kh5 48.Ba3
Kg6 49.Kf1 Kh5 50.Bb4 Kg6 51.Nd2 Kh5 52.Nc4 Bc7 53.Be7 Kh4 54.Kg2 Bd8 55.Bb4 Bc7
56.Nd2 Kh5 57.Ne4 Bd8 58.Kf1 Kg6 59.Bd6 Kh5 60.Ba3 Kg6 61.Bb2 Be7 62.a4 Kh5 63.Be5 Kh4
64.Kg2 Nc5 65.Nxc5 Bxc5 66.Bf6 Bd6 67.Kf1 Kxh3 68.Bxg5 Kh2 69.Ke2 Kg2 ½-½

507
Waiting for the event to finish... (photo Anastazia Karlovich)
The war was finally over, and Veselin took last place in the tournament.

Our ideas had been fruitful, although sometimes the final result hadn’t been the one we expected after
getting our novelties on to the board. Taking the event as a whole, the loss against Svidler as Black in

508
particular was a huge blow.
Other reasons for this failure are difficult to find. Veselin seemed to be fit, motivated and sleeping well.
Sometimes a tournament doesn’t go well, without any clear explanation.
Nevertheless we came away with many good memories of this Grand Prix cycle. The ambition in our
work brought Veselin many points, and I’m pretty sure both of us are now happy that we met. Working
with someone who is such a strong player and at the same time easy to get along with was a priceless
experience!

509
Chapter 10
A Few Novelties More

You’ll have realised that working with Topalov was more than just ‘a job’.
A few weeks after the Candidates tournament, I won the strong Dubai Open, and got yet more proof of
Veselin’s and Silvio Danailov’s friendship when they immediately called to congratulate me, and said
they truly hoped all the work done with Vesko had helped make it happen.
Nowadays we don’t do any chess work, but always take the opportunity to have coffee and discuss
chess or life together.
One thing that left an impression on me was an interview Veselin gave in Gibraltar in 2015. He was
asked if his plan was still to become World Champion, and his answer was a collector’s item:
“Theoretically yes, but there is this Norwegian guy Carlsen who is younger, fitter, better, and still
improving while I’m not”.
Still, one year after we stopped working together, Veselin managed to beat Magnus with Black, thanks
to a novelty we had prepared together, which was nice to see, even if I wasn’t physically there to assist.
Carlsen, Magnus (2853)
Topalov, Veselin (2816)
Saint Louis 2015

1.e4 c5 2.Nf3 d6 3.Bb5+ Nd7 4.0-0 Ngf6 5.Re1 a6 6.Bd3 b5 7.c4 g5!?N

Position after: 7...g5!?N

One of the many novelties we had prepared at the time of the Candidates. The idea is copied from

510
another line of the same opening: 1.e4 c5 2.Nf3 d6 3.Bb5+ Bd7 4.Bxd7+ Qxd7 5.0-0 Nf6 6.Qe2 Nc6
7.Rd1 g5!?.

7...g5!?N 8.Nxg5

The other pawn capture, 8.cxb5, is the second most critical move, but I believed Black’s position was
interesting and the play double-edged, e.g. 8...g4 9.e5 [9.Nh4 e6 10.Nc3 Bg7 11.Be2 0-0 12.a4 d5
13.exd5 Nxd5 14.Nxd5 Qxh4 15.Ne3 f5!‚] 9...dxe5 10.Ng5. This position was played many times at
the World Blitz Championship in Berlin in 2015!

Position after: 10.Ng5

A) 10...e6?! was Anand’s choice, but Mamedov proved it wrong in magnificient fashion: 11.Nc3 h5
12.Qe2 Nd5 13.Nxe6! fxe6 14.Bg6+ Ke7 15.d4! Bh6 16.Nxd5+ exd5 17.dxc5‚ and Black’s position
was a nightmare. Mamedov, R (2657) – Anand, V (2803) Berlin 2015, 1-0 (38).
B) 10...Bg7, Grischuk’s choice in two blitz games, also seems reliable: 11.Qe2 [11.Nc3 0-0 12.bxa6
e6 13.Nge4?! Nxe4 14.Bxe4 Rxa6 15.Qe2?! f5! 16.Bd3 Rd6 17.Bc4 e4 18.d3 Ne5! 19.dxe4 f4
20.Qf1 Qh4–+ 0-1 (23) Mamedov, R (2657) – Grischuk, A (2774) Berlin 2015] 11...0-0 12.Nc3 Nb6
13.bxa6 c4 14.Bc2

511
Position after: 14.Bc2

14...h6!N [14...Bxa6 15.Qxe5 h6 16.Nge4 was a better version for White in Bartel, M (2622) –
Grischuk, A (2774) ½-½ (44), Berlin 2015] 15.Nge4 Nxe4 16.Bxe4 Rxa6!?∞ …...f5
C) 10...Nb6!?N

Position after: 10...Nb6!?N

This was our plan, which has yet to be played. 11.Bf1 [11.Qe2 Bg7 12.bxa6 0-0 13.Nc3 c4∞]
11...Rg8 12.Ne4 Nxe4 13.Rxe4 Bg7 14.Nc3 f5 15.Re1 Kf7∞ Our analysis ended here!

512
8...Ne5 9.Be2

Position after: 9.Be2

The bishop is more useful here than on f1. When the g5-knight returns to f3, White is happy to answer
...Nxf3+ with a bishop recapture.

9...bxc4 10.Na3?!

10.Nc3!

513
Position after: 10.Nc3!

A few months later, Magnus showed this move to be Black’s biggest problem in this line.
A) 10...Bh6?! was tried by Ponomariov, but it doesn’t seem to be good: 11.Nf3 Nd3 12.Bxd3 cxd3
13.e5 dxe5 14.Nxe5 Kf8?! 15.Qf3± 1-0 (72) Ter Sahakyan, S (2601) – Ponomariov, R (2715)
Gjakova 2016.
B) 10...Rg8 also happens to be less useful than ...Rb8; when White plays d4 (the most natural follow-
up), it is better to keep options of ...Bg7 and ...0-0, as there’s no attack on the white king anyway.
11.d4 cxd3 12.Bxd3 h6 13.Nf3 Nxd3 14.Qxd3 Bb7 15.Bf4! d5 16.exd5 Nxd5 17.Bg3² ½-½ (98) Ni
Hua (2681) – Li Shilong (2514) China 2016.
C) 10...Bg7? allows 11.f4 Nd3 12.Qa4+! followed by Qxc4, and the g7-bishop will be hanging after
Qxf7+.
D) 10...Rb8 The engine move, chosen by Grischuk. It seems like the most useful waiting move, as the
following analysis shows. As you will see in the game, Grischuk could capture the b2-pawn thanks to
his rook being on b8.

Position after: 10...Rb8

D1) 11.f4 seems critical as well, when the engine comes up with an astonishing line: 11...Nd3
12.Bxd3 cxd3 13.Qa4+ Qd7 14.Qc4

514
Position after: 14.Qc4

14...h6!? 15.Qxf7+ Kd8 16.Nh3 Qg4! 17.Nf2 Qxf4 18.e5 Rg8! 19.exf6

Position after: 19.exf6

19...Rxg2+! 20.Kxg2 Bh3+!! with perpetual check – I wonder if Magnus had prepared it, as over the
board 11.f4 seems very tempting!
D2) 11.d4 cxd3 12.Bxd3 Bg7∞
D3) 11.Rf1!? A very clever move by the World Champion. White is threatening f4, followed by

515
Qa4+ and Qxc4, as there won’t be a rook hanging on e1. 11...h6 12.Nf3

Position after: 12.Nf3

12...Nd3 [12...Nxf3+ 13.Bxf3 Rg8 14.Kh1²] 13.Ne1! The other idea behind 11.Rf1. 13...Nxb2
14.Bxb2 Rxb2 15.Bxc4 Rb4 16.Qe2

Position after: 16.Qe2

and White’s position was much more pleasant: 16...Bg7 17.Nc2 Rb6 18.Rab1 0-0 19.Rxb6 Qxb6
20.Ne3 e6 21.f4 Kh8 22.f5² 1-0 (38) Carlsen, M (2834) – Grischuk, A (2747) London 2015.

516
10...Rg8

Position after: 10...Rg8

11.Nxc4!?

Such inspirational play by Carlsen. Amazing!


11.d3 (or 11.d4) would be the almost universal choice, leading to an unclear position after 11...cxd3
12.Bxd3 h6 13.Nf3 Nxd3 14.Qxd3 d5∞.

11...Nxc4 12.d4!

Of course this was the idea. White has sacrificed a piece but is forcing Black to play
backward/defensive moves.

12...Nb6 13.Bh5

13.dxc5 dxc5 14.Qxd8+ Kxd8 15.Nxf7+ Ke8³ is definitely not what White wants.

13...Nxh5 14.Qxh5

517
Position after: 14.Qxh5

14...Rg7?!

This move gives away part of the advantage, and the position becomes difficult to play.
A) 14...Rxg5 was probably simplest and best, e.g. 15.Bxg5 cxd4 16.Qxh7 [16.Rad1 Nd7 17.Rxd4
Qa5µ] 16...Nd7 17.h4 Qa5! 18.Qg8 Qe5 19.h5 Nf6 20.Bxf6 Qxf6µ.
B) 14...Rg6 was good as well, leading to positions similar to the line immediately above: 15.Qxh7
Rg7 16.Qh8 cxd4 17.Nh7 Rxh7 18.Qxh7 Nd7µ.

15.Nxh7

518
Position after: 15.Nxh7

15...Qd7?

A) 15...Nd7 might have been best, e.g. 16.e5!? dxe5 17.dxe5 e6 18.h4 Bb7 19.Bg5 Be7 20.Rad1
Bxg5 21.Nxg5 Qe7„.
B) 15...Rxh7 16.Qxh7 cxd4 17.Qh8 e5 18.h4 also looks pretty unclear.

Position after: 18.h4

519
Here the engine goes ...Kd7−c6−b7 and gives Black a very small edge!
C) 15...Bg4!? and then ...Rxh7 also made sense.

16.dxc5

16.Nxf8 Qh3! was Topalov’s idea. White has no reason to enter this mess without exchanging pawns
on c5 first.

16...dxc5

Position after: 16...dxc5

17.e5?

Magnus wants too much, and his last move will just result in a much worse position.
17.Nxf8 Qh3™ 18.Qxh3 Bxh3 19.g3 was required. White will play Bh6, when he isn’t worse.

17...Qc6! 18.f3

18.Bg5 Bb7 19.Nf6+ Kd8! [19...exf6?? 20.exf6+ Kd7 21.Qh3++–] 20.Ne4 Qg6µ

18...Qg6 19.Nf6+ Kd8!

520
Position after: 19...Kd8!

White is just a piece down.

20.Qxg6 Rxg6 21.Ne4 Bb7 22.h4 Rc8 23.h5 Rg8 24.Bd2 Nc4 25.Bc3 Bh6 26.Rad1+

26.Kf2 Bf4–+

26...Ke8

521
Position after: 26...Ke8

27.Rd3

White’s 26th move probably made his situation even worse, as now 27.Kf2? loses right away to
27...Be3+! 28.Rxe3 Rxg2+!–+.

27...Bf4

The rest is relatively easy, even against the World Champion.

28.Nf2 Bc6 29.Nh3 Bg3 30.Re2 Bb5 31.Rd1 Bc6 32.Nf2 Bxe5 33.Ng4 Bxc3 34.bxc3 Kf8 35.Kf2
Rh8 36.Ne5 Nxe5 37.Rxe5 Be8 38.g4 f6 39.Re6 Bb5 40.Rde1 Rc7

0-1

Smiling and relaxed (photo Vladimír Jagr)


Veselin is definitely not the only one who benefited from our ‘aggressive’ opening ideas.
In 2016, I qualified for the final of the Karpov Trophy in Cap d’Agde by winning my semi-final
against... Karpov himself!

522
And the work done with Vesko wasn’t without its influence. Have a look...
Karpov, Anatoly (2628)
Edouard, Romain (2628)
Cap d’Agde 2016

1.d4 Nf6 2.c4 e6 3.Nc3 Bb4 4.Qc2 d5 5.a3 Bxc3+ 6.Qxc3 c5!?

Position after: 6...c5!?

A very aggressive line we had prepared in case of a must-win situation with the black pieces.

7.dxc5 d4 8.Qg3

The critical and most aggressive move.

8...Nc6!?

523
Position after: 8...Nc6!?

A very tense atmosphere: playing the semi-final of the Karpov Trophy against Anatoly Karpov himself!

524
(photo Pierre Textoris – CAPECHECS)

9.Qxg7

A) 9.Nf3 is the other critical move. For example, GMs Naiditsch and Moiseenko played the way we
were intending to as well: 9...e5

Position after: 9...e5

A1) We didn’t consider 10.e4 a serious problem: 10...dxe3!? [10...0-0 11.Bd3 Nd7!? 12.b4 a5
13.Rb1 axb4 14.axb4 Ra4∞] 11.Bxe3 0-0 12.Be2 Qa5+ 13.b4 [13.Nd2 Ne4 14.Qh4 f5∞] 13...Nxb4
14.0-0 Nc2 15.Bh6 Nh5 16.Qxe5 Nxa1 17.Qxh5 gxh6 18.Rxa1 Re8„.
A2) 10.Nxe5 Qa5+! 11.Kd1 [11.Bd2 Qxd2+! 12.Kxd2 Ne4+ 13.Ke1 Nxg3 14.Nxc6 Nxh1
15.Nxd4 h5 16.e3 h4³ 0-1 (31) Henrichs, T (2492) – Moiseenko, A (2698) Germany 2014]
11...Ne4!

525
Position after: 11...Ne4!

12.Qf4 [12.Qxg7 Nxf2+ 13.Kc2 Bf5+ 14.Kb3 0-0-0‚] 12...Nxf2+! 13.Qxf2 Nxe5 14.Qxd4 f6³ 0-
1 (29) Debashis, D (2451) – Moiseenko, A (2702) Fujairah 2012.
A3) If 10.Qxg7 Black has at least a draw: 10...Rg8 11.Qh6 Rg6 12.Qh4 Rg4=.
A4) 10.b4! e4!? 11.Nd2 0-0 12.e3!

Position after: 12.e3!

Objectively 12...Re8!? was probably best. [12...a5?! 13.b5 Ne7 14.Be2 Be6 15.Bb2 Nf5 16.Qf4 Rc8

526
17.Rd1! and Black was in big trouble in Ivanisevic, I (2643) – Naiditsch, A (2657) Bled 2016, ½-½
(36).] 13.Bb2 Ne5 and now if White finds 14.Bxd4 Nh5 15.Qxe5 Rxe5 16.Bxe5 he is better:

Position after: 16.Bxe5

16...Qe7 17.Bd4 Nf6 18.h3!² Yes, you’ve got it, the whole line is a bluff! [18.Be2 Bg4∞]
B) 9.b4 e5 10.e4

Position after: 10.e4

is another line, but isn’t dangerous. Carlsen won a very straightforward game when he wasn’t yet

527
World Champion: 10...0-0 [10...Nxe4? 11.Qxg7±] 11.Bd3 b6 12.Nf3 bxc5 [12...Nh5!?] 13.b5 Nh5
14.Qg5 Qxg5 15.Bxg5 Na5 16.Rb1 f6 17.Bd2 Nb7³ 18.h3 g5 19.Ke2 Nd6 20.g4 Ng7 21.a4 Bb7
22.h4 Nxe4 23.hxg5 Nc3+ 24.Bxc3 e4 25.Bxe4 Bxe4 26.Bd2 Rae8 0-1 Van Wely, L (2681) –
Carlsen, M (2733) Nice 2008.

9...Rg8 10.Qh6

Position after: 10.Qh6

10...e5!

10...Na5!? might be worth a try too.

11.b4

The main line.


11.g3 isn’t dangerous: 11...Na5! [I’ve tried the over-optimistic 11...Rg6, but after 12.Qd2 Ne4 13.Qd1
Nxc5 14.Bg2 a5 15.Nf3 my position was objectively dubious in Hambleton, A (2445) – Edouard, R
(2639) Montreal 2015, 0-1 (46).] 12.b4 [12.Bg5 Rg6 13.Bxf6 Qxf6 14.Qxh7 Nxc4∞] 12...Rg6
[12...Nb3 13.Rb1 Rg6 14.Qh4 Rg4 15.Qh3 Rg6 16.Qg2 Nxc1 17.Rxc1 e4] 13.Qh4 Nxc4∞ [13...Rg4=]

11...Rg6!?

11...Be6 is the other option, and theory after this move has developed in absolutely incredible fashion:
12.Bg5 [12.Nf3?! is just a draw: 12...Rg6 and now White should accept a draw by repetition
(Qh4/...Rg6/Qh6), as after 13.Qd2 e4 he’s just clearly worse: Dvirnyy, D (2552) – Ipatov, A (2648)
Gjakova 2016, 0-1 (41)] 12...Rg6 13.Qh4 a5! [13...Bxc4? 14.Nf3 a5 15.Nd2±] 14.b5 Nb4!

528
Position after: 14...Nb4!

The start of insane complications.


A) 15.axb4 axb4 16.Rd1 [16.Rxa8? Qxa8–+; 16.Rb1 b3„] 16...b3 17.Nf3 b2 18.Bxf6 Ra1 19.Bxd8
Rxd1+ 20.Kxd1 b1=Q+ 21.Kd2=
B) 15.Rc1 Critical. 15...Na2! 16.Rc2 Nc3 17.Nf3 Bf5

Position after: 17...Bf5

18.Rc1 [18.Nxe5!N could be a serious issue, e.g. 18...Rxg5 19.Qxg5 Bxc2 20.Qg7! Bg6 21.c6!

529
Position after: 21.c6!

21...d3 22.Qh8+ Ke7 23.Qxd8+ Rxd8 24.exd3 Kd6 25.f4 bxc6 26.Kd2± Obviously this is just some
crazy computer line. 18.Bxf6 Rxf6∞] 18...Qe7 19.Bxf6 Rxf6 20.Qg3 Re6 21.Qg8+ Qf8 22.Qxf8+
Kxf8 23.g3² ½-½ (50) Michiels, B (2536) – Ipatov, A (2612) France 2015.

12.Qd2

After 12.Qh4 Black will have to be creative if he wants to avoid a draw, but after 12...Rg4 it is also
hard to imagine White will be willing to put his queen on h3.

12...Ne4 13.Qb2

530
Position after: 13.Qb2

13...Qf6!?N

Kosteniuk tried 13...a5!? 14.b5 Qf6, but after 15.e3 Ne7 it was just a transposition to the game.

14.e3

In my opinion 14.Nf3 should be the problem for Black, e.g. 14...a5 15.b5 Ne7 16.g3

Position after: 16.g3

531
and the position arising after 16...Be6 17.Bg2 Bxc4 18.0-0 Nc3 is a complete mess, but after 19.Re1 e4
20.Nxd4! Qxd4 21.Be3 Qe5

Position after: 21...Qe5

22.Kh1! followed by Bd2, White should be better. Yes, bluff again!

14...a5 15.b5 Ne7

Position after: 15...Ne7

532
16.Bd3?

A huge mistake.
16.Nf3

Position after: 16.Nf3

A) 16...Bg4!? is also simple and strong: 17.Nxe5™ [17.Ng1 Nxc5µ] 17...Qxe5 18.exd4 Qf6 19.d5
[19.h3?! Bf3!³] 19...Qxb2 20.Bxb2 Nxc5∞
B) 16...Bh3!? This was played by Kosteniuk, and brought her a delightful victory. 17.exd4 Bxg2
18.Bxg2 [18.Nxe5? Bxh1–+ 0-1 (40) Huang, Q (2476) – Kosteniuk, A (2495) Astana 2013] 18...Rxg2

533
Position after: 18...Rxg2

19.Qe2! [19.Nxe5? 0-0-0–+] 19...Qf5! [19...Nc3?! 20.Qd3 exd4 21.Kf1±] 20.Rf1 [20.Be3 Nc3
21.Nh4 Qe4 22.Qf3 Rg4 23.Qxe4 Rxe4 24.Nf3 exd4 25.Nxd4 0-0-0©; 20.Nxe5 0-0-0 21.Be3 Nxf2!
22.Rf1 Qh3! 23.Rxf2 Rg1+ 24.Kd2 Rxa1∞] 20...Rd8!? and the position is a mess. We had all of this
in the file I had prepared with Vesko! [20...0-0-0 21.Bb2 exd4 22.0-0-0 Qf4+ 23.Nd2 is better for
White; if the black king was on e8, Black would have ...Nc3!.]

16...Nxc5

534
Position after: 16...Nxc5

Now White is just lost.

17.Bxg6 Qxg6 18.Kf1 Be6

Position after: 18...Be6

19.g3?

Making things worse.


19.Nf3 Bxc4+ 20.Kg1 Bd5–+

19...Bxc4+ 20.Kg2 Qd3 21.h4 Rc8 22.e4 Nxe4 23.Nh3 Bd5 0-1

535
Position after: 23...Bd5

Thanks to this game, I was able to defeat former World Champion Anatoly Karpov 2-0 in a rapid match
– something that would probably never have happened without the hard work done with Veselin! All in
all, four fantastic years!
Time now to turn to the last chapter of the book, exercises, and see if you can ‘win like Topalov’!

536
Just defeated Anatoly Karpov and had a chance to analyze the game with the legend himself...
Unforgettable! (photo Pierre Textoris – CAPECHECS)

537
Show in Text Mode

EXERCISES
Win Like Topalov!

(Photo Lennart Ootes)

The following exercises are arranged in order of difficulty,


from 1 (very easy) to 5 (very hard).
In all of them, you must find a decisive advantage for
Topalov – of course, the higher the level, the deeper the
investigation necessary!
1
Topalov, V. – Morozevich, A.

(1) □ 33.? +–

Show/Hide Solution

33.Be4!

538
1-0 Topalov, V (2739) – Morozevich, A (2742) Cannes 2002.
Black resigned in view of 33.Be4 Rxb5 (33...Rxe4 34.Rxb8++–) 34.Ra8++–.

2
Topalov, V. – Gelfand, B.

(1) □ 57.? +–

Show/Hide Solution

57.Re8+!

1-0 Topalov, V (2801) – Gelfand, B (2723) Monte Carlo 2006.


Black resigned in view of 57.Re8+ Nxe8 58.Rxe1++–.

3
Grischuk, A. – Topalov, V.

539
(1) ■ 37...? –+

Show/Hide Solution

37...Nxe2!

0-1 Grischuk, A (2781) – Topalov, V (2798) Stavanger 2015.


White resigned in view of 37...Nxe2 38.Qxc6 Nxd4+–+.

4
Timman, J. – Topalov, V.

540
(1) ■ 53...? –+

Show/Hide Solution

53...Qc4! 54.Qa8+

54.Rxc4 f1=Q+ 55.Ka2 Qxc4+–+

54...Ke7 55.Qxa7+ Bc7

0-1 Timman, J (2594) – Topalov, V (2743) Wijk aan Zee 2003.

5
Moen, A. – Topalov, V.

541
(1) ■ 33...? –+

Show/Hide Solution

33...Bh6! 34.Qxh6

34.Qf1 Bxc1–+

34...Qe1+ 35.Rxe1 Rxe1#

0-1 Moen, A (2390) – Topalov, V (2771) Rhodes 2013.

6
Topalov, V. – Adams, M.

542
(2) □ 45.? +–

Show/Hide Solution

45.Qe5+!

Simple chess!

45...Qxe5 46.fxe5 Rgxe8

Otherwise White plays Nf6 and wins.

47.Rxe8+ Rxe8 48.d7

1-0 Topalov, V (2735) – Adams, M (2725) Cap d’Agde 2003.

7
Topalov, V. – Pähtz, T.

543
(2) □ 15.? +–

Show/Hide Solution

15.Bxd5+! cxd5 16.Rc7

1-0 Topalov, V (2739) – Pähtz, T (2365) Gibraltar 2017.


Black resigned in view of 16.Rc7 Rb8 17.Rxb7 Rxb7 18.Qxd5++–.

8
Ljubojevic, L. – Topalov, V.

544
(2) ■ 32...? –+

Show/Hide Solution

32...Bxf3+! 33.Rxf3 Rg5+ 34.Kh1

34.Rg3 Rxg3+ 35.Qxg3 Qf1#

34...Qxf3+

0-1 Ljubojevic, L (2570) – Topalov, V (2743) Monte Carlo 2003.

9
Milov, V. – Topalov, V.

545
(2) ■ 40...? –+

Show/Hide Solution

40...Qd2! 41.Qxd2 Rxf1+

0-1 Milov, V (2606) – Topalov, V (2745) Prague 2002.


41...Rxf1+ 42.Kg2 Rg1#

10
Nakamura, H. – Topalov, V.

546
(2) ■ 26...? –+

Show/Hide Solution

10

26...Rxc6 27.Nxc6 f5!

Both of White’s knights are hanging! 0-1 Nakamura, H (2780) – Topalov, V (2749) Saint Louis 2017.

11
Mamedyarov, S. – Topalov, V.

547
(2) ■ 27...? –+

Show/Hide Solution

11

27...Nxa2! 28.Rd2

28.Kxa2 Rc4–+

28...Qa4

0-1 Mamedyarov, S (2760) – Topalov, V (2780) Nice 2008.

12
Topalov, V. – Iturrizaga Bonelli, E.

548
(2) □ 43.? +–

Show/Hide Solution

12

43.Nd5!

1-0 Topalov, V (2752) – Iturrizaga Bonelli, E (2627) Istanbul 2012.


43.Nd5 Rxd5 44.Bxd5 exd5 45.Ra5+–

13
Topalov, V. – Bareev, E.

549
(3) □ 23.? +–

Show/Hide Solution

13

23.Nxb5+! axb5 24.Rxb5 Qc6

Position after: 24...Qc6

550
25.Rdxd5! exd5 26.Qe7+ Ka6 27.Rb3!

1-0 Topalov, V (2745) – Bareev, E (2726) Dortmund 2002.

14
Topalov, V. – Gelfand, B.

(3) □ 27.? +–

Show/Hide Solution

14

27.Ng4! Qg5

27...Qf8 28.Nxh6++–

28.f4!

1-0 Topalov, V (2780) – Gelfand, B (2737) Wijk aan Zee 2008. Black resigned in view of 28.f4 Qxh5
(28...Qf5 29.Nxh6++–) 29.Nf6++–.

15

551
Topalov, V. – Naiditsch, A.

(3) □ 26.? +–

Show/Hide Solution

15

26.Re1!

Creating the deadly threat of Qf6+.

26...Raa7

26...h6 loses for many reasons, e.g. 27.Rf3 Qg7 28.Rf6 Kh7 29.Qf5+ Kh8 30.Re8++–.

27.Qf6+!

1-0 Topalov, V (2788) – Naiditsch, A (2612) Dortmund 2005.

16
Ivanchuk, V. – Topalov, V.

552
(3) ■ 40...? –+

Show/Hide Solution

16

40...b3+!

Eliminating the white king’s only escape square!

41.Kd2

41.axb3 Qe2+–+

41...Rd8+ 42.Ke1 Rd1+! 43.Kxd1 Qxf1+ 44.Kd2 Qf4+ 45.Kd1 bxa2–+

553
Position after: 45...bxa2

White doesn’t have a perpetual check, so Black wins! E.g. 46.Qc8+ Kf7 47.Qd7+ Kf6 48.Qd8+ Kf5
49.Qf8+ Ke4 50.Qa8+ Kd3 51.Qxa6+ Qc4 52.Qa7 g3 53.h3 Qb3+ 54.Ke1 Kc2 0-1 Ivanchuk, V
(2746) – Topalov, V (2812) Sofia 2009.

17
Topalov, V. – Beliavsky, A.

554
(3) □ 38.? +–

Show/Hide Solution

17

38.Nh5+! gxh5 39.Rfxf6

Position after: 39.Rfxf6

White has too many threats on the dark squares. Black is defenceless.

39...Nb5

39...Kg8 40.Bxe5 Rxe5 41.Qg3++–

40.Qg5+ Kf8 41.Qh6+ Kg8 42.Bxe5

1-0 Topalov, V (2690) – Beliavsky, A (2618) Batumi 1999. Black resigned in view of: 42.Bxe5 Rxe5
43.Rd8++–.

18
Topalov, V. – Ponomariov, R.

555
(3) □ 32.? +–

Show/Hide Solution

18

32.Nxf6! Bxf6 33.d4!+–

…Bb1. 1-0 (65) Topalov, V (2804) – Ponomariov, R (2738) Sofia 2006.

19
Topalov, V. – Leko, P.

556
(4) □ 55.? +–

Show/Hide Solution

19

55.Nf6! Rxd6 56.Re8+ Kc7 57.Re2+–

…Rxb2/Ne8+. 1-0 (71) Topalov, V (2801) – Leko, P (2740) Linares 2006.

20
Topalov, V. – Lutz, C.

557
(4) □ 27.? +–

Show/Hide Solution

20

27.Nf6+! gxf6 28.Rd8+ Rxd8 29.Rxd8+ Kh7 30.Qf8

Position after: 30.Qf8

558
30...Kg6

30...h5 31.g4!? (31.Qh8+ Kg6 32.Rg8+ Kf5 33.Qxh5+ Kf4 34.Rd8! also wins) 31...hxg4 32.fxg4
Bxg4 (32...Qe4 33.Qh8+ Kg6 34.Rg8#) 33.Qg8+ Kh6 34.Qxg4+–

31.Qg8+

Mate follows after 31.g4!? Kg5 32.Qg7+ Kf4 33.Qxh6+ Kxf3 34.Rd3+ Kxg4 35.Rd4+.

31...Kh5

31...Kf5 32.Qg4+ Ke5 33.f4+ Ke4 34.Qf3++–

32.Qg7!

32.Rd4 with the idea of g3 also wins, but the text move is even stronger as Black doesn’t have
...Qb5−g5.

32...f5 33.Rd4 Bc8 34.g3

1-0 (34) Topalov, V (2745) – Lutz, C (2650) Dortmund 2002


34.g4+!? also wins: 34...fxg4 35.fxg4+ Kh4 36.Qe5+–

21
Topalov, V. – Vallejo Pons, F.

(5) □ 20.? +–

559
Show/Hide Solution

21

20.Bf6+!

The only way to continue the attack.

20...Kxf6 21.fxe5+ Ke7 22.Qh4+ f6

22...Kd7 23.Qxe4+–

23.exf6+ Kd8

23...Nxf6 24.Rxf6 Rxf6 25.Re1+ Kf7 26.Qh7+ Kf8 27.Qe7+ Kg8 28.Qxf6+–

24.Qxe4

24.f7+!? Kc7 25.Qxe4 Bd7 26.Rf6+–

24...Bd7 25.Qe7+ Kc7 26.Rf4!

Position after: 26.Rf4!

Black is busted. White has three pawns and a strong attack.

560
26...Qe8 27.Re1! Qxe7 28.Rc4+! Kb8 29.Rxe7 Bf5 30.Rcc7?

30.f7! Ka7 31.g4 Bg6 32.Rcc7 Rab8 33.h4+–

30...Rxf6 31.Rxb7+ Kc8 32.Rec7+ Kd8 33.Rg7 Kc8 34.Rxb6±

1-0 (62) Topalov, V (2767) – Vallejo Pons, F (2684) Dos Hermanas 2008.

22
Topalov, V. – Ding, L.

(5) □ 64.? +–

Show/Hide Solution

22

64.Rhf8! Rbb2

A) 64...Kh4 65.Rc1+–
B) 64...Bxg2+ 65.Kh2!!+– …Kg3, h4

65.Rc7! g6

65...Bxg2+ 66.Nxg2 Rxg2 67.Rxg7++–

561
66.g3!

Black can’t defend against both h4 and Rc5. 1-0 Topalov, V (2761) – Ding, L (2755) Saint Louis 2016.

23
Topalov, V. – Hansen, C.

(5) □ 62.? +–

Show/Hide Solution

23

62.Bxe7 Rxe7 63.Rf8! Kxh6

Black was in zugzwang: 63...Rd7 64.Kf6+–.

64.Kf6! Re6+ 65.Kxf7 Rxe5 66.Kf6

562
Position after: 66.Kf6

Brilliant! 1-0 Topalov, V (2707) – Hansen, C (2613) Istanbul 2000.

24
Topalov, V. – Polgar, J.

(5) □ 45.? +–

563
Show/Hide Solution

24

45.e6!!

A killing move if it works, isn’t it?

45...Qxe3+

45...h3 46.g3 doesn’t help Black.

46.Qxe3 Rb1+

Position after: 46...Rb1+

47.Qc1! Rxc1+ 48.Kf2 Rc6 49.Rd7

49.d7 also wins.

49...Bb5 50.Ke3+–

With an easy win for White, e.g. 50...Rc2 51.Rc7 Re2+ 52.Kf4 Rf2+ 53.Kxe4 Re2+ 54.Kf5 Rf2+
55.Ke5 Re2+ 56.Kf6 Rf2+ 57.Ke7 Re2 58.d7 Bxd7 59.Kxd7 Rd2+ 60.Ke8 1-0 Topalov, V (2750) –
Polgar, J (2665) Novgorod 1996.

564
Table of Contents
Title page 3
Key to Symbols 4
Preface by Veselin Topalov 5
Chapter 1 – Topalov: A World Apart 7
Chapter 2 – The Start of Our Cooperation 45
Part 1 77
Part 2 103
Chapter 4 – London: first sucess! 136
Chapter 6 – Tough times in Thessaloniki 256
Chapter 8 – Preparing for the Candidates 353
Part 3 454
Part 4 479
Chapter 10 – A Few Novelties More 510
Exercises – Win Like Topalov! 538

565

You might also like